contact us

Use the form on the right to contact us.

I will not respond to requests for medical advice.

         

123 Street Avenue, City Town, 99999

(123) 555-6789

email@address.com

 

You can set your address, phone number, email and site description in the settings tab.
Link to read me page with more information.

Puswhisperer Year 10

Journey or Destination

Sep 6, 2017

Just a small town doc, living in a lonely world

He took the ID train going anywhere

Just a city boy, born and raised in NE PDX

He took the ID train going anywhere

A clinician in an ICU room

A smell of alcohol and cheap quats

For a smile they can order diagnostics

It goes on and on and on and on.

I am all about the destination. I want a diagnosis. But most of the time, I am left with the journey and unknown consult that, despite a careful history and evaluation, remains unknown. But thanks to that series of tubes, I often learn a lot from researching my unknowns. And now so will you.

The patient has a week of nonproductive cough and fevers and is eventually admitted with multilobar, interstitial/patchy pneumonia. Gram stain shows WBCs, no organisms, and cultures, and the respiratory PCR panel yield nothing.

It is the pattern of an atypical pneumonia, and since people get what they are exposed to, I ask an exposure history.

The symptoms began at the end of a trip to Yosemite, where they drove through lots of smoke from forest fires, saw the geysers, and were downwind from, but did not get close to, bighorn sheep and bison.

By the way, as a complete aside, this is tomato season, which is caprese season, and until just this week, I thought Mozzarella was from buffalo, i.e., bison. Nope. I wondered how they milked a bison, but it turns out bison cannot make enough milk for human production; for cheese making, they are an udder failure. It is water buffalo milk. And you probably knew that.

Anyway, the first step was to call the Wyoming Departement of Health, who were very helpful but could report no outbreaks of human pneumonia associated with Yellowstone.

Forrest fire smoke? Nothing on the internet.

Geysers? Curiously, while the waters of Yellowstone are teaming with Legionella.

... using PCR amplification with Legionella-specific primers targeting 16S rRNA genes, detected four known Legionella species, as well as Legionella sequences from species that are not represented in sequence databases, in mat samples and cultivated isolates.

there are no cases of Legionella I can find associated with Yellowstone. And there probably should be. All his Legionella workup was negative. There are other potential pathogens in water, like Simkania negevensi, but I can't test for this. He wasn't that sick outside of the lung, so possibly?

While Bison have Brucella and Mycoplasma, I can't see being far downwind would lead to infection with these organisms.

Q fever? Maybe. Q fever can be airborne and is a problem in Yellowstone Big Horn. Perhaps. Yellowstone Big Horns have had an issue with an unnamed pneumonia (perhaps Mycoplasma or Pasturella, the reports tend to be vague), and there is Q fever in the area.

But the Q fever serology was negative.

In the end, he did fine, probably on his own, although I did suggest a course of quinolone pending all the tests. I doubt it did anything.

The journey was certainly interesting but like an MC (who knew he was a mic controller?) Escher print led back to the start with no clear destination.

Such is medicine.

Rationalization

Appl Environ Microbiol. 2005 Jan;71(1):507-11. Legionella species diversity in an acidic biofilm community in Yellowstone National Park.

https://www.ncbi.nlm.nih.gov/pmc/articles/PMC544253/

BMC Nephrol. 2017 Apr 13;18(1):133. doi: 10.1186/s12882­017­0548­z. Seroprevalence of a "new" bacterium, Simkania negevensis, in renal transplant recipients and in hemodialysis patients.

https://www.ncbi.nlm.nih.gov/pubmed/28407799

Brucellosis and Yellowstone Bison.

https://www.aphis.usda.gov/animal\_health/animal\_dis\_spec/cattle/downloads/cattle-bison.pdf

Bacteriol Rev. 1961 Sep; 25(3): 285_x0013_293. PMCID: PMC441106 AIRBORNE Q FEVER

https://www.ncbi.nlm.nih.gov/pmc/articles/PMC441106/

Bighorn Sheep Near the North Entrance of Yellowstone National Park are Dying from Pneumonia.

http://www.thewildlifenews.com/2014/12/15/bighorn-sheep-near-the-north-entrance-of-yellowstone-national-park-are-dying-from-pneumonia/

POLL RESULTS

I like

  • the journey 13%
  • Journey, but with only with Steve Perry fronting 17%
  • the destination 8%
  • unexpected side trips 42%
  • staying home and letting others travel 17%
  • Other Answers 4%
  • Zappa. Any Zappa.

Threes

Sep 11, 2017

Over the summer vacation, I let my beard grow back. It is 95% white. That's fine. Grey hair is hereditary: you get it from your children. Every medical group advertisement needs an old grey man. I am ready to be typecast. And being old has its advantages. Today, a patient said he could not believe how young all his doctors were, and it was nice to have an old, experienced doctor. I get that more often of late. I was admitted to the surgical service last month (I'm fine), and the housestaff and attendings all seemed especially young from the vantage point of a patient.

They also seemed a bit uncomfortable taking care of me. My wife tells me all the time to stop scowling and that I look angry. I'm not. My natural expression when I am left to myself is evidently the scowl. And I learned this week there is a name for it. Resting Richard Face for men and Resting Witch Face for women. Well, the diminutive of Richard and something that rhythms with witch for women. This is a (quasi) professional blog, after all. But, instead of FACP or some such, perhaps I should put RDF after my MD.

Three patients all with the same bug today. And they were not S. aureus.

First was a liver abscess with bacteremia after a failed stent for biliary obstruction.

S. intermedius in the blood and abscess.

Fine. Been there, seen that.

The second was a pneumonia with empyema in a patient with chronic aspiration after resection of a head and neck tumor. S. intermedius.

Fine. Been there, seen that as well.

The third? A spontaneous deep abscess in the thigh. No risks. I think. S. intermedius.

There is one case in the Pubmeds:

We present a case of primary pyomyositis of the quadriceps in a diabetic patient with a remote traumatic antecedent of the affected zone and caused by an unusual germ.

But my patient? No diabetes. No trauma. Good dentition. No trip to the dentist. Just a large collection of pus in the muscle growing S. intermedius.

The closest I can get to a reason is a new puppy at home. Puppies are all frolic and spittle and jumping and gnawing. Now the patient can't remember a specific instance of trauma, but the microbiology of a dog's mouth is not that different from a human, although most humans do not lick their rear.

The second most common species isolated from dog bite wounds in equal frequencies were Streptococcus and Staphylococcus species (206)... Among members of the genus Streptococcus, the most common species isolated was Streptococcus mitis (22%). Other species, in order of decreasing frequency, included Streptococcus mutans (12%), Streptococcus pyogenes (12%), Streptococcus sanguis biotype II (8%), Streptococcus intermedius (6%), Streptococcus constellatus (4%), Streptococcus equinus (2%), Streptococcus sanguis biotype I (2%), Streptococcus agalactiae (2%), Streptococcus sanguis (2%), beta-hemolytic Streptococcus group G (2%), and Streptococcus dysgalactiae (2%).

So maybe from fido. Maybe not. And has anyone ever named a dog fido?

No matter the source, I can kill a Streptococcus intermedius. All three got better with I&D and a beta-lactam.

Rationalization

Eur J Med Res. 2010;15:319-22. Multiple liver abscesses with isolation of Streptococcus intermedius related to a pyogenic dental infection in an immuno-competent patient.

https://www.ncbi.nlm.nih.gov/pubmed/20696645

Case Reports in Pulmonology Volume 2016 (2016), Article ID 7452161, 5 pages Case Report Streptococcus intermedius Causing Necrotizing Pneumonia in an Immune Competent Female: A Case Report and Literature Review

http://dx.doi.org/10.1155/2016/7452161

An Sist Sanit Navar. 2007 May-Aug;30(2):273-9. [Primary pyomyositis caused by Streptococcus intermedius].

https://www.ncbi.nlm.nih.gov/pubmed/17898821

Clin Microbiol Rev. 2011 Apr; 24(2): 231–246. doi: 10.1128/CMR.00041-10 PMCID: PMC3122494 Microbiology of Animal Bite Wound Infections.

https://www.ncbi.nlm.nih.gov/pmc/articles/PMC3122494/

POLL RESULTS

One day I will name a dog

  • fido 6%
  • spot. But only if a solid color 14%
  • after my ex. 6%
  • pus 11%
  • vector 53%
  • Other Answers 11%
  • Rover.
  • No, I won't. I'll name a cat. And I'll name him Doctor Mark. And he'll have a brain the size of a planet.
  • I have too much respect for dogs to name one after my ex.

Check Another One Off The List

Sep 13, 2017

I have said before that ID is like birding, only interesting. I have my life list of diseases I have seen, have yet to see, and a few (smallpox, measles) I hope never to see.

While Oregon is perhaps the best place on earth to live, it can be dull as a source of Infectious Diseases. We have little in the Great Pacific NW to interest an ID doc. I have to wait for my patients to travel and bring the exotic illnesses home. Exotic for Portland anyway.

Patient is a healthy male who spent a few weeks in the US NE hiking and traveling. He had no bites he could remember, but he had fevers, rigors, sweats, and severe fatigue a week after returning home.

He sought care at Doc in the Box, who called it a viral syndrome and treated it symptomatically. A CBC showed an anemia and thrombocytopenia, and both GI and oncology evaluated him for a reason, neither of which, according to the patient, evidenced much interest in the fevers. I guess fevers are not important to consider in the approach to anemia. For me, it almost always comes down to the why behind the fevers; fever is the singularity from which the multiverse of ID differentials start.

Anyone from the NE, or who is an ID doc, knows where this is going. No one here in PDX did, and it took a phone call to a friend on the East coast to suggest an evaluation for tick bourne (usually Matt Damon is the main vector) illnesses, and the IgM for Babesia was positive.

He was started on atovaquone and azithromycin and sent to me, where I found a perfectly healthy male with no problems or complaints.

Nothing for me to do. Expect.

I hate serologic diagnoses. I want to either grow the bug or see it in the flesh.

So I called the lab that diagnosed the anemia last month when he was at his most ill and had them pull the slide.

Find me my first Babesia on the slide, I commanded imperiously.

And they did—lots of intra-erythrocytic thin ring forms on the slide.

Still, nothing for me to do, but I still had a frisson of joy confirming the diagnosis.

This is not the first time I have confirmed a diagnosis by pulling an old CBC and reviewing it. Only machines look at CBC's these days, and I wonder if this accounts for the decline in relapsing fever this century. It was the tech looking at the smear which usually first made the diagnosis.

But you have to know to look, and that takes an appreciation of fevers after travel. As my patient noted, you need to see the right doctor.

Rationalization

Babesiosis

http://emedicine.medscape.com/article/212605-overview

POLL RESULTS

I have a life list to complete of

  • birds 5%
  • beers 19%
  • infections 16%
  • books 32%
  • concerts. 11%
  • Other Answers 16%
  • food
  • beer
  • Mountains to climb.
  • Urks.

That is no way to die

Sep 18, 2017

As some patients age and develop a disease that will eventually kill them, they occasionally opt for no treatment. It is a reasonable decision.

I recently had a patient in their late 80's with metastatic pancreatic cancer. She decided against treating her cancer. Fair enough. But then she developed Streptococcus gallolyticus subsp. pasteurianus aortic valve endocarditis, perhaps a complication of her gi cancer. Maybe even a seeding of marantic endocarditis.

She didn't want to treat that either.

Man. That is a hard decision to support.

In the US, I rarely get a chance to see the natural history of infections. Between vaccines, antibiotics, and modern health care, there is almost always some disease attenuation. Even if I can't treat the infection, I can modify the symptoms.

Subacute bacterial endocarditis is no way to die. It is slow and symptomatic, and you get emboli and progressive heart failure. I have never watched a patient die of untreated subacute endocarditis, but I know the classic description.

In 1931, Alfred S. Reinhart, a medical student, developed SBE and kept an extensive, detailed disease diary. Endocarditis is no way to die.

So I talked with the patient and told her: I appreciate that you don't want to treat your diseases and the reasons for your decision. We all want a quick and painless end, but that is not what will likely happen with a heart valve infection. Endocarditis is no way to die. It is neither quick nor comfortable and probably will be slow and awful. You might want to reconsider.

And she did. And she was cured. And has done quite well since all things considered.

Rationalization

Clin Microbiol. 2013 Dec; 51(12): 4249–4251. doi: 10.1128/JCM.01709-13 PMCID: PMC3838054 Splenic Abscess Caused by Streptococcus gallolyticus subsp. pasteurianus as Presentation of a Pancreatic Cancer

https://www.ncbi.nlm.nih.gov/pmc/articles/PMC3838054/

Monaldi Arch Chest Dis. 2013 Dec;80(4):189-92. Nonbacterial thrombotic endocarditis in pancreatic cancer.

https://www.ncbi.nlm.nih.gov/pubmed/25087296

CMAJ. 2002 Dec 10; 167(12): 1379–1383. PMCID: PMC137358 Subacute bacterial endocarditis observed: the illness of Alfred S.

https://www.ncbi.nlm.nih.gov/pmc/articles/PMC137358/

The sandfly is the vector but the gerbil is the reservoir.

Sep 20, 2017

There is mansplaining: "a portmanteau of the word man and the informal form splaining of the verb explaining and means "to explain something to someone, characteristically by a man to woman, in a manner regarded as condescending or patronizing."

Lucy had some splaining to do (not really), Ricky had mansplaining.

I hope I never mansplain, but I suspect I am increasingly into idsplaining. I'm not arrogant; I am condescending and patronizing as I explain why vancomycin and piperacillin-tazobactam are so very stupid for acute cellulitis. Again.

The patient hiked Greece and Albania and, after leaving, had a half dozen raised patches about the size of a half-dollar on her skin that looked like psoriatic plaques.

Of course, this being an ID blog, it isn't psoriasis. In the next country, she sought care, and a biopsy was done.

Leishmania. But which one?

Leishmania is in Greece:

visceral leishmaniasis has spread in Greece with autochthonous human cases appearing in 41 of the 54 prefectures. The occurrence of the disease was mapped and related to dog seropositivity, environmental and geospatial risk factors. Average dog seropositivity was 22.1% and positive animals were found in 43 of 54 prefectures.

and is mostly L. infantum and L. tropica

In Albania, it is mostly Leishmania infantum and Leishmania major.

So which one is it? It makes a difference. And that is where the ever-helpful CDC comes in. A biopsy was sent and identified as L. major, an organism with many nicknames: Aleppo boil, Baghdad boil, Bay sore, Biskra button, Chiclero ulcer, Delhi boil, Kandahar sore, Lahore sore, Oriental sore, Pian bois, and Uta. If there is a colloquial name in Albania where she likely had her sandfly bite, I can't find it on the interwebs.

It is a zoonosis of gerbils of all things.

In this study, seasonal variation of natural infection with Leishmania parasites in Rhombomys opimus (Rodentia: Gerbillinae) population of an endemic focus of ZCL in Iran was monitored...The results showed that Leishmania infection rate was 55.8% (29 out of 52 gerbils) using nested PCR. The highest and lowest Leishmania infection rates were observed in fall and summer, respectively. Gerbils that were found to be infected only with L. major were 5.8%, and that with Leishmania turanica were 23.1%.

The two nice things about L. major: it is confined to the skin, it is self-limited, and can be treated with fluconazole.

A six-week course of oral fluconazole is a safe and useful treatment for cutaneous leishmaniasis caused by L. major.

And she got all better on fluconazole.

Rationalization

Am J Trop Med Hyg. 2013 Nov 6; 89(5): 906–915. doi: 10.4269/ajtmh.13-0070 PMCID: PMC3820334 Leishmaniases in Greece

https://www.ncbi.nlm.nih.gov/pmc/articles/PMC3820334/

Genetic diversity and structure in Leishmania infantum populations from southeastern Europe revealed by microsatellite analysis

https://doi.org/10.1186/1756-3305-6-342

N Engl J Med. 2002 Mar 21;346(12):891-5. Fluconazole for the treatment of cutaneous leishmaniasis caused by Leishmania major.

https://www.ncbi.nlm.nih.gov/pubmed/11907288

Bull Soc Pathol Exot. 2010 May;103(2):84-9. doi: 10.1007/s13149-010-0044-1. Dynamics of Leishmania infection rates in Rhombomys opimus (Rodentia: Gerbillinae) population of an endemic focus of zoonotic cutaneous leishmaniasis in Iran.

https://www.ncbi.nlm.nih.gov/pubmed/20390397

Ampicillin Allergy and Listeria

Sep 25, 2017

The elderly patient presents with headache, nausea, and protracted vomiting. She is not coherent, does not have a stiff neck, and has a non-focal exam. Imaging is negative, and her LP shows 98 WBC, a normal glucose, and a protein of 450. Negative gram stain. She is started on vancomycin, ceftriaxone, and TMP-Sulfa with a well-documented allergy to ampicillin.

I am called to see her the next day. It is more an encephalitis than meningitis, and the patient is already better, but the LP bothers me.

The CSF changes in meningitis in order of protein, then WBC, and finally the glucose. She had been sick for at least 24 hours before coming to the hospital, so a slow-growing organism with a more protracted presentation is suggested by the LP. So if bacterial, it should be Listeria, and you have to worry about an epidural abscess with that protein.

We now have a CSF PCR panel that I added to the LP (many are as of yet unaware of its existence), and it, along with the cultures, simultaneously were positive for Listeria.

She is elderly and has cancer in remission, but no dietary indiscretions that are risky for Listeria. She does live in the county, on well water, with horses on one side of their property and a cattle ranch on the other. So who knows what the source is. There is nothing to suggest water as a source for Listeria, so it must have been something she et.

And after 48 hours of tmp/sulfa, she was much better. While ampicillin is considered the treatment of choice

Cotrimoxazole was uniformly successful treatment of human listeriosis in this series.

Given the improvement, I opted to continue the tmp/sulfa rather than challenge with ampicillin.

And I don't know about aminoglycosides. Experts recommend it, and I have to say I have little faith in expert opinion, even, or especially, my own. I still think the three most dangerous words in medicine are "in my experience." As best I can tell, adding gentamicin is based on anecdotes and the plural of anecdote is anecdotes, not data. Physicians do what they do because they do it.

Gentamicin is certainly toxic and not always helpful in patients:

We did not observe differences in mortality of patients who were treated with beta-lactam monotherapy in comparison with those who were treated with beta-lactam/aminoglycoside combination.

Even when combination antibiotics look good in the test tube.

She did fine on the tmp/sulfa alone. So in my experience...

Rationalization

Enferm Infecc Microbiol Clin. 2001 Aug-Sep;19(7):297-303. [Listeria monocytogenes infections in the adult. Clinical and microbiological issues of a changing disease].

https://www.ncbi.nlm.nih.gov/pubmed/11747787

Antimicrob Agents Chemother. 1994 Mar;38(3):438-46. Inhibition of intracellular growth of Listeria monocytogenes by antibiotics.

https://www.ncbi.nlm.nih.gov/pubmed/8203836

POLL RESULTS

The three most dangerous words in medicine are

  • in my experience 36%
  • I lack insurance 6%
  • repeal and replace 28%
  • this won't hurt 6%
  • I'll be fine (husband to wife) 19%
  • Other Answers 4%
  • That's not significant.

Discordant

Sep 27, 2017

The last month or two has seen a deluge of cases of enterococcal endocarditis and liver abscesses. And like snowflakes, no two are alike. Well, that may not be true for snowflakes or liver abscesses, but both have remarkable variation.

One case was typical: middle-aged make with fevers, RUQ pain, history of diverticulitis. I said it was likely S. intermedius group with or without anaerobes. Safe bet. But wrong. The cultures were discordant with my prediction; I hate it when reality doesn't conform with my belief. That's one aspect where medicine differs from popular culture: you can't ignore reality. You can, but you do so at your peril.

Cultures were pure growth Fusobacterium nucleatum. It happens, but it is my first

Forty-eight cases were identified, 41 in men. The median age was 42.5, with an interquartile range of 33. F. nucleatum and F. necrophorum were in involved in 22 cases each, and 4 cases were not further speciated. Among cases of F. nucleatum liver abscess, nine were attributed to periodontal disease, four to lower gastrointestinal tract disease, one to Lemierre's Syndrome, and eight were considered cryptogenic. All patients treated made a full recovery.

My case is cryptogenic, but there is a suggestion that Fusobacterium might be associated with colon adenomas and cancer. Whether a cause or an effect is uncertain, the patient will get a colonoscopy as an outpatient. He was due anyway.

The other was an elderly female with a history of biliary disease with presents with fever and confusion. The blood cultures grow S. intermedius, and as part of the evaluation, a small liver abscess was discovered. It is in a tricky location for drainage, and she is on anticoagulation for a prosthetic valve, so we, and by we I mean I, decide for medical therapy. We have the causative bug in the blood and probably endocarditis with seeding to the liver. After two weeks of therapy, she is clinically worse, and the abscess is bigger. So we bite the bullet and drain it.

This time there are discordant cultures from the blood and liver—pure culture MRSA. Not a Streptococcus to be seen in the abscess. No MRSA bacteremia, no MRSA colonization, a clean TEE. What the? Where did the MRSA come from? No clue.

Cryptogenic liver abscesses from MRSA are rarely reported, so it happens. As I learned at the movies last weekend, you do NOT want It to happen.

On MRSA therapy, she finally improved.

I like everything to be tied up in a nice neat package. It so rarely happens.

Rationalization

Is it really true that no two snowflakes are alike?

http://www.its.caltech.edu/~atomic/snowcrystals/alike/alike.htm

BMC Infect Dis. 2017 Jun 20;17(1):440. doi: 10.1186/s12879-017-2548-9. Fusobacterial liver abscess: a case report and review of the literature.

https://pubmed.ncbi.nlm.nih.gov/28633639/

Fusobacterium Is Associated with Colorectal Adenomas Amber N. McCoy, Félix Araújo-Pérez, Andrea Azcárate-Peril, Jen Jen Yeh, Robert S. Sandler, Temitope O. Keku Published: January 15, 2013https://doi.org/10.1371/journal.pone.0053653

http://journals.plos.org/plosone/article?id=10.1371/journal.pone.0053653

Genome Res. 2012 Feb; 22(2): 299–306. doi: 10.1101/gr.126516.111 PMCID: PMC3266037 Fusobacterium nucleatum infection is prevalent in human colorectal carcinoma.

https://www.ncbi.nlm.nih.gov/pmc/articles/PMC3266037/

J Investig Med High Impact Case Rep. 2016 Jul-Sep; 4(3): 2324709616660576. Published online 2016 Aug 3. doi: 10.1177/2324709616660576 PMCID: PMC4974568 Community-Acquired Methicillin-Resistant Pyogenic Liver Abscess.

https://www.ncbi.nlm.nih.gov/pmc/articles/PMC4974568/

Did Not Believe it at First

Oct 2, 2017

First a complaint. Or two. Again. But I like to winge.

In the internet era, I do not need to know the details of a study like I did when I had to slog through the Index Medicus to find a reference. I know the basic result like fosfomycin has been used with success for prostatitis, and it was in CID. But what dosing and for how long? I don't remember that. I don't need to. I can google it. In fact, I just did. It took me about 30 seconds to find Fosfomycin for Treatment of Prostatitis: New Tricks for Old Dogs sticking "clinical infectious diseases fosfomycin prostatitis" into Google. Piece of cake.

I often get called to answer a question when I am nowhere near a computer. I will tell the calling HCW you can use drug x for disease y, and the paper is in journal Z if you want the details. And increasingly, they want me to look it up for them.

Really. Dude. Or Dudette. Since I am in a teaching hospital, I often finish up a consult note with a few key references from PubMed pasted into the note. I think all consultants should do that. But I am not a librarian's assistant. I don't mind answering your curbside, but I am not going to do your internet scut. Google just isn't that hard.

I often say that while you cannot always believe the government, your kids, your spouse, or parents, you can always believe the cultures, at least if you know how to translate what the cultures are saying. Recently I didn't believe the cultures or what they were saying.

The patient is an elderly male, no important comorbidities admitted with fever, weakness, and altered mental status. The chest x-ray report called it pneumonia, but the blood cultures grew E. coli, and they called me.

Huh. I have never seen a community-acquired E. coli pneumonia. No such thing, I said. What I suspected is that no one actually looked at the chest x-ray.

Here is an awful secret in modern medicine. While it has never been easier to look at an x-ray (the one great aspect of the EMR), about half the time, no one does. They just read the report. And fewer bother to go down to radiology to review the films with the radiologist. I look at every film and, in the case of CT's or MRI's, I do it with a radiologist.

But when it comes to chest x-rays, there are infiltrates, and there are infiltrates, and you need to look at the film in the context of the rest of the patients presentation to know if what you are seeing is indeed a pneumonia and, if so, what kind. The dense consolidation of bacteria, the patchy infiltrate of an atypical, etc. etc. But as JRR Tolkein noted

All that is gold does not glitter,

Not all those who wander are lost;

Not every infiltrate is pneumonia,

Look at the x-ray, you fool of a Took.

I looked at the CXR. Yep. A nicely consolidative pneumonia, and while the patient had some shortness of breath, there was no productive cough or chest pain.

I looked it up. Myself. E. coli community-acquired pneumonia is a thing:

representing 0.4% of CAP cases admitted. Main symptoms were fever and dyspnoea. 18 patients were classified into class IV and class V of the Pneumonia Severity Index (PSI). Diagnosis was based on blood culture in 24 cases, PSB in 4 cases and by TNA in 1 case. Three of the patients died, the longer time evolution of the symptoms being the only factor related to higher mortality (p<0.05). data-preserve-html-node="true" Mean hospitalization time was 7.1+/-3.1 d, and correlated with severity at admission (r=0.43; p<0.003). data-preserve-html-node="true" This study demonstrates that CAP caused by E. coli is infrequent. It has an unspecific presentation and mortality rate is 10.3%, associated with longer time before admission to hospital.

I felt like I had to order an abdominal CT looking for a more typical source of E. coli bacteremia. Clean.

E. coli CAP is was. And no reason I could find. The cultures don't lie.

Rationalization

Scand J Infect Dis. 2008;40(5):424-7. doi: 10.1080/00365540701732913 Escherichia coli: an unknown and infrequent cause of community acquired pneumonia.

https://www.ncbi.nlm.nih.gov/pubmed/18418804

POLL RESULTS

I will not

  • lookup your references 7%
  • write your notes 19%
  • send you flowers 15%
  • be your scut monkey 41%
  • follow your suggestions. 4%
  • Other Answers 15%
  • I won't deny you your learning opportunity.
  • All of the above.
  • All of the above
  • use Google. Google is evil. https://duckduckgo.com/ is your real friend.

No other options? I think not.

Oct 4, 2017

The patient is admitted for an esophageal perforation and an empyema.

She has an esophageal stent placed to repair the tear and a VATS. The blood grows Lactobacillus and Bifidobacterium. The pleural space grows Candida albicans. They call me for the final antibiotic choice, and I see the patient is on fluconazole, ciprofloxacin, and metronidazole.

Ciprofloxacin. What is it about ciprofloxacin? People do love to give ciprofloxacin. And quinolones are great drugs, but there is the whole issue of toxicity. Leave aside the issue that no gram negatives were found in the blood and pleural space. And ignore the fact that ciprofloxacin is not what one would give for either Lactobacillus and Bifidobacterium.

Leave aside the driving of C. difficile and MRSA. And forget about the issues of qt prolongation and drug-drug interactions with fluconazole.

Twenty-one (22.3%) of the studied patients had clinically significant changes in the QTc while receiving combination fluoroquinolone-azole therapy.

All are excellent reasons for giving an antibiotic other than cipro. No, forget about all that. She has a hole in her esophagus that needs healing, and I would add to the list the worry about slowing healing of wounds due to the quinolones. Perhaps the least important issue with using a quinolone, but there is the whole literature that quinolones really mess with connective tissues. Everyone, or at least everyone who matters, know quinolones are associated with tendon rupture.

But there is aortic aneurysms and ruptures:

Fluoroquinolones are associated with subsequent tendon ruptures and may also contribute to aortic aneurysms.

retinal detachment,

Current oral fluoroquinolone use was associated with an increased risk for RD, including the rhegmatogenous and exudative types.

carpel tunnel,

We conducted a case-control study of over 6 million patients. FQ use is associated with increased risk of CTS (RR = 1.34, 95% CI 1.31 - 1.37).

slowed healing of bone

These data suggest that experimental fractures exposed to therapeutic concentrations of ciprofloxacin in serum demonstrate diminished healing during the early stages of fracture repair. The administration of ciprofloxacin during early fracture repair may compromise the clinical course of fracture-healing.

and slowed cornea healing

The ciprofloxacin eyes were significantly more prone to impaired or delayed wound healing and to the development of corneal haze.

With an esophagus that needs to repair, while not proven, the healing could be slowed by the ciprofloxacin.

There are a lot of reasons why the answer to the choice of antibiotics should not be quinolone. The FDA says to avoid them for uncomplicated infections.

... advising that fluoroquinolones should be reserved for these conditions only when there are no other options available due to potentially permanent, disabling side effects occurring together.

I try and avoid them for complicated infections as well. In this case? Amoxicillin is just fine.

Rationalization

FDA updates warnings for fluoroquinolone antibiotics. https://www.fda.gov/newsevents/newsroom/pressannouncements/ucm513183.htm

Effect of Combined Fluoroquinolone and Azole Use on QT Prolongation in Hematology Patients.

http://aac.asm.org/content/57/3/1121.full J Cataract Refract Surg. 2000 May;26(5):690-4.

Epithelial healing rates with topical ciprofloxacin, ofloxacin, and ofloxacin with artificial tears after photorefractive keratectomy. J Bone Joint Surg Am. 2000 Feb;82(2):161-73. Ciprofloxacin inhibition of experimental fracture healing.

https://pubmed.ncbi.nlm.nih.gov/10831898/

Clin Infect Dis. 2017 Apr 24. doi: 10.1093/cid/cix362. [Epub ahead of print] Fluoroquinolone Use and Risk of Carpal Tunnel Syndrome: A Pharmacoepidemiologic Study.

https://pubmed.ncbi.nlm.nih.gov/28444196/

BMJ Open. 2015 Nov 18;5(11):e010077. doi: 10.1136/bmjopen-2015-010077. Fluoroquinolones and collagen associated severe adverse events: a longitudinal cohort study.

https://bmjopen.bmj.com/content/5/11/e010077

JAMA Ophthalmol. 2016 Apr;134(4):415-21. doi: 10.1001/jamaophthalmol.2015.6205. Association Between Oral Fluoroquinolone Use and Retinal Detachment.

https://pubmed.ncbi.nlm.nih.gov/26967005/

In the damndest places

Oct 9, 2017

The patient had a spider bite on the back of his thigh many years ago. I do not believe in spider bites, at least outside of Peter Parker. People never see said spider. And they always think it was a brown recluse. Sorry. The brown recluse lives nowhere near the great Pacific NW.

the known range of the brown recluse spider (Loxosceles reclusa) extends from Nebraska to Ohio and across the south from Texas to Florida....Although brown recluse bites are claimed in states outside of these regions, these bites, as well as those from within the recluse’s range, are most commonly a result of other medically related causes such as bacterial infections of the skin.

As far as I can tell, venomous spider bites in the Great Pacific NW are not a thing:

This study compiled 33 verified spider bites from the state of Oregon (USA). The initial goal was to amass a series of bites by the hobo spider to assess whether it possesses toxic venom, a supposition which is currently in a contested state. None of the 33 bites from several spider species developed significant medical symptoms nor did dermonecrosis occur.

Although no one will believe me. Your local milage will vary.

Still, whatever the skin lesion was became black and required surgical intervention. And it recurred several months later, again becoming necrotic and requiring debridement.

After that, it came back every 6 months, a cellulitis that burned and eventually receded on its own. She lived with it for years, and when she was finally referred to me, there was a patch of blisters and erythema sitting right above the old scar.

It looked like HSV to me, and cultures grew HSV 2. Neither she nor her husband had symptomatic HSV. I suspect spiders do not carry HSV.

So was this always HSV? HSV can cause retinal and CNS necrosis, but skin? I can find one case on Pubmed and some cases in a book called Skin Necrosis. That must have made for a great dinner time conversation while it was being written. But they were all in immunoincompetent hosts; she is healthy as a horse.

So I wonder if the initial necrotic lesion was an MRSA; the records are from a hospital far away and lost in time, so I will never know.

That would make the HSV nosocomial. The classic transmission is from patient to HCW fingers with resultant Whitlow, although there are reports of various other nosocomial HSV transmissions.

We don't see much nosocomial transmission of HSV anymore. Hand hygiene probably helps, and everyone seems to wear gloves in my hospital when they touch a patient regardless of the isolation status.

The HSV is next to the old I&D scar, not on the I&D scar, suggesting, at least to me, that the HSV came later. There are a smattering of wound infections from HSV in the PubMeds. HSV can show up in the damndest places.

As is so often the case, more questions than answers, more speculation than hard facts. She improved rapidly on acyclovir and is probably heading for lifetime suppression.

Rationalization

Toxicon. 2014 Jun;84:51-5. doi: 10.1016/j.toxicon.2014.03.009. Epub 2014 Apr 13. Verified spider bites in Oregon (USA) with the intent to assess hobo spider venom toxicity.

https://www.ncbi.nlm.nih.gov/pubmed/24726469

J Dermatol. 2013 May;40(5):411-3. doi: 10.1111/1346-8138.12096. Epub 2013 Feb 18. Severe cutaneous necrosis due to mixed infection with herpes simplex virus and fungi in an unrelated cord blood stem cell transplantation recipient.

https://www.ncbi.nlm.nih.gov/pubmed/23414251

J Arthroplasty. 2003 Jun;18(4):516-8. Herpes simplex virus causing superficial wound infection in total hip arthroplasty.

https://www.ncbi.nlm.nih.gov/pubmed/12820099

AANA J. 1990 Feb;58(1):8-13. Herpetic whitlow: an occupational hazard.

https://www.ncbi.nlm.nih.gov/pubmed/2316323

Rate Au Vent

Oct 11, 2017

Time to complain or perhaps, in medical terminology, vent my spleen.

Made rounds as usual. In the middle of the night, a patient I was treating for endocarditis became more short of breath. No fevers, no increase in WBC, and a CXR that, to my eye, was all CHF.

So, of course, antibiotics had to be broadened. By broadened, I mean the antibiotics she needed were mostly replaced with antibiotics that were not only not needed but were of lessor efficacy for the primary infection.

Figures.

No harm was done, as I found out within 8 hours. But.

I have concluded that antibiotics should be like cancer chemotherapy agents and limited to those who know what they are doing.

Some channeling of my inner Rumsfeld. There are those things in medicine that are absolutes. MSSA should be treated with a beta-lactam. There are those things where there is room for discussion. I don't really know how best to treat MRSA. I have a qualified opinion.

And then there is the practice of medicine that flies in the face of known medical reality. What adjective to use? Naturopathic? Stupid? How about let's be nice and call it suboptimal. I know that nothing in medicine is 100%. There are exceptions to every rule. But sometimes, it seems the exception has become the rule.

I have mentioned these before in one blog or another, but I am a grumpy old man who likes to kvetch at length. So in honor of JAMA, I give you the 2017 Update on Medical Suboptimal.

Stop giving metronidazole for acute aspiration, especially in the patient with no teeth, like my patient mentioned above. No teeth equals no anaerobes.

For patients in the ICU admitted with pneumonia, there is a high occurrence of inappropriately prescribed anaerobic antibiotics,

Stop giving quinolones for streptococcal infections instead of a beta-lactam.

Stop treating asymptomatic bacteriuria. And altered mental status and increased spasticity are NOT UTI symptoms.

Stop referring to antibiotics as strong, big gun, or powerful.

Stop calling a temperature below 100.4 a low-grade temperature. It is a normal temperature.

Stop treating S. aureus bacteremia with oral clindamycin. In fact, stop treating S. aureus bacteremia. Call an ID doc; at least they know what to do.

Look at the CXR yourself before starting antibiotics for 'pneumonia.' Much of the time, there will be no pneumonia.

Stop ordering an ECHO to rule out endocarditis in patients with unexplained fevers as the only sign.

Quit starting vancomycin for 1/4 gram-positive cocci in the blood on day two.

Quit giving vancomycin and piperacillin/tazobactam for acute cellulitis. They need cefazolin at most.

I can come up with more stupidities if had the time, but I try and limit the time spent on this blog to an hour, and yes, I know it shows.

BTW: for regular readers (those not on laxative). I am taking a huge amount of time off 16 days in a row, the most ever. See you in November. Maybe I will be less grumpy when I get back. Doubt it.

Rationalization

2017 Update on Medical Overuse.

https://jamanetwork.com/journals/jamainternalmedicine/article-abstract/2655246

Naturopathic Diaries.

https://www.naturopathicdiaries.com/

Respirology. 2017 Jul 4. doi: 10.1111/resp.13111. Anaerobic antibiotic usage for pneumonia in the medical intensive care unit.

https://www.ncbi.nlm.nih.gov/pubmed/28677255

I Have Returned

Oct 30, 2017

Back from vacation. And your terrible emptiness of being without rubor, dolor, calor, and tumor can be filled.

16 days off in a row. The most ever, not counting various surgeries, in my career. No EMR, no journals, no calls, no pages, and barely any email. No internet for most of the trip. I was surprised I did not miss it. At all. But hiking every day in the canyon country of Utah and Arizona is so mind-boggling amazing who has time to consider ID. I highly recommend it before the west burns. My only complaint is Utah is no good at making, or even offering, cocktails and beer. Can't have everything.

Time and tides wait for no man. Or perhaps I should say And te tide and te time þat tu iboren were, schal beon iblescet. The pus, it keeps on a flowing.

The patient survives Fournier's gangrene but is readmitted two months later with pelvic pain. The CT shows gas in the PS joint and osteomyelitis around the symphysis pubis.

The symphysis pubis is a weird joint, or, like the sacroiliac, a weird pseudo-joint. Like oregano.

There is this whole unsatisfying literature on inflammation of the PS joint, called osteitis pubis, that occurs in athletes and postpartum. It is thought to be an overuse syndrome and not an infection, but I have always wondered. Perhaps it is more akin to the (admittedly controversial) idea that some herniated discs get secondarily infected with low pathogenic potential organisms that also get better on their own. Some bacterial infections do resolve on their own without antibiotics. But there has never been a systematic prospective evaluation of the syndrome that I can find. It is too rare, so it is case series, aka unhelpful hodgepodge.

The few PS infections I have seen have been complications of GU procedures and so obvious. E. coli is not subtle, even in a PS. In this case, one of the Fournier's organisms has managed to get into the PS joint and adjacent bone. But which one?

My definition of futile is treating chronic osteomyelitis in a smoking diabetic with no debridement and empiric antibiotics. I have submitted the definition to the OED for their consideration. I am the professor in this metaphor btw. But, like the SI joint, try and find a surgeon to operate on it. No one has experience with the joint in smaller hospitals, so I have to transfer him for definitive diagnosis and treatment.

Rationalization

Acta Chir Orthop Traumatol Cech. 2016;83(6):411-417. [Infectious Inflammation of Pubic Symphysis (Symphysitis Pubis Purulenta): Five Case Reports and Literature View].

https://www.ncbi.nlm.nih.gov/pubmed/28026738

World J Orthop. 2015 Oct 18;6(9):672-9. doi: 10.5312/wjo.v6.i9.672. eCollection 2015 Oct 18. Osteitis pubis in elite athletes: Diagnostic and therapeutic approach.

https://www.ncbi.nlm.nih.gov/pubmed/26495244

Acta Orthop Belg. 2006 Oct;72(5):541-8. Osteitis or osteomyelitis of the pubis? A diagnostic and therapeutic challenge: report of 9 cases and review of the literature.

https://www.ncbi.nlm.nih.gov/pubmed/17152416

South Med J. 1985 Feb;78(2):213-4. Postpartum osteitis pubis

https://www.ncbi.nlm.nih.gov/pubmed/3975721

BMC Med. 2015 Jan 22;13:13. doi: 10.1186/s12916-015-0267-x. Could low grade bacterial infection contribute to low back pain? A systematic review.

https://www.ncbi.nlm.nih.gov/pubmed/25609421

The Professor and the Madman: A Tale of Murder, Insanity, and the Making of the Oxford English Dictionary. Simon Winchester.

"The Professor and the Madman is an extraordinary tale of madness, genius, and the incredible obsessions of two remarkable men that led to the making of the Oxford English Dictionary—and literary history."

I got nothing. Which is something

Nov 1, 2017

There is one consult where I know in advance that I will have nothing to add to the patients care: recurrent UTIs. I will see the patients. I'll see anyone who wants to make an appointment (except delusions of parasitism, I'm not a psychiatrist), but I make sure they know in advance that I am unlikely to have much to offer.

Recurrent UTIs are not an ID issue, it is a urologic/anatomic issue, and I can't fix an anatomical problem, certainly not with antibiotics. And none of the 'medical' interventions do anything.

Except.

It turns out drinking 12 cups of water a day decreases UTIs in postmenopausal women by 50% over those that drank 6 cups a day. But in truth, that's an anatomical intervention. Unless bacteria are sensitive to hyponatremia.

Today, the consult was an elderly nutritionally bankrupt male with COPD/bronchiectasis, no teeth, who keeps being admitted with 'pneumonia' due to aspiration events. The presentation is mostly increased SOB, cough, and secretions after he chokes on food or fluids. Because he is in a nursing home, he keeps being treated for HAP.

It is the CXR and CT's that are interesting. They don't have any lobar consolidated infiltrates, just some faint, patchy ground-glass pneumonitis.

Housestaff are well aware of my take, even if I am consistently ignored: an aspiration event usually does not result in an aspiration pneumonia. In the old days, aspiration pneumonia meant an indolent anaerobic lung abscess/empyema in patients who lose consciousness and have bad dentition.

The term aspiration pneumonia has lost all clinical meaning. HCW's treat all aspiration events like the classic description even though anaerobes are not a player in the pneumonia:

For patients in the ICU admitted with pneumonia, there is a high occurrence of inappropriately prescribed anaerobic antibiotics, the use of which was associated with a longer ICU LOS.

But all bacterial pneumonias are likely due to aspiration; that is how the organisms get into the lower respiratory tract.

I want to ban the use of the term aspiration pneumonia, it is meaningless as to etiology, and it just confuses people.

As best as can be determined, in patients who aspirate after stroke, antibiotics do nothing to prevent pneumonia.

Antibiotic prophylaxis cannot be recommended for prevention of post-stroke pneumonia in patients with dysphagia after stroke managed in stroke units.

And I suspect it is true of all acute aspirations.

Now I realize just how difficult it can be to diagnose pneumonia accurately, and safe remains better than sorry. But the long course of broad-spectrum antibiotics has no utility if there is no consolidative pneumonia on the CXR.

Part of my grouchy opinion that this century antibiotics should have the same status as cancer chemotherapy agents and limited to those who know what they are doing.

Me.

So I told them I got nothing. Antibiotics will not treat or prevent aspiration events. I guess that is something.

Rationalization

Drinking More Water Reduces Repeat Urinary Tract Infections.

https://www.medscape.com/viewarticle/886775

Lancet. 2015 Nov 7;386(10006):1835-44. doi: 10.1016/S0140-6736(15)00126-9. Epub 2015 Sep 3. Prophylactic antibiotics after acute stroke for reducing pneumonia in patients with dysphagia (STROKE-INF): a prospective, cluster-randomised, open-label, masked endpoint, controlled clinical trial.

https://www.ncbi.nlm.nih.gov/pubmed/26343840

J Crit Care. 2015 Feb;30(1):40-8. doi: 10.1016/j.jcrc.2014.07.011. Epub 2014 Jul 22. Aspiration pneumonia: a review of modern trends

https://www.ncbi.nlm.nih.gov/pubmed/25129577

Respirology. 2017 Nov;22(8):1656-1661. doi: 10.1111/resp.13111. Epub 2017 Jul 4. Anaerobic antibiotic usage for pneumonia in the medical intensive care unit.

https://www.ncbi.nlm.nih.gov/pubmed/28677255

Plus ça change, plus ça change

Nov 6, 2017

Endocarditis is booming, thanks to the opioid epidemic. There have been at least a 12 fold increase in cases, and where it used to be a disease of males, now females are approaching equality. At any given time, I have 3 or 4 patients in the hospital with no resources, getting antibiotics for their S. aureus endocarditis.

It may be my most common consult the last few years.

What has also been booming the last few months has been enterococcal endocarditis.

This is a problematic bug that used to pester old men with big prostates. Like the consults, both of whom had urosepsis and both of whom had a prosthetic valve and E. faecalis in the blood.

One was treated for two weeks at Outside Hospital and immediately relapsed, although the TEE remained negative. The other? I am not giving it a chance to relapse.

When to worry that enterococcal urosepsis is more? In the old days (1988)

Endocarditis was identified in association with 12 of 35 community-acquired bacteremias, but only 1 of 118 bacteremias acquired in the hospital (P less than .001). Endocarditis was also significantly associated with pre-existent valvular heart disease and cryptogenic bacteremia, and was negatively associated with polymicrobial enterococcal bacteremia

But more recent studies suggest less gender difference, nosocomial endocarditis is increasing, and the importance of a prosthetic valve:

Enterococcal endocarditis can no longer be considered exclusively a unimicrobial, community-acquired disease of Caucasian men. Instead, our data suggest that the presence of a PV and infection by E. faecalis are associated with an increased risk for IE.

And bacteremia with no urinary source, as I have had with several other enterococcal endocarditis patients?

There is bowel cancer, which

was found in more than half of patients with EFIE and an unclear focus of infection who underwent colonoscopy.

More than half! That is a higher risk than the popular S. bovis. Most are still awaiting their colonoscopy, although one does have a funky focal bowel lesion on CT.

Odd diseases, like endocarditis, become more common while HIV OI's, which made up half of my practice in 1990, have all but vanished.

The French had it wrong. The more things change, the more things change. Part of what makes ID so much fun.

Rationalization

Hospitalizations for Endocarditis and Associated Health Care Costs Among Persons with Diagnosed Drug Dependence — North Carolina, 2010–2015

https://www.cdc.gov/mmwr/volumes/66/wr/mm6622a1.htm?s\_cid=mm6622a1\_w

Increasing Infectious Endocarditis Admissions Among Young People Who Inject Drugs

https://www.ncbi.nlm.nih.gov/pmc/articles/PMC5084714/

Infection. 2004 Apr;32(2):72-7. Risk factors for infective endocarditis in patients with enterococcal bacteremia: a case-control study.

https://www.ncbi.nlm.nih.gov/pubmed/15057570

Rev Esp Cardiol (Engl Ed). 2017 Jun;70(6):451-458. doi: 10.1016/j.rec.2016.10.013. Epub 2017 Mar 9. Relationship Between Enterococcus faecalis Infective Endocarditis and Colorectal Neoplasm: Preliminary Results From a Cohort of 154 Patients.

https://www.ncbi.nlm.nih.gov/pubmed/27916708

POLL RESULTS

I

  • embrace change 11%
  • hate change. I am just too damn old to do something new 15%
  • hate EPIC upgrades with the new interface change destroying my hard-won productivity 11%
  • hate it when my grocery store changes where the food is 56%
  • don't worry about change as I only have currency. 7%

More is not better

Nov 8, 2017

“Two great tastes that taste great together." That might work for Reese's Peanut Butter Cups, but putting two medical conditions together is rarely beneficial.

The first is the bacteremia of life. Or at least of the dental life. To brush and floss is to be bacteremic:

Positive blood cultures were detected in 29.6% of patients after dental extraction, in 10.8% of patients after tooth brushing and in no patients after chewing.

It is why I do neither.

Clean teeth become colonized teeth, and the first colonizer is Streptococcus gordonii which

has a high affinity for molecules in the salivary pellicle (or coating) on tooth surfaces. S. gordonii therefore can rapidly colonize clean tooth surfaces, and S. gordonii along with related organisms comprise a high percentage, up to 70%, of the bacterial biofilm that forms on clean tooth surfaces

So I guess that good dental hygiene would lead to more, not less, Streptococcus gordonii in the mouth and then bacteremia. Unless mouthwashes are used, which are quite good at killing S. gordonii.

The other is hypertrophic cardiomyopathy. The anterior mitral valve leaflet whacks against the bulging wall leading to trauma and thickening, just what an oral Streptococcus likes to glom on to.

Mitral endocarditis complicating hypertrophic cardiomyopathy occurs predominantly on the left ventricular aspect of the anterior mitral valve leaflet in the presence of outflow tract obstruction.

because

The long mitral leaflets seen in HCM patients and the centrifugal effect of the turbulent flow in the left ventricular outflow tract present in up to two thirds of HCM individuals render these patients more susceptible to IE due to erosion of the endocardium layer.

Combine the two? You get three weeks of fevers, chills, sweats, fatigue, and a sustained bacteremia with Streptococcus gordonii, aka endocarditis.

No vegetation on the TTE and no outflow obstruction, but it is endocarditis. Nothing else gives a sustained bacteremia, and hypertrophic cardiomyopathy is a risk for endocarditis even in the absence of outflow obstruction. And his teeth look great.

While Streptococcus gordonii is a known cause of endocarditis, there are no reported cases I can find in hypertrophic cardiomyopathy. So I call first. Again. I wonder how many firsts I have reported over the years.

There are but a smattering of endocarditis cases reported specifically by Streptococcus gordonii, likely as it has been identified as S. sanguis group. I only know it is an S. gordonii from the MALDI-TOF. But there are differences:

By means of recently described physiological schemes, the most common species identified (in endocarditis) were Streptococcus sanguis sensu stricto (31.9%), S. oralis (29.8%) and S. gordonii (12.7%)

Only the ID doc cares about the real name of the organism. But, as you learn in fantasy novels, knowing the true name of something gives you power over it.

But I can still kill a Streptococcus gordonii even if I don't know its name.

Rationalization

Cardiovasc J Afr. 2012 Jul; 23(6): 340–344. doi: 10.5830/CVJA-2012-016 PMCID: PMC3734757 An investigation of the frequency of bacteraemia following dental extraction, tooth brushing, and chewing.

https://www.ncbi.nlm.nih.gov/pmc/articles/PMC3734757/

Streptococcus gordonii.

https://en.wikipedia.org/wiki/Streptococcus\_gordonii

Infective endocarditis in hypertrophic cardiomyopathy A multicenter, prospective, cohort study.

https://www.ncbi.nlm.nih.gov/pmc/articles/PMC4937928/

Mitral valve endocarditis in hypertrophic cardiomyopathy: case report and literature review. Heart. 2002 Jun; 87(6): e8.

https://www.ncbi.nlm.nih.gov/pmc/articles/PMC1767125/

J Investig Clin Dent. 2014 May;5(2):151-3. doi: 10.1111/jicd.12088. Epub 2014 Mar 7. Effective oral health in infective endocarditis: efficacy of high-street mouthwashes against the viridans group streptococci.

https://www.ncbi.nlm.nih.gov/pubmed/24610586

J Med Microbiol. 1993 Sep;39(3):179-82. Identity of viridans streptococci isolated from cases of infective endocarditis.

https://www.ncbi.nlm.nih.gov/pubmed/8366515

The Name of the Wind.

https://en.wikipedia.org/wiki/The\_Name\_of\_the\_Wind

So Many Possibilities

Nov 13, 2017

The patient has issues with opiates, having a long history of recalcitrant heroin use.

This time he is in the hospital for right-sided endocarditis, complete with septic pulmonary emboli and a vegetation on the tricuspid valve by TTE.

After several weeks of inpatient nafcillin where he occasionally used heroin, he went to a nursing home to finish his antibiotics.

Given that endocarditis is 100% fatal, I remain of the opinion that I would rather keep antibiotics going despite drug use as dead people can't turn their lives around. The occasional patient has gone on to get the monkey of addiction off their backs.

However, he comes back 48 hours later with fevers, partial loss of vision in one eye, clot around the picc, and Pantoea in the blood.

TEE this time shows the vegetation is unchanged, but they find an ASD.

So a paradoxical embolus from?

Righted sided endocarditis. I suppose. Emboli can occur late in therapy, although I can't find a reference. I have seen a fatal embolism on the last day of therapy.

Is it the S. aureus, or is this new endocarditis from the Pantoea? Seeding of vegetations by new infections does occur; endocarditis is a risk for endocarditis. I do not think so as only one of 4 bottles grew the organism, and the fevers resolved after discontinuing the picc.

From the arm clot? It happens but is usually not clinically relevant, although tiny clots to the eye have more consequences than a tiny clot to the lung.

From being infected?

Infection was the most common trigger of hospitalization for VTE, occurring in 52.4% of the risk periods prior to hospitalization. The adjusted incidence rate ratios (IRR; 95% CI) were 2.90 (2.13, 3.94) for all infection, 2.63

From inorganic material in the heroin?

Changes in the content of impurities, adulterants and diluents are described for 383 samples of illicit heroin seized in the western part of Denmark during the 12-year period 1981 through 1992. A wide range in purity was found within each year, whereas the average purity did not vary much from one year to another. The average purity of wholesale samples (45%) was only slightly higher than the purity of retail samples (36%). The SW Asian type of heroin, containing high concentrations of noscapine, predominated from the mid-eighties. Heroin base and heroin hydrochloride each accounted for approximately half of the samples during the eighties. However, in recent years the base form has become predominant. During the early eighties caffeine and procaine were the most frequent additives next to sugars. During the middle and late eighties an increasing number of heroin samples were cut with phenobarbital and methaqualone. During the early nineties the occurrence of phenobarbital and methaqualone has decreased, whereas paracetamol in combination with caffeine has become predominant.

There are so many possibilities for the source of an embolus that, given the vegetation's size (0.9 x 1.1), it is hard to push for surgery to remove the vegetation. However, it would be nice to close the ASD for all the above reasons.

So, for now, finish the course of antibiotics and see what happens. He left AMA, never to be seen again, and presumed dead. An all too common outcome.

Rationalization

Vascular. 2008 Mar-Apr;16(2):73-9. Internal jugular, subclavian, and axillary deep venous thrombosis and the risk of pulmonary embolism.

https://www.ncbi.nlm.nih.gov/pubmed/18377835

Triggers of Hospitalization for Venous Thromboembolism https://doi.org/10.1161/CIRCULATIONAHA.111.084467 Circulation. 2012;CIRCULATIONAHA.111.084467

https://www.ahajournals.org/doi/10.1161/CIRCULATIONAHA.111.084467

Drug Test Anal. 2011 Feb;3(2):89-96. doi: 10.1002/dta.220. Epub 2010 Dec 29. Adulterants in illicit drugs: a review of empirical evidence.

https://pubmed.ncbi.nlm.nih.gov/21322119/

Forensic Sci Int. 1994 Feb;64(2-3):171-9. Impurities, adulterants and diluents of illicit heroin. Changes during a 12-year period.

https://pubmed.ncbi.nlm.nih.gov/8175088/

J Intern Med. 2012 Jun;271(6):608-18. doi: 10.1111/j.1365-2796.2011.02473.x. Epub 2011 Dec 8. Acute infections and venous thromboembolism.

https://pubmed.ncbi.nlm.nih.gov/22026462/

Veni, vidi, vici

Nov 15, 2017

I lean towards simplicity. Less is almost always better. Except beer and ice cream. But for the medical record, less is almost certainly better unless the goal is to snow your referral base with BS. A good note should be haiku, not the Odyssey.

I am, as I have said many times, an Occam's kind of guy.

Non sunt multiplicanda entia sine necessitate.

Fer sure dude.

The longer I do ID, the simpler it becomes. Me find bug, me kill bug, me go home. Or, as Google translate would have it, Invenient mihi cimex occidere me cimex mihi vade in domum tuam.

ID docs seem to wallow in the idea that they, and only they, do these uber complete patient evaluations as if it is a beneficial attribute.

Sorry. Most consults can be done in about 15 minutes, start to finish, if you know what you are doing and know how to prioritize the acquisition of the important data. S. mitis in the blood cultures on two consecutive days in a patient with a prosthetic valve or MSSA in a septic prosthetic joint really takes very little time to get to the heart of the case

A lot of what we do in addition is more to be complete or to communicate with the patient and family.

And most cases need at most need a page to a page and a half of dictation. Not that you would know it by many notes, where I suspect the physician thinks they get paid by the word, not the deed. You do not need to use a lot of words to convey complex ideas. Again less is better. It's Hemmingway, not another damned thick chart note! Always scribble, scribble, scribble! Eh, Dr. Gibbon?

Simplicity rules. Until it doesn't.

The young patient has trauma to the knee that needs fixing. Two months later, his knee becomes increasingly painful, and a tap shows 22,000 WBC, so it is washed out, and they call me.

No PMH or unusual, or usual, risks, and the joint fluid has pyrophosphate crystals. Gram stain negative, as are the cultures at three days. Pseudo-gout. Simple. It makes more sense for the timing. Infections in a normal joint would be odd two months out. I explained all the above to the patient and was on my way. Understanding the case took at most 30 seconds; the rest was dotting t's and crossing i's.

Cue the music to jaws.

On day 5, all the joint cultures grew Finegoldia magna.

That is an odd bug. There are only a dozen cases reported, mostly in prosthetic joints. I think I have seen a case or two in the past, but if I so I have not blogged about it.

Which came first? I would bet the pseudo-gout, but I don't know. One study suggests infection leads to pyrophosphate release. The nice thing about Finegoldia is it is killed easily enough.

Still, the positive cultures added about 15 seconds to the case, 2/3's of which were spent swearing.

Rationalization

Chicken and Egg.

http://boards.medscape.com/forums/?128@@.2a7b8557!comment=1

Eur J Clin Microbiol Infect Dis. 2014 Aug;33(8):1355-64. doi: 10.1007/s10096-014-2073-3. Epub 2014 Feb 28.v Bone and joint infections due to anaerobic bacteria: an analysis of 61 cases and review of the literature.

https://www.ncbi.nlm.nih.gov/pubmed/24577953

POLL RESULTS

I am ruled by

  • simplicity 23%
  • complexity 6%
  • medicare 0%
  • my spouse 14%
  • the voices in my head 46%
  • Other Answers 11%
  • Mathematics
  • Friggen EPIC
  • my cats
  • My cat.

Another SAB

Nov 20, 2017

If I were a superhero, S. aureus would be my arch enemy. It is the Lex Luthor to my Superman. I was a DC person. And like comic book villains, every time you think it is dead, it comes back, even harder to kill.

The first consult of the week was a S. aureus bacteremia (SAB). In my institution, SAB is an automatic ID consult, which is as it should be.

There are now multiple studies that show ID consultations benefit patients with SAB:

Evidence-based clinical management enforced by IDC may improve outcome of patients with SAB.

As well it should. Who would have thunk it? Having the involvement of HCW's who know what they are doing is good for patients. In this case, no one had considered the pacemaker as an issue in the patient unexplained SAB.

ID is probably the one part of medicine where people who really have no clue what they are doing are allowed free reign in patient care. Don't believe me? How about Association of Evidence-Based Care Processes With Mortality in Staphylococcus aureus Bacteremia at Veterans Health Administration Hospitals, 2003-2014

Risk-adjusted mortality decreased from 23.5% (95% CI, 23.3%-23.8%) in 2003 to 18.2% (95% CI, 17.9%-18.5%) in 2014. Rates of appropriate antibiotic prescribing increased from 2467 (66.4%) to 1991 (78.9%), echocardiography from 1256 (33.8%) to 1837 (72.8%), and ID consultation from 1390 (37.4%) to 1717 (68.0%).

Appropriate care improved outcomes. Wow. But they are still letting 32% of SAB cases be treated by people who likely have no real idea what they are doing.

You would never see a paper like this in the oncology literature. People are not allowed to treat cancer unless they know what they are doing.

Imagine if people treated cancer like infectious diseases. No oncologists taking care of the patient. Chemotherapy would be given based on FUD. It might be lymphoma, so let's give CHOP. You don't want to miss lung cancer, so add cis-platinum. And it could be pancreatic, so radiate. And you gotta double cover leukemia. And when the biopsy came back, the results would be ignored, and the chemo continued as the patient was getting better.

Yes, I know. Not every patient needs an ID consult. And given the acuity of infections, empiricism is reasonable at times.

But still.

Rationalization

J Infect. 2016 Jan;72(1):19-28. doi: 10.1016/j.jinf.2015.09.037. Epub 2015 Oct 9. Infectious disease consultation for Staphylococcus aureus bacteremia - A systematic review and meta-analysis.

https://pubmed.ncbi.nlm.nih.gov/26453841/

JAMA Intern Med. 2017 Oct 1;177(10):1489-1497. doi: 10.1001/jamainternmed.2017.3958. Association of Evidence-Based Care Processes With Mortality in Staphylococcus aureus Bacteremia at Veterans Health Administration Hospitals, 2003-2014.

https://www.ncbi.nlm.nih.gov/pubmed/28873140

POLL RESULTS

I am/was

  • DC 23%
  • Marvel 32%
  • Archie 18%
  • Dark Horse 5%
  • Indie 5%
  • Other Answers 18%
  • Dostoyevskian.
  • Calvin and Hobbes

A Rapidly Growing Mass

Nov 22, 2017

The patient presents with shortness of breath. She is a kidney transplant patient on the usual medication, has no unusual exposures, and the only other symptoms are a nonproductive cough and malaise.

CT shows a pleural effusion and a mass in the right lung that was not present on CT three months ago. Before any diagnostic testing can be done, she leaves AMA but returns 3 days later with more dyspnea, and now the hemithorax is filled with fluid.

I see the patient, and she doesn’t look all that ill, and the labs are mostly unimpressive. So if I figure that if this mass is infectious, it is a fungus and if so, I bet on Cryptococcus as that is the only opportunistic infection of note in the great Pacific NW. Of the fungi, it is the one I have most often seen as a (relatively) asymptomatic mass. So I ramble on at length to the residents as to my reasoning.

I add a cryptococcal antigen to the blood work and await the results of blood, sputum, and pleural fluid. I don’t think he needs antibiotics.

The next day the blood grows a gram-negative rod. Wrong on two accounts.

The day after that, it is identified: a pan-susceptible Haemophilus influnzae; not B.

It has been a while since I have seen an H. influenzae, and it is nice that all my pontificating to the house staff as to the likely etiology was totally and completely wrong. Sigh. When I am wrong, I am spectacularly wrong. But that is why we do testing.

Two take-homes, one I knew, one I didn’t.

The first, the one I knew, is that being a transplant patient modifies the presentation and outcomes of sepsis.

These findings suggest that the immunosuppression associated with transplantation may provide a survival advantage to transplant recipients with sepsis through modulation of the inflammatory response.

And the other?

We report the radiographic and computed tomographic findings in an asymptomatic patient who had a Haemophilus influenzae lung abscess. The diagnosis was made by Gram's staining and culture of abscess material obtained by transthoracic needle aspiration. The radiographic abnormalities cleared after specific antimicrobial therapy.

As did this patient.

Rationalization

Is bacteremic sepsis associated with higher mortality in transplant recipients than in nontransplant patients? a matched case-control propensity-adjusted study. Clinical Infectious Diseases, 60(2), 216-222. DOI: 10.1093/cid/ciu789

https://pubmed.ncbi.nlm.nih.gov/25301215/

Can Assoc Radiol J. 1990 Apr;41(2):98-9. Haemophilus influenzae abscess presenting as an asymptomatic lung mass.

https://pubmed.ncbi.nlm.nih.gov/2328430/

POLL RESULTS

As to me

  • when I am wrong, I am spectacularly wrong 41%
  • I am always right, the patient has the wrong disease 16%
  • I never commit to a diagnosis so I never look bad 5%
  • I defer to my consultants and avoid responsibility 5%
  • I use qualifiers: perhaps, maybe, likely to I have an out 27%
  • Other Answers 5%
  • I no longer opine on anything. Observable facts rock.
  • I always commit clinically and take a call. Medicine is about decision making and not playing safe for yourself. The decision must favor the patient.

It Happens

Nov 27, 2017

The patient is an elderly male, admitted with fevers, chills, and abdominal pain. Nothing of note in the history, he goes nowhere, does little. I understand the feeling.

CT shows inflammation in the abdomen around the iliopsoas. They suppose diverticulitis, although no -osis was seen, and give Augmentin. Of course. He does not get much better, and a repeat CT shows the inflammatory mass has doubled. They tap it. And it grows...

Most of the iliopsoas infections, pyomyositis, I see are in young men who have a groin pull and a few days later have a fever and a S. aureus abscess in the muscle. That is not this patient.

The classic pyomyositis, so-called tropical pyomyositis, is most often due to a S. aureus. Oregon is many things, but tropical is not on the list. But there are a hodgepodge of organisms that have been isolated in pyomyositis:

Staphylococcus aureus accounted for 83.3% (25 cases), and Escherichia coli, Nocardia spp., Streptococcus agalactiae, nontuberculous mycobacteria, Pseudomonas aeruginosa were isolated in one case each.

That's not this patient.

Streptococcus pyogenes is a vicious pathogen, causing pharyngitis, cellulitis, necrotizing fasciitis, toxic shock syndrome, and pyomyositis.

Other Streptococci look up to Group A and try and emulate it, often with success. Especially Group G Streptococcus, which so wants to be a Group A. It can cause all of the above, albeit rarely.

This patient grew group G Streptococcus, Streptococcus dysagalactiae subsp. equisimilis, trying to be one of the big boys.

It happens. It always happens.

There are a smattering of cases of Group G Streptococcus pyomyositis, and evidently, the bug an issue in Japan, where I love the detail in the case report:

On admission, right shoulder pain became worse, and he could not eat breakfast with the right hand. After he ate it with the left hand, he had vomiting and lost the consciousness.

It is why I never eat left-handed.

Some iv beta-lactam my pateint slowly improved.

Rationalization

J S C Med Assoc. 2015 Dec-2016 Jan;111(4):137-8. Tropical Pyomyositis in a Temperate Climate.

https://www.ncbi.nlm.nih.gov/pubmed/27141707

Pediatrics. 1979 Feb;63(2):298-300. Tropical pyomyositis: a case report and review.

https://www.ncbi.nlm.nih.gov/pubmed/440823

J Clin Microbiol. 2003 Feb;41(2):613-8. Analysis of a viridans group strain reveals a case of bacteremia due to lancefield group G alpha-hemolytic Streptococcus dysgalactiae subsp equisimilis in a patient with pyomyositis and reactive arthritis.

https://www.ncbi.nlm.nih.gov/pubmed/12574255

Streptococcus dysagalactiae pyomyositis

http://onlinelibrary.wiley.com/doi/10.1002/jgf2.86/full

POLL RESULTS

I eat my breakfast

  • right handed 53%
  • left-handed 10%
  • through a straw 0%
  • with my face in the plate 23%
  • with my feet. I like sole food. 3%
  • Other Answers 10%
  • with a silicone rubber spoonula, so as to avoid clicking and clanking noises that would irritate she who must be obeyed.
  • Intravenously
  • At dinner time

Harm or Benefit

Nov 29, 2017

93-year-old male, with no past medical history of note except an AICD a year ago, is admitted with a gi bleed.

As an outpatient, he had been having crops of marble-sized red bumps on first his arms, then legs that started two months ago. They had been biopsied and cultured elsewhere, and the day after admission, the cultures turned positive for M. chelonae.

So they called me.

He has zero risks for M. chelonae. No immunocompromise, no exposure to water or organic material that would serve as a focus for infection.

He has had a failure to thrive and some weight loss, but no fevers.

A pan scan shows pleural effusions and perhaps some small nodules in the lung. I'm not impressed. TTE with no vegetations.

So he likely has disseminated M. chelonae. But why only on the extremities, they ask. That one is easy. M. chelonae prefers a temperature lower than 37 C for optimal growth. The arms and legs are cooler.

But why this disease, an odd manifestation of an odd organism. I suspect from the distribution that he may have an AICD infection. It happens. Or happened. Once.

Pacemaker infection with Mycobacterium chelonae has not been reported previously. We report the first case of pacemaker lead endocarditis due to M. chelonae, which was successfully treated with multidrug regimen.

Now what to do? This led to a long discussion with the patient and his family about the options and the side effects.

TEE? AICD removal? Multidrug therapy?

One thing you learn in practice is the older the patient, the easier it is to do harm.

When I was a medical student, I told my general surgery attending that my patient was a healthy 85 for surgery. He replied with a touch of scorn that it didn't matter; the patient had 85-year-old meat. He was a well-known ass, but the idea was valid: the older the patient, the more fragile they are.

Many of the medications used for M chelonae are toxic to either the wallet (linezolid) or the body (aminoglycosides).

They are interested in doing less rather than more and in quality of life over quantity.

So we will temporize for now. Sensitivities are pending, as are blood cultures. He is tolerating the clarithromycin and feeling better.

Time will tell.

Rationalization

BMJ Case Rep. 2014; 2014: bcr2014206042. Published online 2014 Dec 22. PMCID: PMC4275738 Case Report Disseminated Mycobacterium chelonae infection causing pacemaker lead endocarditis in an immunocompetent host

https://www.ncbi.nlm.nih.gov/pmc/articles/PMC4275738/

Heading for Death

Dec 4, 2017

After 30 years of taking care of infections in IVDA, I remain impressed at the siren call of heroin.

The patient has been in and out of the hospital 4 times in the last three weeks for right-sided MRSA endocarditis. He gets a few days of vancomycin and then leaves to get his heroin.

Each time he is admitted, the lung abscesses are a little bigger, the vegetation a little larger, and the MIC?

That is what concerns me. Initially, the MIC to vancomycin was 0.5, but now it is 3.0. Damn.

I assume this is a heteroresistant MRSA:

Heteroresistance refers to the presence, within a larger population of fully antimicrobial-susceptible microorganisms, of subpopulations with lesser susceptibility... hVISA have minimum inhibitory concentrations (MICs) in the intermediately susceptible range and likely represent a step on the path to the development of a fully VISA population

I can't kill this bug with vancomycin, so perhaps ceftaroline, where the MIC is still 0.5.

But how much? Every 8-hour dosing may be better for severe infections:

This regimen is often used because pharmacokinetic studies have suggested optimal time-dependent killing with more frequent dosing, but superiority has yet to be proven clinically.

And maybe add some aminoglycoside. I know that is passé for S. aureus endocarditis but

the combination of ceftaroline plus tobramycin at 0.5 x MIC was synergistic against the two MRSA strains and the hVISA.

So maybe if I pound on the organism for a short period of time, keep him in for two weeks, maybe I can cure the endocarditis with a shorter than usual course. It is not going to get better on its own. Usually.

Bacterial endocarditis was generally considered a fatal disorder until around 1900 when a few scattered reports described rare "spontaneous" cures particularly in patients with disease caused by the gonococcus. During the 1930's there appeared descriptions of successful use of fever therapy for gonococcal endocarditis.

This time he was in for less than 12 hours; before I could make the change from vancomycin, he left for heroin and certain death.

Rationalization

Vancomycin Heteroresistance and Methicillin-Resistant Staphylococcus aureus Stan Deresinski The Journal of Infectious Diseases, Volume 199, Issue 5, 1 March 2009, Pages 605–609,

https://doi.org/10.1086/596630

Open Forum Infect Dis. 2017 Spring; 4(2): ofx084. Published online 2017 May 2. doi: 10.1093/ofid/ofx084 PMCID: PMC5499876 Ceftaroline for Severe Methicillin-Resistant Staphylococcus aureus Infections: A Systematic Review

https://pubmed.ncbi.nlm.nih.gov/28702467/

Role of aminoglycosides in the treatment of bacterial endocarditis J. C. Graham F. K. Gould Journal of Antimicrobial Chemotherapy, Volume 49, Issue 3, 1 March 2002, Pages 437–444,

https://doi.org/10.1093/jac/49.3.437

Int J Antimicrob Agents. 2010 Jun;35(6):527-30. doi: 10.1016/j.ijantimicag.2010.02.006. Epub 2010 Mar 26. In vitro evaluation of ceftaroline alone and in combination with tobramycin against hospital-acquired meticillin-resistant Staphylococcus aureus (HA-MRSA) isolates.

https://www.ncbi.nlm.nih.gov/pubmed/20346632

PROGRESS IN MANAGEMENT OF PATIENTS WITH INFECTIVE ENDOCARDITIS

https://www.ncbi.nlm.nih.gov/pmc/articles/PMC2279546/pdf/tacca00096-0076.pdf

Don't believe everything you think.

Dec 6, 2017

I will go out on a limb and suggest that injection drug use is a bad idea for patients but great for clinicians who take care of its complications. I have always felt a little guilty that all the great/interesting cases are the result of the pain and suffering of my fellow man.

My service has been booming the last several years thanks to the opioid crisis, with a remarkable uptick in endocarditis cases. Rarely seen manifestations of endocarditis are popping up because of the large number of cases.

As a change, the current patient is an iv meth user, starting in his late 50's. He comes in with MSSA endocarditis and emboli everywhere: eyes, hands, feet, kidney, and brain. His MRI shows one of the emboli bled, a hemorrhagic conversion.

He is put on cefazolin, his fever resolves, his BC became negative, and he does well. Except. He never improves neurologically, remaining delirious.

A repeat MRI shows that the previous hemorrhagic conversion is now a lemon-sized brain abscess.

Hmm. Been a long time since I have seen that

Bacterial brain abscess is a rare complication of endocarditis, affecting between 1% and 7% of patients with IE. They are most commonly seen in the setting of methicillin-resistant S aureus (MRSA) IE

I have been using a lot more cefazolin the last year. It is less expensive, fewer side effects, of equal efficacy, and may even have improved mortality:

We found a large reduction in 90-day mortality in those receiving cefazolin compared to nafcillin for MSSA BSI, but this finding was not statistically significant.

and

In this large, multicenter study, patients who received cefazolin had a lower risk of mortality and similar odds of recurrent infections compared with nafcillin or oxacillin for MSSA infections complicated by bacteremia.

I do fret about cefazolin susceptibility to some Staphylococcal beta-lactamases, which I blogged about a couple of years back.

I have always carried it in my head from fellowship days that cefazolin is not a good choice for CNS infections as it crosses the BBB poorly. Is that why he went on to get a brain abscess? The PDR says

Cefazolin does not reach therapeutic levels within the CSF.

But without a reference. Perhaps it goes to a study from 1976 that says

In four neurosurgical patients with an intraventricular catheter, neither bolus injection nor continuous infusion resulted in a demonstrable concentration of cefazolin in the cerebrospinal fluid.

Is that true? Maybe not. It is always good to go back and review those 'facts' you learned from your betters as a youth. They do not always hold up. Cefazolin may have less penetration than a third-generation cephalosporin, but it isn't always that bad.

Eleven of 13 brain specimens contained detectable nafcillin concentrations between 0.36 and 11 micrograms/g of tissue (mean, 2.7 micrograms/g for all 13 specimens). Fourteen of 18 brain tissue specimens contained detectable methicillin concentrations between 0.56 and 5.0 micrograms/g of tissue (mean, 2.0 micrograms/g for all 18 specimens). Ten of 11 brain tissue specimens contained detectable cefazolin concentrations between 2.0 and 40 micrograms/g of tissue (mean, 10.6 micrograms/g for all 11 specimens). Each antibiotic penetrated "abnormal" brain tissue better than "relatively normal" brain tissue. Because nafcillin is more active against Staphylococcus aureus, we conclude that nafcillin is preferable to methicillin for the therapy of central nervous system staphylococcal infections. Cefazolin achieves higher brain tissue concentrations than the penicillins, but has not been clinically evaluated for the therapy of central nervous system infections.

and

Five of the six patients given multiple doses of cefazolin sodium had notable CSF accumulation of the drug (11.3 +/- 2.7% of the serum concentration).

And there are at least 3 cases of cefazolin seizures associated with high CSF concentrations, more than enough to kill an S. aureus with an MIC of 1:

Serum and CSF concentrations of cefazolin one day later were 470 and 64 micrograms/ml, respectively...Serum and CSF concentrations eight hours after the last dose of cefazolin were 360 and 34 micrograms/ml, respectively. ...Serum and CSF concentrations of cefazolin measured 28 hours later were 1000 and 106 micrograms/ml, respectively.

That's all I could find. One study that said no CSF cefazolin, 3 suggest otherwise with about 10% of serum reaching the CSF.

If there are more definitive studies, I can't find them, as it would be nice to know what the best therapy is for a S. aureus brain abscess.

But he is now on nafcillin.

Rationalization

Antimicrob Agents Chemother. 2016 Jul 22;60(8):4684-9. doi: 10.1128/AAC.00243-16. Print 2016 Aug. Cefazolin versus Nafcillin for Methicillin-Sensitive Staphylococcus aureus Bloodstream Infection in a California Tertiary Medical Center.

https://www.ncbi.nlm.nih.gov/pubmed/27216053

Clin Infect Dis. 2017 Jul 1;65(1):100-106. doi: 10.1093/cid/cix287. Comparative Effectiveness of Cefazolin Versus Nafcillin or Oxacillin for Treatment of Methicillin-Susceptible Staphylococcus aureus Infections Complicated by Bacteremia: A Nationwide Cohort Study.

https://pubmed.ncbi.nlm.nih.gov/28379314/

Little Things Matter

http://boards.medscape.com/forums/?128@@.2a7ac7e4!comment=1

Neurosurgery. 1983 Feb;12(2):142-7. Penetration of nafcillin, methicillin, and cefazolin into human brain tissue

https://www.ncbi.nlm.nih.gov/pubmed/6835496

Am J Hosp Pharm. 1981 Oct;38(10):1496-9. Effect of multidose therapy on cerebrospinal fluid penetration of cefazolin.

https://www.ncbi.nlm.nih.gov/pubmed/7294044

Am J Hosp Pharm. 1980 Feb;37(2):271-3. Seizures associated with high cerebrospinal fluid concentrations of cefazolin.

https://www.ncbi.nlm.nih.gov/pubmed/7361804

Antimicrob Agents Chemother. 1976 Sep;10(3):395-8. Pharmacological study of cefazolin during intermittent and continuous infusion: a crossover investigation in humans.

https://www.ncbi.nlm.nih.gov/pubmed/984782

Neurohospitalist. 2014 Oct; 4(4): 213–222. doi: 10.1177/1941874414537077 PMCID: PMC4212418 Neurologic Complications in Infective Endocarditis

https://www.ncbi.nlm.nih.gov/pmc/articles/PMC4212418/

Complications

Dec 11, 2017

An 80-year-old female, at baseline until the day before admission when she became febrile and delirious.

She was admitted, had the usual fever workup, and the only positive result was blood cultures, all of which were grew Enterococcus in less than 24 hours.

So they called me.

ID is all about the source for infections, but there isn't one. A source, not an infection.

So that alone makes it likely endocarditis. Then we add to the mix that the patient had a transaortic valve replacement (TAVR) 5 months ago.

There is nothing like endovascular hardware to raise the stakes, and TAVR's are not good. It is like the patient read the abstract and applied it to herself:

A total of 250 cases of infective endocarditis occurred in 20 006 patients after TAVR (incidence, 1.1% per person-year; 95% CI, 1.1%-1.4%; median age, 80 years; 64% men). Median time from TAVR to infective endocarditis was 5.3 months (interquartile range [IQR], 1.5-13.4 months). The characteristics associated with higher risk of progressing to infective endocarditis after TAVR was younger age (78.9 years vs 81.8 years; hazard ratio [HR], 0.97 per year; 95% CI, 0.94-0.99), male sex (62.0% vs 49.7%; HR, 1.69; 95% CI, 1.13-2.52), diabetes mellitus (41.7% vs 30.0%; HR, 1.52; 95% CI, 1.02-2.29), and moderate to severe aortic regurgitation (22.4% vs 14.7%; HR, 2.05; 95% CI, 1.28-3.28). Health care–associated infective endocarditis was present in 52.8% (95% CI, 46.6%-59.0%) of patients. Enterococci species and Staphylococcus aureus were the most frequently isolated microorganisms...

I was surprised to see the Enterococcus at the top of the list.

The ECHO did not show a vegetation. Yet. But it is probably too early. I was taught, but have never been able to confirm, that it takes at least a week to get a vegetation that can be seen on a TTE. This fact was given to me from a time before TEEs. Is that true? Got me. But I have been saying it for 30 years.

Patients get a TAVR because they are not good surgical candidates to begin with. So what to do after she finishes a course of antibiotics. I have never used suppressive antibiotics for bacterial endocarditis, prosthetic or otherwise, and a search of Pubmed using "prosthetic valve endocarditis suppressive" only gets hits for Candida. So if anyone has tried chronic antibiotics for PVIE, they are not bragging about it. The closest is pacer infections, where it kind of sort of works.

I'll have to cross that bridge later.

Postscript

After discussing the pros and cons with the patient, we went chronic amoxicillin.

Rationalization

Association Between Transcatheter Aortic Valve Replacement and Subsequent Infective Endocarditis and In-Hospital Death.

https://jamanetwork.com/journals/jama/fullarticle/2552209

Long-Term Suppressive Antimicrobial Therapy for Intravascular Device-Related Infections

http://www.sciencedirect.com/science/article/pii/S0002962915345808

No such thing as contaminant

Dec 14, 2017

12 months ago, the patient had a prosthetic aortic valve placed. You already know where this is going, don't you? ID doc, writing an ID blog, starting out with a prosthetic valve. It isn't going to be a case of sprue, that's for sure.

He initially has does well, then slides into failure to thrive. He has chills, a cold feeling, and malaise, and blood cultures are drawn.

5 days later, one set grows a P. acnes. He has no fevers, and his inflammatory parameters (WBC, ESR, CRAP) are normal. It is considered a contaminant.

Nope.

I would not consider P. acnes a contaminant. P. acnes mostly grows in hair follicles, and the antecubital fossa is mostly hairless.

A lot of the what are called contaminants on blood cultures are not:

...certain organisms have been found to represent contamination in a significant proportion of cases. These organisms include coagulase-negative staphylococci, Corynebacterium species, Bacillus species other than Bacillus anthracis, Propionibacterium acnes, Micrococcus species, viridans group streptococci, enterococci, and Clostridium perfringens.

My take, and you can take it or leave it, is except for coagulase-negative Staphylococci, Corynebacterium species, and Bacillus species; the rest are real but clinically unimportant, bacteremias.

Of course, anything in the blood of a patient with endovascular hardware has to be considered the real deal.

So repeat blood cultures all grew P. acnes. Unfortunately, the TEE had the predicted ring abscess, and he went on to have his valve replaced.

I used to blame acquisition of P. acnes on the trauma surgery, but I am not so sure it is not more akin to oral streptococci:

Anaerobic bacteria were isolated from all root canals. In the 2 patients where over instrumentation had occurred, Propionibacterium acnes was recovered both from the root canals and from the blood samples taken during and after the treatment had been completed. Biochemical profiles, antibiotic susceptibility tests and electrophoresis of soluble proteins revealed that Propionibacterium acnes isolated from the root canal and blood samples were identical within patients,

It is why I don't ever brush or floss.

This is the third PVIE from P. acnes I have seen, a bug that causes more than its share of difficult infections.

Rationalization

Clin Microbiol Rev. 2006 Oct; 19(4): 788–802. doi: 10.1128/CMR.00062-05 PMCID: PMC1592696 Updated Review of Blood Culture Contamination.

https://www.ncbi.nlm.nih.gov/pmc/articles/PMC1592696/

Endod Dent Traumatol. 1992 Dec;8(6):248-54. Profiling of Propionibacterium acnes recovered from root canal and blood during and after endodontic treatment.

https://www.ncbi.nlm.nih.gov/pubmed/1302689

Prosthetic Valve Zit

http://boards.medscape.com/forums/?128@@.2a81808c!comment=1

A Connecticut Yankee

http://boards.medscape.com/forums/?128@@.2a5ad057!comment=1

Bog Body

Dec 18, 2017

Dead Bugs Dead Bugs

What cha gonna do

What cha gonna do

When then come for you

Zorak. Kind of.

The patient had a prosthetic knee infection, MSSA, many months ago. The knee was removed after a salvage attempt, a spacer placed, and he finished a course of IV beta-lactam. After a couple of months, a tap of the residual space was negative of cells, and he went to the OR for a new knee.

In the OR, the knee looked fine.

But the surgeon sent off some gram stains, and one came back with 3 plus gram-positive cocci, and the case was aborted. They called me.

He had no symptoms of infection pre-op, negative ESR, and CRAP (C ReActive Protein). I will admit I did not trust the gram stain. It was done by the afternoon crew, not the microbiologists. We do have the occasional false positive gram stain after hours. Gram stains are hard to read and are as much an art as a science, but the A team took a look at the strain. Yep. Gram-positive cocci, but they would call it two plus. And no WBC cells.

As best I can remember, I have never had this problem before. All the reimplants I have taken care of have had negative cultures and gram stains at the time of surgery. Evidently, I have better than average outcomes:

In 27 patients (16.6%), ≥ 1 positive culture was identified at re-implantation and eight (29.6%) of these subsequently failed compared with 20 (14.7%) patients who were culture-negative.

But that's cultures, as my patients cultures were never positive. What about gram stains? Not too good for the diagnosis of an infection with the joint in place

a poor clinically acceptable diagnostic value for detecting PJI

and

Thirty-two were found to be infected (11 hips and 21 knees) and 137 had no evidence of infection. Intraoperative Gram staining was negative in all 169 cases. The method therefore had a sensitivity of 0% for detecting infection.

So the gram stain is not the most helpful of tests if negative for PJI infections. I suspect with negative cultures, CRAP, and no WBC's these were dead bugs. The only place I know where dead bugs can persist is endocarditis vegetations, where the corpses of bacteria can persist for months after stopping antibiotics. The space around the antibiotic spacer would seem to me to be another place where bacterial cadavers could persist, a microbial bog body.

I still gave him another course of antibiotics. Safe >> Sorry.

Rationalization

Bone Joint J. 2017 Nov;99-B(11):1490-1495. doi: 10.1302/0301-620X.99B11.BJJ-2017-0243-R1. A positive bacterial culture during re-implantation is associated with a poor outcome in two-stage exchange arthroplasty for deep infection.

https://pubmed.ncbi.nlm.nih.gov/29092988/

Exp Ther Med. 2015 May; 9(5): 1857–1864. Published online 2015 Feb 25. doi: 10.3892/etm.2015.2315 PMCID: PMC4471807 Limitations of Gram staining for the diagnosis of infections following total hip or knee arthroplasty.

https://www.ncbi.nlm.nih.gov/pmc/articles/PMC4471807/

J Bone Joint Surg Br. 1996 Sep;78(5):838-9. Gram stain detection of infection during revision arthroplasty.

https://www.ncbi.nlm.nih.gov/pubmed/8836084

Bacterial Mummies.

http://boards.medscape.com/forums/?128@@.29f7d89b!comment=1

Cartoon Planet - Bad Bug

https://youtu.be/1AUuZ021ppQ

Strong Bad

Dec 20, 2017

The patient has end-stage HIV. He is from SE Asia and, for reasons that are never clearly articulated (and we have asked, believe me), has long refused to take HAART.

The results have not been good: fevers, weight loss, malaise, then Cryptococcal meningitis with cryptococcomas, CD4 of 50. He finally agreed to take medications and was slowly getting better when he began to have nausea and vomiting but no diarrhea. HAART? IRIS? Other meds? New OI? New diagnosis?

We started working through the differential diagnosis. Holding medications did little; there was no oral pathology. No Addisons. So after a week, he was sent for EGD.

No thrush was seen, just diffuse esophagitis and gastritis, which was biopsied and showed?

Worms?!? Worms. Strongyloides.

Huh. Never saw that coming. I have often fretted about Strongyloides hyperinfection in patients over the years, but not in HIV patients. Perhaps I should, especially in immigrants as there is a lot of Strongyloides in the world and

The meta-analysis showed an association between HIV-infection/alcoholism and S. stercoralis infection.

although it depends on the country of origin:

...prevalence rate of 11.8% in Vietnamese refugees versus 76.6% in Cambodian refugees...A study in Saudi Arabia by al-Madani and colleagues analyzed 5,518 female housekeepers originating from different Asian countries. The overall prevalence reported was 0.6%; 0.4% in Filipinos, 0.5% in Indonesians, 1.5% in Sri Lankans, 2.6% in Indians and 3.4% in Thais...

Brazil also has a problem with Strongyloides.

HIV is likely a risk:

We identified 16 case-control studies comparing HIV-positive individuals with sero-negative controls. Four reported a lower or similar prevalence in the two groups. All other studies showed an increased S. stercoralis infection risk for HIV-positive individuals; three showed a statistically significant risk. Our meta-analysis resulted in a pooled OR of 2.17 (95% BCI: 1.18–4.01) for HIV-positive individuals compared to the HIV-negative controls.

He did not have bacteremia, eosinophilia, or other signs of the hyperinfection syndrome, and perhaps the increased gi symptoms were a sign of IRIS.

Rather, immune reconstitution appears to provoke a phenotypic switch in the parasite population, leading to higher proportions of invasive filariform larvae.

His gi symptoms improved after a course of ivermectin.

My take-home? Think Strongyloides in immigrants.

Rationalization

PLoS Negl Trop Dis. 2013 Jul 11;7(7):e2288. doi: 10.1371/journal.pntd.0002288. Print 2013. Strongyloides stercoralis: Global Distribution and Risk Factors.

https://www.ncbi.nlm.nih.gov/pubmed/23875033

Strongyloides stercoralis Infection as a Manifestation of Immune Restoration Syndrome

https://academic.oup.com/cid/article/39/3/439/354159

Novel Findings in HIV, Immune Reconstitution Disease and Strongyloides stercoralis Infection.

http://www.sciencedirect.com/science/article/pii/S0002962916302889

Strong Bad.

https://en.wikipedia.org/wiki/Strong\_Bad

I have a tee-shirt.

False Memory

Dec 27, 2017

It has become a not infrequent story. The patient has some pulmonary symptoms: a chronic, minimally productive cough for the last three years. No other symptoms, but she is an ex-smoker. A CXR shows a maybe something in the left lung which showed a mass on a CT that glowed on PET that lead to a partial lung resection that led to an ID consultation. The ID equivalent of Because a Little Bug Went Ka-Choo. BTW, it was not amenable to percutaneous biopsy.

The good news it was not cancer. The other good news is it was Coccidioides immitis, not my first to present after a resection. Atypical mycobacteria and endemic fungi can radiographically look for all the world like a cancer and on occasion they get diagnosed and treated with lung resection.

She has no symptoms or risks for disseminated Coccidioides and the complement fixation test is <1:2. data-preserve-html-node="true" But when did she get it?

The last time she had been in Coccidioides country, Arizona, was 15 years ago. She had no illness at the time, and she has had no other travel exposure.

Coccidioides, unlike love, can last a life time, so manifesting 15 years after leaving an endemic area doesn't bother me. I went back and reviewed the CXR's going back 20 years I did not see much until the films this year, and with the CT I knew just where to look. And I am a pro at over-reading a CXR if I know where the pathology is ahead of time. I saw nothing, nor did the radiologist, until the last 3-4 years.

I have all these facts and stories in my head. I did my fellowship in SoCal so I saw a lot of Coccidioides and heard a lot of Coccidioides stories. Coccidioides is one of those 'fun' infections with a lot of curious stories for exposures and presentations. People can have fairly minimal exposures to dust and come down with Coccidioides.

I swear there is a case of Coccidioides in a patient in the East coast whose exposure was a gift box of oranges from SoCal that had Coccidioides dust on it. I have been telling housestaff that story for years. Today I went looking for the case, because I think every ID statement I make should have a reference. I can't find it. No combination of Coccidioides and fruit I can think of yields the reference. Is that a false memory? One of the many true things I have learned from others as part of training? A wee bit of Apocrypha? I don't know. Medicine is filled with unsubstantiated lore.

I mention it as I asked and for the last 5 years the patient receives oranges from SoCal as a Christmas gift every year, roughly coinciding with the onset of the cough.

But. The cough is unchanged with the resection. So I bet the Coccidioides had nothing to do with her cough.

From the ID perspective there was nothing further to do, I bid her a happy new year and told her to see her PMD about the cough.

And I am forced again to ponder the saying, "Don't believe everything you think."

See you in 2018.

Rationalization

Lung. 2014 Aug;192(4):589-93. doi: 10.1007/s00408-014-9589-2. Epub 2014 May 7. Positron emission tomography in the evaluation of pulmonary nodules among patients living in a coccidioidal endemic region.

https://www.ncbi.nlm.nih.gov/pubmed/24801058

Lung cancer or valley fever? CT can spot the difference.

http://www.auntminnie.com/index.aspx?sec=ser&sub=def&pag=dis&ItemID=112598

POLL RESULTS

I

  • believe everything I think 7%
  • alter what I think to manufacture a comforting reality 48%
  • have a peer reviewed reference for every fact 10%
  • believe whatever I am told at the moment 24%
  • think all viewpoints and facts are equal 0%
  • Other Answers 10%
  • believe everyone is just as in the dark as I am.
  • views/facts varying value according to source!
  • think, therefore I am wrong a significant part of the time.

String Theory

Jan 3, 2018

The patient is a young male with no medical problems who is admitted with fevers and a transaminitis. One thing led to another, and three liver abscesses were found.

Two of the three were drainable, and the initial gram stain was read as WBC and but no organisms.

They called me, and I got all excited. My first amoebic liver abscess of the century? Well, maybe not.

I looked at the time of the gram stain—7 pm. Never trust a gram stain read after hours. Interpreting gram stains is an art that the night shift, with all due respect, may not have mastered. Gram-negatives can be especially hard to see against the background of pus.

I called the A team and asked them to review the stain. GNB was indeed there, and the culture grew the K. pneumoniae. And it had a positive string test,

A positive string test is defined as the formation of viscous strings of >5 mm in length when a loop is used to stretch the colony on an agar plate

suggesting a hypermucoviscosity-phenotype Klebsiella pneumoniae

Zero reasons for this organism I can find. No travel (it is a big problem in Asia), no biliary problems, no nothing.

It was pansensitive (and can't we all wait for the combination of hypermucoviscosity and CRE?), and he was started on ceftriaxone and remained febrile. In the middle of the night, concerned that fevers persisted, the hospitalist broadened coverage. The reason? Pure FUD and not the Elmer kind.

Sigh.

I realized today that being a consultant is a lot like being a husband (or, I suppose, a wife). You watch people you like and respect, do things that you wouldn't do, and you kind of sort of understand their motivation, but not really. But you smile and nicely nudge events the way you think they should go as you in it with them for the long haul.

This is not an issue of antibiotics, I said, it is undrained pus. And these hypermucoviscosity strains are a shortness of breath to kill. A repeat CT showed the prior small undrainable abscess was now a large, drainable abscess, and it was drained, and he got better.

It is aways about source control. No drainage, no cure. Know drainage, know cure.

Drain the pus.

Words to live by. I want them on my tombstone.

Mark Crislip

1957-2166

He wanted the pus drained

RIP

Rationalization

Clinical utility of string test as a screening method for hypermucoviscosity-phenotype Klebsiella pneumoniae.

https://www.ncbi.nlm.nih.gov/pmc/articles/PMC5997228/

Microbiological and Clinical Characteristics of Hypermucoviscous Klebsiella pneumoniae Isolates Associated with Invasive Infections in China.

https://www.frontiersin.org/articles/10.3389/fcimb.2017.00024/full

But Wait. Theres More.

Jan 8, 2018

Getting older is all about less. Less energy, less concentration, less stamina, less time. Last night we went to see one of our favorite musicians, Emily Wells, at the Doug Fir, a small venue that is standing room only.

I have seen Ms. Wells 6 or 8 times over the last decade, and, sadly, that may well be my last. I can't do it anymore. I'm too old. I can't stand for three hours at a concert.

We go to many concerts and are comfortable being the oldest in the audience by 20 or 30 years. But this time a young couple said we were so adorable being at the concert. So adorable. Sigh. The first time, but not likely the last, I was condescended to due to my old age. I am cautious with my elderly patients and parents, not to talk to them like they are 'adorable.' Now I am on the receiving end. Aging stinks on ice, but it is better than the alternative.

Remember the case from Wednesday, String Theory? Well, there's more.

My first job is to make the right diagnosis. Only after there is the correct diagnosis can proper treatment start. Me find bug, me kill bug, me go home. Simple enough most of the time.

But why? Why, why, why is there a particular infection. It is fun to consider causes, but it is rare to hit pay dirt, and usually, I am left with bad luck. Blind, stupid, simple, doo-dah, clueless bad luck.

Not this time.

On a hail Mary, I sent of immunoglobulins, and the patient has no IgG2 and IgG4. IgG2 mostly recognizes carbohydrates/polysaccharides, and what is a hypermucoviscos Klebsiella but a ball of polysaccharide?

An initial study that investigated its nature reported that it was distinct from the constitutive K2 capsule and the surface polysaccharide colanic acid. However, subsequent investigators have reported that it represented an increased amount of capsular material with the same components as the constitutive capsule. Whether this increased amount of polysaccharide represents an increase in the amount of the constitutive capsule or whether it is an extracapsular polysaccharide (exopolysaccharide) that is physically distinct but biochemically the same as the constitutive capsule remains contested.

and there is one other similar case:

...the first case report of a patient with hypermucoviscous K pneumoniae without liver abscess and in the setting of an IgG2 subclass deficiency.

So mine is the second, and as we said in grade school, two is higher than one.

This has been the patients' only complication of low IgG2. He had zero sinopulmonary infections as a child, so is this indication enough for monthly IgG infusions? I doubt it. It is expensive and does have complications:

...thirty two percent of patients given IVIG infusions experienced adverse events. Fever was the most common manifestation. Symptoms occurred within 1 to 6 h from onset of infusion, were affected by fast infusion rates, and managed by reducing the rate of infusion.

Given the lack of hypermucoviscous K. pneumoniae in the US, I also doubted the utility of IVIG as treatment and, as noted, he improved with old school drainage and antibiotics.

Rationalization

Virulence. 2013 Feb 15; 4(2): 107–118. Jan 9. doi: 10.4161/viru.22718 PMCID: PMC3654609 Hypervirulent (hypermucoviscous) Klebsiella pneumoniae A new and dangerous breed.

https://www.ncbi.nlm.nih.gov/pmc/articles/PMC3654609/

Open Forum Infect Dis. 2014 Sep; 1(2): ofu080. Published online 2014 Sep 9. doi: 10.1093/ofid/ofu080 PMCID: PMC4281779 Hypermucoviscous Klebsiella Syndrome Without Liver Abscess in a Patient With Immunoglobulin G2 Immune Deficiency

https://www.ncbi.nlm.nih.gov/pmc/articles/PMC4281779/

Asia Pac Allergy. 2013 Oct; 3(4): 249–256. Oct 31. doi: 10.5415/apallergy.2013.3.4.249 PMCID: PMC3826603 Adverse events of intravenous immunoglobulin infusions: a ten-year retrospective study

https://pubmed.ncbi.nlm.nih.gov/24260730/

Why, why, why

https://youtu.be/QedVjF5vpQE

POLL RESULTS

As I get older, I want to

  • become adorable 5%
  • become a very stable genius 26%
  • stay continent 33%
  • never winge about the young 12%
  • tell those punks to get off my lawn 16%
  • Other Answers 9%
  • Win one now and then.
  • retire
  • All of the above, plus stop dreaming about what was, and about what will never be.
  • Retain my memory.

More Like Cool Tub

Jan 10, 2018

The what is simple. Fevers and chills and altered mental status, and the sudden onset of severe focal back pain.

The patient has diabetes and a hematologic malignancy in remission.

Exam shows focal back pain and some minimal skin breakdown on the legs but nothing all that impressive.

The MRI shows lumbar discitis/osteomyelitis, and the blood cultures are growing P. aerugenosa.

The what is easy enough. Pseudomonas bacteremia with discitis. But why? I would expect a urinary source with the Batson's plexus to blame as I so like to do, but his UA/UC is negative.

Pseudomonas means water, so I start asking. Do you get in hot tubs or pools? Yep. Every day. The temperature? 102. What is your water source? A well. A lot of people who live around the city have private water supplies.

So I suspect it is the perfect storm of a poor host and repeated exposure, although she is adamant that the tub is well maintained with bromine.

Hot tub folliculitis is the famous Pseudomonal infection, but think of sepsis, and other infections, as well. My favorite from the literature was due to sex in a hot tub filled with stream water:

We report a case of a previously healthy 38-year-old male with acute prostatitis and concurrent Pseudomonas aeruginosa urosepsis. Pulsed-field gel electrophoresis analysis confirmed that the source of the organism was the patient's newly purchased hot tub, which was filled with water from a stream.

I will admit I did not ask about sex in the hot tub, although I have seen at least one endometritis case I think was from sex in a hot tub.

102 (38.9) is not that hot, well within the range where Pseudomonas can grow:

P. aeruginosa had the widest range of growth temperature (25-42 C) and also showed rapid growth.

And she used bromine instead of chlorine, which may be part of the issue, as least as these titles suggest (I can't get the full text)

Pseudomonas folliculitis: an outbreak associated with bromine-based disinfectants--British Columbia.

and

A retrospective comparison of the effectiveness of bromination and chlorination in controlling Pseudomonas aeruginosa in spas (whirlpools) in Alberta.

The patient was moved to a hospital more insurance-appropriate, so I never did know how she did or if they cultured the hot tub water.

Rationalization

J Clin Microbiol. 2009 May; 47(5): 1607–1608. Published online 2009 Mar 18. doi: 10.1128/JCM.02376-08 PMCID: PMC2681869 Pseudomonas aeruginosa Acute Prostatitis and Urosepsis after Sexual Relations in a Hot Tub.

https://www.ncbi.nlm.nih.gov/pmc/articles/PMC2681869/

Infection. 2011 Jun;39(3):265-9. doi: 10.1007/s15010-011-0096-6. Epub 2011 Apr 1. Fatal Pseudomonas aeruginosa pneumonia in a previously healthy woman was most likely associated with a contaminated hot tub.

https://www.ncbi.nlm.nih.gov/pubmed/21455711

Microbiol Immunol. 1982;26(1):15-24. The effects of temperature and pH on the growth of eight enteric and nine glucose non-fermenting species of gram-negative rods.

https://www.ncbi.nlm.nih.gov/pubmed/7087800

POLL RESULTS

I like to

  • sit in the hot tub contemplating life 28%
  • drink in the hot tub contemplating postural hypotension 22%
  • have sex in the hot tub contemplating microbiology 9%
  • eat in the hot tub 6%
  • go back in time in the hot tub 22%
  • Other Answers 13%
  • not soak in the fetid fermentations of body fluids and sloughings of strangers, insufficiently killed by chlorine.
  • watch other people in the hot tub innocently stew themselves.
  • I don't like a hot tub.
  • I vastly prefer a steam room.

Duration: Disease and Treatment?

Jan 16, 2018

The patient presents with RLQ pain and a red, tender bulge in the inguinal area. CT shows an abscess in the pelvis that extends next to the bladder as well. She goes to the OR, where the infection is drained, a small perforation in the bladder repaired, and there was also an awful mess of inflammation, thickened tissues, and mesh from a 25-year-old hernia repair.

Culture grows pure B. fragilis. Bowel source, right? But there was no bowel perforation found in the OR, and B. fragilis doesn't cause bladder infections.

Here is where the plot thickens. 9 years ago, she had an appendectomy complicated by an abscess that involved the mesh, and it grew out pure B. fragilis. The details are unavailable, and EPIC is like Sumerian tablets, hinting at what occurred in a place far away and distant in time. She does not remember exactly what they did for the abscess as she was 'pretty out of it' during the hospitalization.

Has this infection been festering for almost a decade? It seems that way, but I can find nothing like it in the PubMeds. No B. fragilis mesh infections, no chronic abscess lasting a decade. I know from medical lore that in the old days, before antibiotics and CT, that people could have peritoneal abscesses for extended periods of time. I just can't find personal 'bests.'

Early in my practice, I saw a mute, bed-bound kid with immune complex glomerular nephritis from an appendiceal abscess that we guessed might have been 6 months old. That may be my record as best I can remember.

The other thing mentioned in the notes was a question as to whether the patient had early-onset dementia. I didn't see it, but I wasn't really looking. She seemed OK to my cursory ID eyes.

But there is this interesting association, and association is not causation, between chronic bacterial inflammation/infection and dementia. In the modern world, we do not have a lot of chronic bacterial processes, but I offer for your consideration:

Data stemming from both epidemiologic studies and animal experiments seem to be rather encouraging, tending to confirm the hypothesis that Helicobacter pylori infection might influence the course of Alzheimer's disease pleiotropically.

and

Dementia risk was 1.6-fold higher (95% confidence interval [CI]: 1.4-1.83) in the chronic osteomyelitis cohort than in the control group.

and

Periodontitis appears to be associated with cognitive impairment after controlling for confounders such as age, sex, and education level.

Something to think about and yet another reason I do not want my immune system boosted. Ever.

Now the usual problem with infected mesh: with inflammation and scarring, to take all the mesh out is to take out most of her pelvic organs.

So metronidazole for how long? Got me. Cure will be difficult to define.

Rationalization

Scand J Infect Dis. 2006;38(11-12):1108-10. Vaginal mesh infection due to Bacteroides melaninogenicus: a case report of another emerging foreign body related infection.

https://www.ncbi.nlm.nih.gov/pubmed/17148089

Helicobacter. 2018 Feb;23(1). doi: 10.1111/hel.12454. Epub 2017 Nov 27. Review: Impact of Helicobacter pylori on Alzheimer's disease: What do we know so far?

https://www.ncbi.nlm.nih.gov/pubmed/29181894

Eur J Clin Microbiol Infect Dis. 2015 Jan;34(1):153-159. doi: 10.1007/s10096-014-2200-1. Epub 2014 Aug 7. Increased risk of dementia among chronic osteomyelitis patients.

https://www.ncbi.nlm.nih.gov/pubmed/25098680

J Periodontol. 2015 Feb;86(2):244-53. doi: 10.1902/jop.2014.140340. Epub 2014 Oct 27. Is periodontitis a risk factor for cognitive impairment and dementia? A case-control study.

https://www.ncbi.nlm.nih.gov/pubmed/25345338

Fashionista

Jan 17, 2018

Fashions come and go in medicine, and I am not talking sartorial.

At some point this century, the abbreviation for pneumonia became pna. PNeumoniA?!? Really? It occurred about the same time people started spinning their pens on the thumb, a nervous habit unknown when I was in medical school. This skill is likely to become lost thanks to the fidget spinner and the computer. I can't even find a pen in my hospital.

Also, at some point, it became common to write "ID recommendations appreciated" sometimes before the recommendations are even made. Maybe appreciated. Sometimes they are, and sometimes they are not. But it is a hollow sentiment. I much preferred the phrase by a new intensivist: "ID recommendations noted and will be considered."

Also, of late, it has become de rigueur to keep repeating blood cultures after antibiotics have been started to prove they are negative. I was never trained to do this, except for S. aureus. And people start counting days of therapy from the first negative blood culture. I have never done that.

Where did both of these fashions come from? Got me. It gets down to what you want to prove with the blood cultures and what you are going to do with the result.

If you are worried about an endovascular infection, as you are with S. aureus bacteremia, repeat positive blood cultures may prove the diagnosis and alter therapy.

For any other bug? Well, Candida is a tricky one, but that's it.

For Streptococci, antibiotics will render repeat cultures an exercise in futility, although I guess Americans need more exercise, and perhaps that is why it is done.

Same with most gram-negative rods. Repeating blood cultures is a waste of time and money:

Patients with S. aureus bacteremia or endovascular infection are at risk of persistent bacteremia. Achieving source control within 48 h of the index bacteremia may help clear the infection. Repeat cultures after 48 h are low yield for most Gram-negative and streptococcal bacteremias.

Even with S. aureus, once the repeat blood cultures are positive and you have a complicated bacteremia/endocarditis, you expect fevers and bacteremia to persist for a week or so, so why repeat the cultures as it does not alter therapy? Sure, sustained bacteremia makes a complication more likely, but what are you going to do? You don't change therapy or start looking for a complication of bacteremia until the patient remains febrile for longer than a week (or has focal signs and symptoms). At least I wouldn't. Patience is more often all that is needed, a virtue often lacking in modern inpatient medicine.

And if there is data to support the contention that the last negative blood culture is day zero, I can't find it. It is a universal recommendation that does add several days to the duration of therapy with subsequent cost, but if it improves outcomes, I can't find it either. To my mind, part of what defines an uncomplicated S. aureus bacteremia is negative repeat blood cultures at 24-48 hours, so adding a day or two to the 14 days is going to make a difference? Nope. And like I ever see an uncomplicated S. aureus bacteremia.

I have a complicated S. aureus bacteremia, which is pretty much most of them. They are going to get 28 or 42 days of antibiotics anyway. You are telling me that adding on a few days for the time it takes to get negative cultures will make a whit of difference? I don't buy it.

We give multiples of 7 for duration primarily as we have seven-day weeks. If we had 6 or 8 day weeks, the duration would be multiples of that.

Now there may be data out there for the count starting from the first negative culture, and if so, I will be happy to eat my words and change my practice. But until then, I remain unfashionable, not repeating blood cultures without a good reason, and counting day one of antibiotics from day one.

Rationalization

BMC Infect Dis. 2016; 16: 286. Published online 2016 Jun 13. doi: 10.1186/s12879-016-1622-z PMCID: PMC4906775 Sending repeat cultures: is there a role in the management of bacteremic episodes? (SCRIBE study).

https://www.ncbi.nlm.nih.gov/pmc/articles/PMC4906775/

JAMA. 2014 Oct 1; 312(13): 1330 1341. doi: 10.1001/jama.2014.9743 PMCID: PMC4263314 NIHMSID: NIHMS640142 Clinical Management of Staphylococcus aureus Bacteremia

https://www.ncbi.nlm.nih.gov/pmc/articles/PMC4263314/

POLL RESULTS

Fashions I do not understand:

  • tattoos 9%
  • man buns 18%
  • visible underwear 32%
  • integrative medicine clinics 23%
  • gluten free 11%
  • Other Answers 7%
  • The appeal of being "fashionable."
  • All of the above

Magic

Jan 22, 2018

Any sufficiently advanced technology is indistinguishable from magic.

Arthur C Clark

Not really true. I kind of agree with Technology isn't Magic: Why Clarke's Third Law always bugged me. But ID has always seemed so 18th century. Agar plates, serology’s, and, god forbid, a history and physical to make a diagnosis. It seemed that all the other specialties got the new high-tech gizmos that, if we were lucky, could be applied to ID. Like I can get a PET paid for as part of an ID workup.

But we are getting closer to Dr. McCoy's tricorder. PCR panels are broadening the differential diagnosis of a variety of infections. The MALDI-TOF is giving names to hitherto unheard of bacteria, at least unheard of by me. And molecular microbiology allows me to diagnose the ungrowables, a Clintonian description, I suppose, although far more than half of pathogens are ungrowables. Compared to seeing a colony on a plate, the 16S almost seems like magic.

The patient is a young female, on dialysis for years. The renal failure was due to congenital/anatomical issues. For several months she had been plagued by recurrent/refractory ascites. The fluid had 3000 WBC’s, mostly PMN’s, but gram stains, cultures, and histopathology were unrevealing. Biopsy showed fibrous adhesions and acute/chronic inflammation with reactive peritonitis.

But reactive to what?

Aparecium

We, and as always by we, I mean I, sent the specimen for 16S ribosome, and it came back Ureaplasma parum.

You may, or may not, remember the Mycoplasma hominis peritonitis from a while back; the diagnosis was also made by 16S.

Ureaplasma parum is part of the normal female gu tract:

This study shows that UP was the most frequent isolated ureaplasma species (92.6 %).

and I presume it reached the peritoneum by ascension. There are a smattering of cases of U. parvum peritonitis on the PubMeds, mostly in CAPD and transplant patients.

I wonder as I did with the Mycoplasma, if these organisms are underdiagnosed due to the stone knives and bearskins we usually rely on to diagnose infections.

How to treat? Doxycycline and specifically moxifloxacin looked the best, but U. parvum are often resistant.

Speciation indicated that U. parvum was the predominant Ureaplasma spp. conferring antimicrobial resistance.

and there appears to be country variability. So we will see.

Rationalization

Arch Gynecol Obstet. 2014 Feb;289(2):407-12. doi: 10.1007/s00404-013-2980-z. Epub 2013 Aug 4. Ureaplasma urealyticum and Ureaplasma parvum in women of reproductive age.

https://www.ncbi.nlm.nih.gov/pubmed/23912532

Antimicrob Agents Chemother. 2016 Jul 22;60(8):4793-8. doi: 10.1128/AAC.00671-16. Print 2016 Aug. Antimicrobial Susceptibility and Clonality of Clinical Ureaplasma Isolates in the United States.

https://www.ncbi.nlm.nih.gov/pubmed/27246773

BMC Infect Dis. 2014; 14: 171. Published online 2014 Mar 28. doi: 10.1186/1471-2334-14-171 PMCID: PMC3976045 Antimicrobial susceptibility patterns of Ureaplasma species and Mycoplasma hominis in pregnant women.

https://www.ncbi.nlm.nih.gov/pmc/articles/PMC3976045/

Technology isn't Magic: Why Clarke's Third Law always bugged me

https://io9.gizmodo.com/technology-isnt-magic-why-clarkes-third-law-always-bug-479194151

POLL RESULTS

My idea of magic is

  • 16 S ribosome test 14%
  • Irvin Johnson 32%
  • The Gathering 11%
  • Art Metrano https://www.youtube.com/watch?v=\_-SNrvHdZJE 4%
  • the stock market 32%
  • Other Answers 7%
  • 24 hours without pissing off my wife.
  • Donald Trump is president.

Second Degree

Jan 24, 2018

The last 48 hours have been, let's say, interesting for a grumpy old man who increasingly only wants to kvetch.

A couple of months ago, I mentioned that I have resting Richard face. At the time, I said I wasn't angry or upset; it is just that my baseline expression is to scowl. My dad was the same way.

Now I am not so sure. Sometimes, to quote one of my favorite writers, the stupid, it burns. Maybe that's why I scowl: burns hurt.

It is flu season. Lots of flu. Lots of flu testing.

The patient presents with fever, chills, and increased urinary frequency. Another has fever and chills on dialysis. A third presents with fever and groin pain.

The first had E. coli urosepsis, the second Streptococcal endocarditis, and the third had a scrotal cellulitis.

What they all had in common was flu testing when they hit the er. All three were, not unsurprisingly, negative. And not a one had symptoms that were even remotely respiratory, although the dialysis patient did get oseltamivir. Didn't help the endocarditis. But no cough, no sore throat, no runny nose, or scratchy eyes.

Flu is a respiratory illness, and the utility of a rapid flu test when the patient has no flu-like symptoms is zero.

Of course, no one seems to know what flu-like symptoms are. Patients, and I guess doctors, seem to think any febrile illness is a flu-like illness. And the stomach flu? Good god.

Flu-like symptoms:

Common signs and symptoms include fever with nonproductive COUGH

or other suggestive respiratory symptoms, often with myalgias or headache.

and

Fever, COUGH

, and rhinorrhea individually correlated with influenza infections (PPV: 0.30, 0.20, and 0.20, respectively; NPV: 0.92, 0.87, and 0.85, respectively).

and

When influenza is circulating within the community, patients with an influenza-like illness who have both COUGH

and fever within 48 hours of symptom onset are likely to have influenza.

Fever and a COUGH.

COUGH COUGH COUGH COUGH COUGH COUGH COUGH COUGH COUGH

I know. Nothing in medicine is 100%. But it is nice for the patient to have something that vaguely resembles a respiratory symptom before testing.

While I may fret about wasted money with flu testing, not understanding that influenza is a COUGHING illness has real-world catastrophic consequences when it leads to early diagnostic closure.

A day after Alyssa Alcaraz was sent home early from Green Acres Middle School due to an illness, her family took her to local Kaweah Delta Urgent Care in Visalia, California, where doctors diagnosed the seventh-grader with influenza. Alyssa was given ibuprofen, cough syrup (though she didn’t have a cough) and nausea medication to ease her vomiting, and was told to return home to rest.

She died a few days later from Streptococcal sepsis. No flu. And no symptoms of flu.

If the patient doesn't have respiratory symptoms, it likely ain't flu. Keep looking.

Rationalization

J Clin Epidemiol. 2005 Mar;58(3):275-9. Influenza-like illness criteria were poorly related to laboratory-confirmed influenza in a sentinel surveillance study.

https://www.ncbi.nlm.nih.gov/pubmed/15768487

Arch Intern Med. 2000 Nov 27;160(21):3243-7. Clinical signs and symptoms predicting influenza infection.

https://www.ncbi.nlm.nih.gov/pubmed/11088084

I never meta static infection I liked

Jan 29, 2018

The patient is admitted with pneumonia. It was a biphasic illness, a week of fevers, myalgias, headache, and a dry cough that resolved for about three days, enough to return to work. Then it came back with a vengeance, more chest pain, the cough now productive.

He is admitted with a nice, consolidated lobar pneumonia and pneumococcal bacteremia. Certainly been there, doing that this year.

Secondary bacterial infections after influenza have always been a problem. In 1918

...secondary bacterial infection occurred in approximately 17% of those diagnosed with influenza of which approximately 35% were fatal.

For H1N1

Bacterial pneumonia complicated between 25% and 50% of severe infections in both children and adults

although the spring/summer outbreak of H1N1 in 2009 may have been protective:

The fact that much of the 2009 pandemic occurred outside the regular season for pneumococcal disease in temperate regions may help to explain the lack of a marked increase in risk of pneumococcal infection at this time.

It is more than just trashing the lung that leads to bacterial infections. Influenza also messes large sections of the immune system to increase susceptibility:

Both innate and adaptive antibacterial host defenses are impaired in the context of preceding influenza virus infection.

Read all about it in the second reference if you love immunology; it makes my brain hurt.

He responded nicely to antibiotics expect his WBC continued to rise, and he had dull abdominal pain. The usual suspects for disseminated infection (joint, pericardium, and pleura) were negative, and he had no issues with HIV or immunoglobulins. Still, he did have a known abdominal aneurysm that had started to grow. And grow. It increased a centimeter in two days. Not good.

In the OR, the AAA was inflamed and surrounded by pus, although cultures were negative.

There are a surprising number of cases of S. pneumoniae infecting AAA's, about 40, although none I can find as a post-influenza complication. Pneumococcus usually spreads to spaces, not into solid tissues like clot, so I wonder if the immune dysfunction helped the organism along. Or it was 'just' bad luck.

He did fine with surgery and what will likely be on lifetime penicillin.

Rationalization

Influenza Other Respir Viruses. 2013 Sep;7 Suppl 2:105-13. doi: 10.1111/irv.12089. Bacterial and viral infections associated with influenza.

https://www.ncbi.nlm.nih.gov/pubmed/24034494

Influenza and Bacterial Superinfection: Illuminating the Immunologic Mechanisms of Disease

http://iai.asm.org/content/83/10/3764.full

Ann Vasc Surg. 2011 Feb;25(2):266.e9 - 16. doi: 10.1016/j.avsg.2010.07.014. Epub 2010 Oct 6. Abdominal infectious aortitis caused by Streptococcus pneumoniae: a case report and literature review.

https://www.ncbi.nlm.nih.gov/pubmed/20926247

Odd Odd

Feb 5, 2018

There are common presentations of common diseases, odd presentations of common diseases, common presentation of odd diseases and, what you never want to be, is the odd presentation of an odd disease.

The patient is young and healthy who presented to his primary doctor with shoulder pain in his dominant arm. Maybe, just maybe, he might have tweaked it playing softball. But zero medical problems, zero exposure history. The shoulder was swollen but not red or hot, and he had no fevers or chills. So it was treated as a musculoskeletal strain/injury, and the pain only worsened.

After a couple of weeks of no improvement and a variety of normal diagnostic tests, it was tapped. Pus. So he was admitted for I&D.

And the cultures grew? Gamella haemolysans. Okay. Why? No idea. Zero reasons I can find by history or physical.

There are maybe 8 cases of Gamella septic arthritis on the Pubmeds, often associated with endocarditis, which he did not have. It looks like I have reported one prior Gamella on this blog, an endocarditis.

There are 9 different Gamella, and they mostly reside in the mouth and upper gi tract. G. morbillorum and G. haemolysans are the most commonly reported. His teeth were fine, and he had no unusual dental habits, including khat use.

We report a 22-year-old man who presented in January 2009 in Djibouti for an aortic-abscessed endocarditis due to Gemella morbillorum (G. morbillorum). A voluminous aortic abscess that extended to the perimembranous ventricular septum was fistulized into the right atrium. Atrioventricular conduction abnormalities were observed. The portal of entry was dental with multiples caries and a periodontitis attributed to khat chewing.

I assume it was bad luck after routine dental care. It happens. Sometimes:

From each patient several blood samples were drawn aseptically during dental treatment and cultured. The majority of aerobic bacteria recovered belonged to Streptococcus sp and Gemella sp., anaerobic bacteria mainly belonged to Porphyromonas gingivalis and Peptostreptococcus sp.

I will note that outside of the above quote, the bacteremia/dental care studies I found did not mention Gamella.

Source control and a course of beta-lactam and the infection, at least, was cured. How much joint damage occurred is yet to be determined.

Odd presentation of an odd bug. Never good.

Rationalization

Rev Med Interne. 2010 Aug;31(8):e7-9. doi: 10.1016/j.revmed.2009.07.021. Epub 2010 Apr 21. [Lethal aortic endocarditis due to Gemella morbillorum in a Djiboutian khat user].

https://www.ncbi.nlm.nih.gov/pubmed/20413192

J Clin Periodontol. 1999 Jul;26(7):469-73. Bacteremia after dental treatment in mentally handicapped people.

https://www.ncbi.nlm.nih.gov/pubmed/10412852

Nutri-Matic Medicine

Feb 12, 2018

Sometimes I think people need to take a course before being admitted to the hospital called How to Tell Your Doctor Everything that Does Not Matter but Nothing that Does. But it seems people excel at telling me information that has nothing whatsoever to do with the issue at hand. It seems to be getting worse of late, perhaps due to all those commercials telling us to talk everything over with our doctors. Or is that just me?

But I listen and try not to look as bored and irritated as I feel as I listen to a pointless story about the dog or the fine points of chicken preparation. I want to know about the illness that brought you into the hospital, not about how hard it was to find your car keys.

So much information I want to ignore. Like the cultures.

The patient is admitted with acute respiratory failure with several days of progressive dyspnea and a modest cough of white sputum: no fevers or chills. He is obese, with sleep apnea, CHF, smoking-induced COPD, and bronchiectasis with a history of a very resistant Pseudomonas. Admitting CXR is hard to read, but no dense consolidation, and the CT the next day shows the usual uninterpretable bibasilar schmutz.

He is hypotensive, the WBC is 12, the gram stain shows no organisms but grows light Serratia.

The next day, after all sorts of ICU care, including cefepime, the WBC is normal, and he is off pressors. Doesn't look the least bit toxic.

Serratia pneumonia? Serratia, and other gram-negative pneumonias, should be really sick, with a necrotizing pneumonia, dense consolidation, and lots of organisms in the culture. He just isn't all that ID sick.

I wish we could more accurately diagnose pneumonia because, more often than not, we have a diagnosis produced by a medical Nutri-Matic machine:

He had found a Nutri-Matic machine which had provided him with a plastic cup filled with a liquid that was almost, but not quite, entirely unlike tea. The way it functioned was very interesting. When the Drink button was pressed it made an instant but highly detailed examination of the subject's taste buds, a spectroscopic analysis of the subject's metabolism and then sent tiny experimental signals down the neural pathways to the taste centers of the subject's brain to see what was likely to go down well. However, no one knew quite why it did this because it invariably delivered a cupful of liquid that was almost, but not quite, entirely unlike tea.

A diagnosis that is almost, but not quite, entirely unlike gram-negative pneumonia. Like most of the gram-negative pneumonias I see. They aren't. I want to stop the antibiotics.

But what if you are wroooooonnnnnggggg and we stop the antibiotics, and the patient does poorly. Look what happens to Indian peafowls (Pavo cristatus). Sudden death.

Maybe it is the real deal, but Serratia a very rare cause of community-acquired pneumonia and with nothing to clinically suggest the presentation was due to anything beyond stress from MOSF.

As I tell my referring physicians, sometimes my suggestions have the weight of Truth, like how to treat MSSA bacteremia. And sometimes, like this time, it is my opinion, although I think my opinion is closer to Truth than truth. I am the OVSG* after all.

We stopped antibiotics. The pateint did fine.

Rationalization

J Vet Med Sci. 2017 Dec 22;79(12):2048-2051. doi: 10.1292/jvms.17-0207. Epub 2017 Oct 30. Sudden death of an Indian peafowl (Pavo cristatus) at a zoo due to non-pigmented Serratia marcescens infection.

https://www.ncbi.nlm.nih.gov/pubmed/29081475

Eur J Clin Microbiol Infect Dis. 2008 Aug;27(8):657-61. doi: 10.1007/s10096-008-0485-7. Epub 2008 Mar 4. Clinical outcomes and risk factors of community-acquired pneumonia caused by gram-negative bacilli.

https://www.ncbi.nlm.nih.gov/pubmed/18317821

OVSG: original very stable genius. I hate posers.

Why so Cereus?

Feb 14, 2018

I got an invite today that I had to not so respectfully decline. The makers of ceftazidime-avibactam (currently $342 a dose) are having a speaker at Mortons (where the cheapest steak is $59) to discuss their product. I entered the speaker's name into Dollars for Docs. As of 2015, he has received $693,000 in payments from drug companies. Whoa. Probably close to a million my now. I guess academia just doesn’t pay all that well.

Although in 30 years I have accepted nothing from pharma (except for a Fleets enema that the Unasin rep sent me on his last day), I have been called a big pharma shill many times as part of my work at Science-Based Medicine. But really. $693,000?

And who goes to this stuff? Is there anyone who honestly thinks it will be more than a biased infomercial? At the end of the day, it is our patients who are footing the bill and paying for our meals with increased health care costs and bankruptcy. It’s like a doctor investing in tobacco stock.

The patient is in her mid-sixties, with a 40-year history of occasional heroin use. Two days before admission, she bought a hit of black tar from a new dealer and within 1.5 hours had severe rigors. The next day she repeated the process and again had rigors, only this time she was short of breath as well, so she went to the ER and was admitted with sepsis and a WBC of 1.6.

She was started on ceftazidime-avibactam to help defray the costs of a steak dinner (just kidding, it was the usual vancomycin/cefepime), and the next day she was all better. But. Both sets of blood cultures grew B. cereus, and the WBC jumped to 27K. So they called me to address the blood cultures. Why so Cereus?

The patient was perfectly stable with zero signs and symptoms. I think it was a transient, but real bacteremia, as Bacillus spp are the most common bacteria found in heroin:

In total, 68 isolates were retrieved from 49 out of 82 samples analyzed (60% culture positive). All isolates were endospore-forming, Gram-positive Bacilli. Completely absent were non-endospore-formers or Gram-negatives. The three most predominant species were Bacillus clausii, Bacillus (para)licheniformis, and Terribacillus saccharophilus.

and

Aerobic endospore-forming bacteria (Bacillus spp. and Paenibacillus macerans) were the predominant microflora isolated and at least one species was isolated from each sample. B. cereus was the most common species and was isolated from 95% of all samples with B. licheniformis isolated from 40%. Between one and five samples yielded cultures of B. coagulans, B. laterosporus, B. pumilus, B. subtilis and P. macerans. Staphylococcus spp. were isolated from 23 (40%) samples; S. warneri and S. epidermidis were the most common and were cultured from 13 (22%) and 6 (10%) samples respectively. One or two samples yielded cultures of S. aureus, S. capitis and S. haemolyticus. The remainder of the flora detected comprised two samples contaminated with C. perfringens and two samples with either C. sordellii or C. tertium.

I never say heroin is contaminated since that implies they are trying to make a sterile product. I was curious. It looks like heroin is about $200 a gram, so it is more expensive and less sterile the ceftazidime-avibactam. That should be the ad campaign for all these expensive antibiotics: cheaper than heroin and less bacterial contamination. It appears that low-level drug dealers only make $24,000 a year. Although others find it more profitable, they are on the apex of the drug dealing pyramid.

Hmmm. Maybe I should look into being on a speakers bureau, given this patient is uninsured and with little in the way of resources. Nah. I prefer to keep my soul.

The WBC indirectly supports my hypothesis this was a transient bacteremia as the first response to an injection of pyrogen is a neutropenia followed by a leukocytosis.

TNF elicited a very rapid neutropenia, reaching a nadir after 15 minutes, followed by a neutrophilia.

So I suggested that unless the bacteremia recurred, he needed no further diagnostic or therapeutic interventions.

Rationalization

Forensic Sci Int Genet. 2018 Jan;32:1-6. doi: 10.1016/j.fsigen.2017.10.001. Epub 2017 Oct 4. Isolation and whole genome analysis of endospore-forming bacteria from heroin.

https://www.ncbi.nlm.nih.gov/pubmed/29024922

J Med Microbiol. 2002 Nov;51(11):1001-8. An investigation into the microflora of heroin.

https://www.ncbi.nlm.nih.gov/pubmed/12448685

Effects on leukocytes after injection of tumor necrosis factor into healthy humans T van der Poll, SJ van Deventer, CE Hack, GJ Wolbink, LA Aarden, HR Buller and JW ten Cate

http://www.bloodjournal.org/content/79/3/693.short?sso-checked=true Can J

Infect Dis Med Microbiol. 2015 Mar-Apr; 26(2): 103–104. PMCID: PMC4419812 A cluster of Bacillus cereus bacteremia cases among injection drug users.

https://www.ncbi.nlm.nih.gov/pmc/articles/PMC4419812/

Blind Pigs and Acorns

Feb 19, 2018

The patient has short bowel requiring TPN and is pacemaker dependent.

Given this blog's nature, you likely have a vague sense of foreboding, knowing that this could be a bad combination since I lead with that information.

It was.

She became fungemic with C. albicans, and her line was removed without a resolution of the fungemia. Evaluation of the valves and pacer failed to demonstrate any vegetation, but her 1-3 beta D glucan was > 500 and remained so for the hospitalization.

So likely endocarditis or pacer infection or both. Cardiology was not thrilled with taking out the pacemaker as it was responsible for her heartbeat.

So we gave a course of micafungin then transitioned her to po fluconazole, making sure that she was actually absorbing the medication with drug levels. She did.

And then we, and by we, I mean I, followed the 1-3 beta D glucan. Over the next year, it slowly inched down until three months ago. Undetectable. Two months ago: undetectable. So I stopped the fluconazole, and a month later, still undetectable.

I called it a cure. Go me.

The one similar case report I found suggested it takes time for the beta D glucan to fall with Candida endocarditis:

The (1→03)-β-D-glucan assay began trending downwards at 197 days into treatment with oral fluconazole. After 16 months of therapy, fluconazole was stopped due to transaminitis. (1→3)-β-Dglucan levels were checked six weeks after the discontinuation of treatment and were negative.

Generally, I do not like to manage device infections without removal as medical therapy is unlikely to work. Pacer infections are not amenable to medical therapy, although I found one case report of success. So maybe it wasn't a pacer lead infection but a 'simple' endocarditis.

That you might be able to cure medically.

Interestingly, in that analysis survival among those receiving combination antifungal therapies appeared similar to that of those receiving adjunctive surgical therapy. One conclusion suggested by the authors was that newer antifungal therapies potentially lent hope to those who could not undergo surgical therapy. In our study, mortality did not differ between those undergoing surgical therapy and those receiving medical therapy alone.

I recall one TV IE I cured with amphotericin B and fluconazole many years ago and one death with C. parapsilosis that I can remember.

But the new antifungals do work on occasion allowing the blind pig to get the occasional acorn. But not truffles. That would be an anosmic pig.

Rationalization

Indian Heart J. 2015 Dec; 67(Suppl 3): S100–S102. Published online 2015 Dec 23. doi: 10.1016/j.ihj.2015.11.025v PMCID: PMC4799021 Pacemaker lead Candida endocarditis: Is medical treatment possible?

https://www.ncbi.nlm.nih.gov/pmc/articles/PMC4799021/

Eur J Clin Microbiol Infect Dis. Author manuscript; available in PMC 2009 Oct 6. Published in final edited form as: Eur J Clin Microbiol Infect Dis. 2008 Jul; 27(7): 519–529. doi: 10.1007/s10096-008-0466-x PMCID: PMC2757733 Candida Infective Endocarditis

https://www.ncbi.nlm.nih.gov/pmc/articles/PMC2757733/

Antimicrob Agents Chemother. 2015 Apr; 59(4): 2365–2373. Published online 2015 Mar 11. Prepublished online 2015 Feb 2. doi: 10.1128/AAC.04867-14 PMCID: PMC4356766 Candida Infective Endocarditis: an Observational Cohort Study with a Focus on Therapy

https://www.ncbi.nlm.nih.gov/pmc/articles/PMC4356766/

J Heart Valve Dis. 2017 Mar;26(2):208-210. (1→3)-β-D-Glucan Assay in Monitoring Response to Anti-Fungal Therapy in Fungal Endocarditis.

https://www.ncbi.nlm.nih.gov/pubmed/28820552

Can't Fight Evolution

Feb 21, 2018

The patient is a recalcitrant IVDA who keeps leaving the hospital before completing a course of antibiotics for her tricuspid valve MRSA endocarditis.

That is putting it mildly. The most she has stayed in hospital is 10 days, usually averaging 4-5 days, just until her fever resolves, when she needs to answer the siren call of heroin.

The result has been evolution in action. Each admission, the MIC to vancomycin has inched up from .5 to 1 to 2, and now it is 4.

She is kind of a human Mega-Plate.

Now the question is treatment. Vancomycin is out.

Daptomycin seems reasonable, but I worry we will breed resistance ever faster to this antibiotic.

Among daptomycin-susceptible MRSA isolates from patients who had never received daptomycin, higher daptomycin MICs tracked with increased resistance to killing by platelet-derived but not neutrophil-derived HDPs. These findings support the notion that endogenous exposure of MRSA to specific HDPs may play a role in selecting strains with an intrinsically higher daptomycin MIC phenotype.

What is endocarditis but platelet clumps and bacteria?

More is better for daptomycin and endocarditis, but I can't see that higher doses (10mg/kg) will lead to less resistance.

Ceftaroline? There is some data suggesting that increased MIC to vancomycin result in less efficacy of beta-lactams, so maybe high dose (q 8) ceftaroline. Beta-lactams are time-dependent in their killing, but higher concentrations in the blood should translate to higher vegetation levels, keeping levels above the MIC in the clot. Right?

Or maybe combination therapy? Vancomycin or daptomycin plus ceftaroline?

Ceftaroline plus daptomycin may be an option to hasten clearance of refractory staphylococcal bacteremia. Ceftaroline offers dual benefit via synergy with both daptomycin and sensitization to innate host defense peptide cathelicidin LL37, which could attenuate virulence of the pathogen.

Finally decided on high dose ceftaroline with an option of adding daptomycin if she did not respond, and as soon as I had the order in EPIC, she left AMA, never to be seen again.

Rationalization

The Evolution of Bacteria on a “Mega-Plate” Petri Dish (Kishony Lab). https://www.youtube.com/watch?v=plVk4NVIUh8

J Infect Dis. 2012 Oct;206(8):1160-7. Epub 2012 Aug 16. Reduced susceptibility to host-defense cationic peptides and daptomycin coemerge in methicillin-resistant Staphylococcus aureus from daptomycin-naive bacteremic patients.

https://www.ncbi.nlm.nih.gov/pubmed?term=22904338

Antimicrob Agents Chemother. 2010 Sep;54(9):4038-40. doi: 10.1128/AAC.00533-10. Epub 2010 Jun 28. Development of daptomycin nonsusceptibility with heterogeneous vancomycin-intermediate resistance and oxacillin susceptibility in methicillin-resistant Staphylococcus aureus during high-dose daptomycin treatment.

https://www.ncbi.nlm.nih.gov/pubmed/20585116

Antimicrob Agents Chemother. 2018 Jan 25;62(2). pii: e01554-17. doi: 10.1128/AAC.01554-17. Print 2018 Feb. Outcomes of Vancomycin plus a β-Lactam versus Vancomycin Only for Treatment of Methicillin-Resistant Staphylococcus aureus Bacteremia.

https://www.ncbi.nlm.nih.gov/pubmed/29133561

Open Forum Infect Dis. 2017 Spring; 4(2): ofx084. Published online 2017 May 2. doi: 10.1093/ofid/ofx084 PMCID: PMC5499876 Ceftaroline for Severe Methicillin-Resistant Staphylococcus aureus Infections: A Systematic Review

https://pubmed.ncbi.nlm.nih.gov/28702467/

Clin Ther. 2014 Oct 1;36(10):1317-33. doi: 10.1016/j.clinthera.2014.05.061. Epub 2014 Jul 10. Antimicrobial salvage therapy for persistent staphylococcal bacteremia using daptomycin plus ceftaroline.

https://www.ncbi.nlm.nih.gov/pubmed/25017183

ID: We make the guidelines. Then ignore them.

Feb 26, 2018

Guideline angst: I have mixed feelings towards guidelines. With one huge exception, I usually do not have much to quibble with when it comes to treatments.

When it comes to diagnostics? Not so much enthusiasm. Guidelines often seem to say to do everything in every patient every time and, like much of the medical world, underestimate the dangers of testing:

...clinicians more often underestimated rather than overestimated harms and overestimated rather than underestimated benefits. Inaccurate perceptions about the benefits and harms of interventions are likely to result in suboptimal clinical management choices.

For example, we have

Echocardiography is recommended for all adult patients with (S. aureus) bacteremia. Transesophageal echocardiography (TEE) is preferred over transthoracic echocardiography (TTE).

Note the 'all.' Evidently, ID docs think this guideline is a load of fetid dingo's kidneys since they ignore it:

Staphylococcus aureus bacteremia, transesophageal echocardiography (TEE) was performed in 24% of cases. Consulting Infectious Diseases physicians most frequently cited low suspicion for endocarditis due to rapid clearance of blood cultures and the presence of a secondary focus requiring an extended treatment duration as reasons for foregoing TEE.

I recently saw a patient with fungemia. The resident asked me if he should go ahead and call ophthalmology as the guidelines say

All nonneutropenic patients with candidemia should have a dilated ophthalmological examination, preferably performed by an ophthalmologist, within the first week after diagnosis (strong recommendation; low-quality evidence).

Again with the 'all.' Unlike the TEE recommendation, it is a strong recommendation based on low-quality evidence, aka pulling it out of, er, well, thin air. Yeah. Thin air. Sort of the current approach of the EPA on climate change. The data suggests

Ocular lesions related to candidemia were found in only 13/168 patients (7.7%), of whom 1 reported ocular symptoms (incidence of symptomatic disease in the whole population, 0.27% [1/365]). Ophthalmological findings led to a change in antifungal therapy in only 5.9% of cases (10/168), and performance of the test was not related to a better outcome. Ocular candidiasis was not associated with a worse outcome and progressed favorably in all but 1 evaluable patient, who did not experience vision loss. The low frequency of ophthalmoscopy and ocular involvement and the asymptomatic nature of ocular candidiasis, with a favorable outcome in almost all cases, lead us to reconsider the need for systematic ophthalmoscopy in all candidemic patients.

He has no visual issues, and since he is going to get two weeks fluconazole anyway as

No case of late-onset ocular candidiasis was detected in unexamined patients who received at least 14 days of antifungal treatment. Based on our results, it would appear that the recommended 2 weeks of treatment after the first negative blood culture are sufficient to avoid late-onset complications due to undiscovered Candida chorioretinitis in patients surviving for more than 60 days after the onset of candidemia.

for what is candidemia from ureteral obstruction, why get the eye exam?

There are two reasons to do a diagnostic test with its risks and costs: the results will alter the patients care, or it scratches an intellectual itch to answer a question. And that second reason is, sadly, no longer valid.

ID: We make the guidelines. Then ignore them.

Rationalization

Clinicians’ Expectations of the Benefits and Harms of Treatments, Screening, and Tests

https://jamanetwork.com/journals/jamainternalmedicine/article-abstract/2596010

Clinical Practice Guidelines by the Infectious Diseases Society of America for the Treatment of Methicillin-Resistant Staphylococcus Aureus Infections in Adults and Children.

https://www.idsociety.org/uploadedFiles/IDSA/Guidelines-Patient\_Care/PDF\_Library/MRSA.pdf

Clinical Reasoning of Infectious Diseases Physicians Behind the Use or Nonuse of Transesophageal Echocardiography in Staphylococcus aureus Bacteremia.

https://academic.oup.com/ofid/article-pdf/3/4/ofw204/18503792/ofw204.pdf

Clinical Practice Guideline for the Management of Candidiasis: 2016 Update by the Infectious Diseases Society of America

http://www.idsociety.org/Guidelines/Patient\_Care/IDSA\_Practice\_Guidelines/Infections\_By\_Organism-28143/Fungi/Candidiasis/

PLoS One. 2017 Oct 24;12(10):e0183485. doi: 10.1371/journal.pone.0183485. eCollection 2017. Is routine ophthalmoscopy really necessary in candidemic patients?

https://www.ncbi.nlm.nih.gov/pubmed/29065121

Am J Ophthalmol. 2015 Nov;160(5):1078-1083.e2. doi: 10.1016/j.ajo.2015.07.033. Epub 2015 Jul 31. Inpatient Ophthalmology Consultation for Fungemia: Prevalence of Ocular Involvement and Necessity of Funduscopic Screening.

https://www.ncbi.nlm.nih.gov/pubmed/26235339

The Exception?

Mar 5, 2018

The patient is a poorly controlled diabetic admitted with gi bleed. As part of the workup, a ua/uc was ordered as her presenting hypotension was not known to be due to an acute bleed. Two months ago, she had been in the hospital for an MSSA osteomyelitis that resulted in a forefoot amputation and included a foley. She has never had a documented bacteremia.

The ua was negative, but the cultures grew > 100,000 MSSA. The house staff are quite tuned in to the importance of S. aureus in any sterile body fluid as they have listened to many a rant about the need for ID consultation for S. aureus. So they called me.

What to do.

The rule is do not send a ua/uc if the patient has no symptoms and do not treat asymptomatic bacteriuria:

Treating asymptomatic bacteriuria in patients with diabetes, older persons, patients with or without indwelling catheters, or patients with spinal cord injuries has not been found to improve outcomes.

But none of the guidelines mention S. aureus, and S. aureus may be different. When S. aureus gets in the blood, it often goes to the urine. But how often is it the other way around?

Although urinary S. aureus may be the source of staphylococcal bacteremia, the proportion of patients with chronic S. aureus bacteriuria who subsequently become bacteremic is unknown.

Well, it is often enough:

Seventy-one patients had follow-up culture data; 58% of cultures were positive for S. aureus at 2 months (median duration of staphylococcal bacteriuria, 4.3 months). Sixteen patients had subsequent staphylococcal infections, occurring up to 12 months after initial isolation of S. aureus; 8 late-onset infections were bacteremic.

and

A total of 326 patients were included. Invasive S. aureus disease was documented within 12 months of bacteriuria in 56 patients (22.3% of methicillin-resistant S. aureus (MRSA) cases and 8.4% of methicillin-sensitive S. aureus (MSSA), p = 0.002)... CONCLUSIONS: S. aureus bacteriuria is associated with significant morbidity and mortality. Patients without UTI symptoms, those with MRSA bacteriuria, and those without pyuria were more to likely to have worse outcomes (invasive S. aureus disease or death).

The problem is there is no data that preemptive antibiotics make a difference for invasive complications, at least in the urinary tract. But it does work for central venous catheters:

Patients with Staphylococcus aureus colonization of an intravascular catheter but without demonstrated bacteremia within 24 h after intravascular catheter removal had a 24% (12 of 49 patients) chance of subsequent S. aureus bacteremia if they did not receive immediate antistaphylococcal antibiotics. Treatment within 24 h after intravascular catheter removal led to a 83% reduction in the incidence of subsequent bacteremia.

And there is that whole literature on the risk of subsequent invasive disease associated with new colonization of MRSA. The closest I can get from the guidelines for a reason to give antibiotics is

Antimicrobial treatment of asymptomatic women with catheter-acquired bacteriuria that persists 48 h after indwelling catheter removal may be considered.

But it has been two months since the foley, so that is a bit of a stretch.

As is always the case, I suspect that S. aureus is different and is an exception to the rule not to treat asymptomatic bacteriuria.

Based on the above, I suggested a short course of cephalexin.

Rationalization

Infectious Diseases Society of America Guidelines for the Diagnosis and Treatment of Asymptomatic Bacteriuria in Adults.

Asymptomatic Bacteriuria in Adults.

https://www.aafp.org/afp/2006/0915/p985.html

Isolation of Staphylococcus aureus from the Urinary Tract: Association of Isolation with Symptomatic Urinary Tract Infection and Subsequent Staphylococcal Bacteremia.

https://academic.oup.com/cid/article/42/1/46/396275

Scand J Infect Dis. 2013 Sep;45(9):688-95. doi: 10.3109/00365548.2013.803291. Epub 2013 Jul 1. Clinical significance of Staphylococcus aureus bacteriuria at a tertiary care hospital.

https://www.ncbi.nlm.nih.gov/pubmed/23808717

Clin Infect Dis. 2008 Jan 1;46(1):114-8. doi: 10.1086/524077. Bacteremic complications of intravascular catheters colonized with Staphylococcus aureus.

https://www.ncbi.nlm.nih.gov/pubmed/18171225

Water You Talking About?

Mar 7, 2018

We have lots of water here in the great Pacific Northwest. Oceans, rivers, creeks, streams, lakes, waterfalls, and rain. Lots of rain. Or at least we used to. While the metropolitan areas have centralized water systems, it is remarkable how many people are on well water. Or perhaps sickly water, since, as best I can tell, most prefer their water unfiltered and chemical-free but not so much bacteria-free. As a result, I ask every patient about the source of their drinking/bathing water.

Patient 1 had the sudden onset of a red, hot, tender, fairly new breast implant. It is removed, cultured, and grows Serratia marcescens. I ask her if she has pink in her shower on the white porcelain.

S. marcescens is commonly found growing in bathrooms (especially on tile grout, shower corners, toilet water line, and basin), where it manifests as a pink, pink-orange, or orange discoloration and slimy film feeding off phosphorus-containing materials or fatty substances such as soap and shampoo residue.

Her prior house did (as does mine), but she recently moved to an old house with the unfortunate design feature of all pink tiles and fixtures in the main bathroom. But I suspect it was showering in a haze of Serratia that led to the infection as Serratia infections have happened before from tap water:

Examination of the outbreak revealed epidemiological evidence that consumption of tap water from a contaminated faucet during receipt of oral medication was the mechanism of S. marcescens acquisition.

Although I will admit I can't find a specific example of Serratia in a showerhead, that's my story, and I am sticking to it.

Patient 2 is an elderly diabetic who had a subdural hematoma drained. He went home doing fine only to return with a seizure and a cranial epidural abscess that grew Enterobacter cloacae. They have well water with a filter that is rarely changed.

Enterobacter has been found in well water and drinking water, so I suspect that he showered in a spray of Enterobacter that tracked down the incision into the epidural space.

I have seen a hodgepodge of infections over the years I suspect have been from the tap and well water: E. coli, Serratia a couple of times, Achromobacter, and Enterobacter that I can remember. Often it is an SSI or central catheter infection.

Over 15 million U.S. households obtain their drinking water from private wells.

And the CDC has reported outbreaks of mostly GI illnesses and Legionella that are associated with well water and

These outbreaks resulted in at least 1,006 cases of illness, 124 hospitalizations (12% of cases), and 13 deaths.

As well as a smattering of heavy metal poisonings.

But I can find few reports of SSIs traced back to home tap or well water. A Pseudomonas here, a Legionella there, an atypical Mycobacterium everywhere. There has never been a systematic evaluation of SSIs from exposure to tap/well water.

As the NIH noted

Issues of source and quality of tap water used as a wound cleansing solution need to be carefully considered, as although it is acknowledged that hospital tap water can be delivered at a constant temperature (having firstly gone through a process ensuring that all harmful bacteria have been killed) the same cannot be said for tap water within the homes of patients.

and one review found

Current NICE guidelines recommend cleaning surgical wounds with sterile saline only for the first 48 h following skin closure. We found no evidence that washing wounds with tap water during this period increases the incidence of SSIs compared to keeping them dry.

But I wonder how many of the odd gram-negative rod SSI/catheter infections are from a perfect storm of tap and/or well water, a fresh incision, and bad luck.

Rationalization

Infect Control Hosp Epidemiol. 2006 Jul;27(7):774-7. Epub 2006 Jun 9. Acquisition of multidrug-resistant Serratia marcescens by critically ill patients who consumed tap water during receipt of oral medication.

https://www.ncbi.nlm.nih.gov/pubmed/16807859

Evaluation of Microorganisms of Drinking Water of Rafha City, Northern Borders, Saudi Arabia.

https://www.omicsonline.org/open-access/evaluation-of-microorganisms-of-drinking-water-of-rafha-city-northernborders-saudi-arabia-2157-7099-1000417.php?aid=80556&view=mobile

Overview of Water-related Diseases and Contaminants in Private Wells.

https://www.cdc.gov/healthywater/drinking/private/wells/diseases.html

Postoperative washing of sutured wounds.

https://www.sciencedirect.com/science/article/pii/S2049080116301212

POLL RESULTS

I

  • avoid water as fish do something in it 26%
  • use only sterile water 15%
  • use bottled water on my wounds, especially the fizzy kinds 7%
  • clean wounds with coke. If it rusts a nail, it should be hell on bacteria. 30%
  • have dogs or cats keep the wound clean 19%
  • Other Answers 4%
  • Believe in exercising the immune system. :-/

Two things I didn't know.

Mar 12, 2018

Well, it's a lot more than two. This blog is witness to all the things I didn't know. But every case is an opportunity to add another brick in the wall. So "You! Yes, you behind the bike sheds, stand still, laddy!" and get some education and thought control.

The patient is a young immigrant who has had a month of progressive fatigue, weight loss, nausea, and night sweats. He comes to Outside Hospital where work up shows a WBC of 13 with a left shift, anemia, elevated LFTs with an LDH of 680. The donut of truth, aka CT, shows diffuse interstitial infiltrates, multiple nodules in the liver and spleen, pancolitis, and adenopathy.

The CT report called it vasculitis, and he was transferred to my institution for further care. He had a bronchoscopy to work up the vasculitis, and the AFB on the BAL was positive and identified a few days later as M. tuberculosis complex.

It was fairly classic miliary TB, but then I have seen many cases over the years and have not had a good time with the disease. My very first consult was a patient who became neutropenic from Bactrim, reactivated his TB, and died of miliary disease. We made the diagnosis at autopsy. My second consult was an alcoholic who died with persistent fevers. We made the diagnosis of cardiac aspergillosis at autopsy. Such an auspicious beginning to my ID practice. My record did get a bit better.

But what is miliary TB but a vasculitis? So the radiologist wasn't that far off.

I think of two infections that cause a sky-high LDH: PJP and disseminated histoplasmosis. Add miliary TB to the list.

All patients diagnosed from 1996 to 2006 were reviewed. TB was diagnosed in 65 patients and NTM in 31. Patients with TB had higher median levels of aspartate aminotransferase (AST) (69.0 vs. 45.0, P = 0.02) and lactate dehydrogenase (LDH) (725.0 vs. 569.0, P = 0.03).

and

Patients with a serum LDH level <225 data-preserve-html-node="true" IU/L had a mean CD4+ lymphocyte count of 159/dl (SE 19.3) as compared to patients with a serum LDH level > or =225 IU/L, who had a mean CD4+ lymphocyte count of 58/dl (SE 6.9) (P<0.01). data-preserve-html-node="true" Non-Caucasian race, a diagnosis of histoplasmosis, disseminated tuberculosis or Pneumocystis carinii pneumonia, and CD4+ lymphocyte count were significantly associated with a serum LDH level > or =225 IU/L in the bivariate analysis.

Although my patient is HIV negative. He was placed on RIPE and, with the colitis, gave the rifampin iv as well as added an iv quinolone. And some vitamin D, since his level was unmeasurable. I wanted to make sure he was getting his meds. And we waited. One week goes by, then a second, then a third—everyday fevers to 39. I am used to the fevers resolving fairly rapidly when treating TB. No other infection was found, although sensitivity and drug levels are still pending. The CXR continued to improve, and the patient did not worsen clinically.

Just how long will fevers last with TB? Usually a week or two, but it can be a long time:

Amoung the 13 patients with fever that lasted 21 days or more the duration was 30 days in 10, 40 days in 8, 60 days on 6, 90 days in 4, 110 days in 2 and 120 days in one.

That's a long time to have a fever.

Eventually (5 weeks), the fever resolved, and the patient was a cure.

Rationalization

Int J Tuberc Lung Dis. 2008 Nov;12(11):1340-3. Laboratory features for presumptive diagnosis of disseminated tuberculosis in HIV-infected patients.

https://www.ncbi.nlm.nih.gov/pubmed/18926048

Int J Infect Dis. 2002 Sep;6(3):178-81. The association of serum lactate dehydrogenase level with selected opportunistic infections and HIV progression.

https://www.ncbi.nlm.nih.gov/pubmed/12718831

Prolonged Fever in Patients Treated for Tuberculosis.

https://www.atsjournals.org/doi/pdf/10.1164/arrd.1968.97.1.140

Mazel Tov

Mar 14, 2018

Or is it MALDI-TOF? One is "used to express congratulations for a happy and significant occasion or event," the other is expanding our understanding of microbiology. For an ID doc, that might be one and the same.

The patient presents to Outside Hospital with fever, rigors, and altered mental status. She has long-standing diabetes and has been on dialysis for a decade through a fistula with no issues. She had been fine before the symptoms and currently has no localizing symptoms.

She gets the usual ICU care, and both sets of admission blood cultures grow a gram-positive rod. It seems to be the real deal, but what is it going to be?

Actinomyces turicensis according to the MALDI-TOF.

The heck.

The patient did well, was changed to po clindamycin because of well-documented anaphylaxis to penicillin. She showed up in my clinic two months later, doing well—no signs and symptoms of ongoing infection.

What to do?

There are many Actinomyces like organisms including, A. turicensis, A. radingae, and A. europaeus, that can be pathogenic.

In a review, A. turincensus was associated with balanitis, urethritis, prostatitis, a variety of female -itisis, UTI, soft tissue infections, some gi infections, and rarely bacteremia. More often than not, it is part of mixed cultures. She had none of the above.

Bacteremia is rare, with a grand total of three cases and associated with dental issues. She had a pan scan for abscess at Outside Hospital with nothing found, although she has been bothered with peri-rectal abscesses on and off.

So is the bacteremia from the abscess? The dialysis shunt? She is doing just fine on the clindamycin, and while there were no sensitivities done, around 18% are reported resistant to clindamycin:

A. turicensis strains showed resistance to clindamycin, tetracyclines (doxycycline and tetracycline), macrolides (clarithromycin and erythromycin), ciprofloxacin, and/or linezolid...

Probably the vancomycin in the hospital did the trick for what was a transient bacteremia from who knows where.

Too many questions left unanswered to be a satisfying case, but as long as the patient does well, I can deal as I got to learn about yet another bacterial species had not yet encountered.

In total, there are ∼1,400 known species of human pathogens (including viruses, bacteria, fungi, protozoa and helminths), and although this may seem like a large number, human pathogens account for much less than 1% of the total number of microbial species on the planet.

Make that 1401.

Rationalization

Clinical Spectrum of Infections Due to the Newly Described Actinomyces Species A. turicensis, A. radingae, and A. europaeus.

http://jcm.asm.org/content/37/1/8.full

J Antimicrob Chemother. 2005 Aug;56(2):407-9. Epub 2005 Jun 21. Antimicrobial susceptibility testing of Actinomyces species with 12 antimicrobial agents.

https://www.ncbi.nlm.nih.gov/pubmed/15972310

Clin Microbiol Rev. 2015 Apr; 28(2): 419–442. Published online 2015 Mar 18. doi: 10.1128/CMR.00100-14 PMCID: PMC4402957 Actinomyces and Related Organisms in Human Infections

https://www.ncbi.nlm.nih.gov/pmc/articles/PMC4402957/

Microbiology by numbers.

https://www.nature.com/articles/nrmicro2644.

We do what we do because we do it

Mar 19, 2018

The patient is a mostly homeless intermittent iv drug user who has the acute onset of a red, hot, swollen knee. 6 years ago, she had an MSSA infection of the same knee. It received a washout and three weeks of iv antibiotic, and the knee had caused her no issues since.

This time it grew Group B Streptococcus, and I was consulted.

I was taught back in the mists of time that S. aureus and gram-negative rod spontaneous septic joints received 3 weeks of iv, and Streptococcal warranted 2 weeks of iv. And so that is what I have been doing for maybe 30 years.

But it occurred to me as I was writing up the consult, just what is the data? I really didn't know the clinical trials that led to this recommendation. It is based on data, right?

The first stop was Up To Date, where the authors recommend 2 weeks of iv followed by two weeks of po. Another guideline

recommends parenteral therapy for 2 weeks followed by 4 weeks of oral therapy. S. aureus infection and Gram-negative septic arthritis requires 4 weeks of parenteral therapy.

Based on?

Nothing. In my experience, aka the three most dangerous words in medicine.

So I went looking.

There is nothing out there to support long courses of iv antibiotics for septic native joints. Nothing. Zip. Nada. Nil. Zilch. It is remarkable how much time and resources have likely been spent following recommendations that are based on the thinnest of gruel, expert opinion. Hey, I'm an expert too. Do what I say instead.

There is, however, data to suggest that short courses of iv are adequate and rapid change to oral antibiotics works just fine.

Median antimicrobial treatment duration was 14 days (IQR, 12-28 days), with amoxicillin/clavulanate administered in 74 (85%) episodes. At follow up, cure of infection was noted in all episodes and good functional outcome in 79% of episodes.

and

Oral antibiotic therapy is non-inferior to IV therapy when used during the first six weeks in the treatment for bone and joint infection, as assessed by definitive treatment failure within one year of randomisation.

and

Seven days of intravenous therapy had the same success rate as 8-21 days (OR 0.4, 95% CI 0.1-1.7) and >21 days (OR 1.1, 95% CI 0.4-3.1). Fourteen days or less of total antibiotic treatment had the same outcome as 15-28 days (OR 0.4, 95% CI 0.1-2.3) or >28 days (OR 0.4, 95% CI 0.1-1.6).

CONCLUSIONS: In this retrospective study of adults with septic arthritis, the duration of antibiotic therapy, or an early switch from intravenous to oral administration, did not statistically influence the risk of recurrence.

This century has made it clear that treating infections is all about source control. Control the source, and short courses of antibiotics are all that is needed. If the joint gets a good washout, all the patient likely needs is a week. Tops. And most of it oral. With the caveat that it is a normal host with a normal joint and a susceptible organism.

It will be interesting to see what the new IDSA guidelines say when released this fall. I am feeling psychic: I bet they continue to recommend the 2 -4 week courses of iv antibiotics because of a lack of clinical trials with the usual A-III justification, where A is Good evidence, and III is the source of the evidence:

evidence of respected authorities, based on clinical experience, descriptive studies or reports of expert committees.

A and III never belong together.

Me? I go with the data. Short course it is.

And my flabber remains gastered the basis of some guidelines.

Rationalization

Clinical Management of Septic Arthritis. Curr Rheumatol Rep (2013) 15:332 DOI 10.1007/s11926-013-0332-4

https://www.med.unc.edu/tarc/events/event-files/septic%20arthritis%20management.pdf

Open Forum Infect Dis. 2017 Mar 25;4(2):ofx058. doi: 10.1093/ofid/ofx058. eCollection 2017 Spring. Pyogenic Arthritis of the Fingers and the Wrist: Can We Shorten Antimicrobial Treatment Duration?

https://www.ncbi.nlm.nih.gov/pubmed/28491895

Oral Versus Intravenous Antibiotics for the Treatment of Bone and Joint Infection (Oviva): A Multicentre Randomised Controlled Trial

https://online.boneandjoint.org.uk/doi/abs/10.1302/1358-992x.2017.22.042?journalCode=procs&;

J Infect Dis. 1987 May;155(5):968-72. Oral antimicrobial therapy for adults with osteomyelitis or septic arthritis.

https://www.ncbi.nlm.nih.gov/pubmed/3494088

Int J Infect Dis. 2013 Mar;17(3):e199-205. doi: 10.1016/j.ijid.2011.12.019. Epub 2012 Nov 22. Short parenteral antibiotic treatment for adult septic arthritis after successful drainage.

https://www.ncbi.nlm.nih.gov/pubmed/23183231

A-III: A contradiction in evidence.

http://boards.medscape.com/forums/?128@@.2a8129d6!comment=1

POLL RESULTS

I think

  • more is always better than less 0%
  • iv is better than po 0%
  • I should use a random number generator to determine duration 37%
  • people worry overmuch about facts and data 11%
  • we do what we do, doobie doobie do 44%
  • Other Answers 7%
  • therefore I am a secular humanist.

The Real Deal

Mar 21, 2018

The patient had the abrupt onset of progressive low back pain. No other issues, fevers, chills, etc. It came out of the blue. First treated as the usual back strain, although no strain by history and the pain only worsened.

So an MRI was done that was consistent with discitis/osteomyelitis/paravertebral abscess, and due to intractable pain, she was admitted.

Evaluation yields none of the usual suspects for a back infection. She had a Mohs procedure three months ago, but it was uneventful, and it is my understanding that hearing issues are the biggest complication. Worst attempt of humor yet, n'est pas?

As always, I want a bug. But how. We usually start and end with a biopsy. A recent study suggests maybe we should start with an open biopsy:

Open biopsy of vertebral tissue had a higher diagnostic yield (70%) than fine needle aspirate (41%) or core biopsy (30%)... Obtaining a microbiological diagnosis is associated with a better outcome.

And what a concept: know what you are treating leads to a better outcome. Strike another blow against empiricism.

Blood cultures were done but only grew a coagulase-negative Staphylococcus. Ignore it.

But a biopsy grew the same Staphylococcus epidermidis as the blood, rapidly and in all three specimens.

Hard to ignore that.

Two of the 88 in the above study had a coagulase-negative Staphylococcus as the cause of the vertebral osteomyelitis. There are a smattering of spontaneous Staphylococcus epidermidis vertebral osteomyelitis in the literature, but it was not high on my list of causes. I think it is my first case.

Now a course of antibiotics and improvement. I hope.

Rationalization

Epidemiology, Microbiological Diagnosis, and Clinical Outcomes in Pyogenic Vertebral Osteomyelitis: A 10-year Retrospective Cohort Study Open Forum Infectious Diseases, Volume 5, Issue 3, 1 March 2018,

https://doi.org/10.1093/ofid/ofy037

Clin Exp Rheumatol. 2000 Jan-Feb;18(1):115-6. Haematogenous vertebral osteomyelitis caused by Staphylococcus epidermidis: report of 4 cases.

https://www.ncbi.nlm.nih.gov/pubmed/10728462

J Med Microbiol. 2010 May;59(Pt 5):599-601. doi: 10.1099/jmm.0.013730-0. Epub 2010 Jan 21. Spontaneous vertebral osteomyelitis due to Staphylococcus epidermidis.

https://www.ncbi.nlm.nih.gov/pubmed/20093381

A Steel Boot

Mar 26, 2018

Spoiler alert: If you want to read the Shattered Seas, and who doesn't, stop here.

"Pride"

Thorn Bathu on why she lost to Grom-Gil-Gorm.

It is so easy to think you know what you are doing and so easy to be wrong. Because, to paraphrase Teen Talk Barbie, medicine is tough.

The patient has a whopping case of pancreatitis, one of the worst I have seen in a long time.

I am called for a leukemoid reaction, her WBC running 50K. Work-up yields nothing unsurprising: lots of phlegmon in the abdomen but almost nothing else of note. I set a 1-3 beta D glucan, but it was barely positive, a bit over 100.

I figured that if he was having Candida or some other cause of a beta D glucan, it should be obvious, and the beta D glucan should be much higher. Over time the phlegmon ripened into some abscesses, one of which was sampled. Nothing. Sterile. A repeat beta D glucan was also around 100. And the leukocytosis persisted. It went on for a couple of weeks and never responded to antibiotics. She was intermittently febrile, and I passed it off to severe pancreatitis.

The phlegmon was followed and eventually ripened into a large, rind-covered collection that between CTs dissected down into the thigh and was taken to the OR for drainage.

And it all grew C. albicans and nothing but C. albicans.

I was so sure this wasn't Candida. In retrospect, the steel boot of my thinking was the leukemoid reaction.

(1-->3)-beta-D-glucans are recognized by macrophages and, perhaps, neutrophils and natural killer cells via a (1-->3)-beta-glucan specific receptor. Following receptor binding...

White cells bind beta D glucan.

I have always known that the fewer the white cells in the blood, the more reliable the fungal antigen assays. But it never quite clicked that white cells were likely acting as a sponge, and with 50,000 in the blood, not to mention the WBC in the phlegmon, my patient was a mega-sponge for beta D glucan.

With intra-abdominal Candida colonization, beta D glucan is around 99 and disease around 250.

It did not, I think, make any difference in the patients outcome as the quantity of phlegmon would have required source control at some point. But still. I thought I knew what was going on. Nope. That's what is great about medicine—so many opportunities to be wrong.

Rationalization

Overview of (1→3)-β-D-glucan immunobiology.

https://www.ncbi.nlm.nih.gov/pmc/articles/PMC2365860/

Front Immunol. 2013; 4: 230. Published online 2013 Aug 12. doi: 10.3389/fimmu.2013.00230 Binding of Soluble Yeast β-Glucan to Human Neutrophils and Monocytes is Complement-Dependent.

https://www.ncbi.nlm.nih.gov/pmc/articles/PMC3740326/

Am J Respir Crit Care Med. 2013 Nov 1;188(9):1100-9. doi: 10.1164/rccm.201211-2069OC. β-glucan antigenemia anticipates diagnosis of blood culture-negative intraabdominal candidiasis.

https://www.ncbi.nlm.nih.gov/pubmed/23782027

Dry Tap

Mar 29, 2018

The patient is a middle-aged human who reached for a falling glass and tweaked her lower back. The pain was moderately severe at onset but over the next week increased in severity to the point she was bedbound.

On the day of admission, she became acutely delirious and febrile and, in the ER, had a stiff neck, and marked leukocytosis. Past medical history was only significant for alcohol and tobacco use and terrible dentition.

After a quick CT of the head, an LP was performed, and they were certain they were in the right place, but no fluid came out.

The usual antibiotics for meningitis were started, and the next day, the blood cultures are growing gram-positive cocci in pairs and chains, so they call me.

I suggested an MRI of the back as I was betting an epidural abscess/discitis that ruptured into the CSF as the cause of meningitis. I saw that once before as an intern with S. aureus, although the blood gram stain did make the diagnosis suspect as Pneumococcus, which I thought it would be, is not much of an abscess former.

MRI did not show an abscess but a phlegmon from L2-L5 and less impressive phlegmon in the thoracic spine.

That is the annoying thing about epidural abscesses: there can be noncontiguous asymptomatic lesions, although, given the coma, we could not tell just how asymptomatic they were.

Concurrent noncontiguous SEAs are even more challenging to diagnose because whole-spine imaging is not routinely performed unless the patient demonstrates neurologic findings that are inconsistent with the identified lesion. Failure to recognize a separate SEA can subject patients to a second operation, continued sepsis, paralysis, or even death...

...comparing patients with skip SEA lesions (n=22) with those with single lesions (n=211) demonstrated significant differences in three factors: delay in presentation (defined as symptoms for ≥7 days), a concomitant area of infection outside the spine and paraspinal region, and an erythrocyte sedimentation rate of >95 mm/h at presentation

So she had 2/3. Maybe even 3/3, but who gets sed rates? But at least we had a good explanation for the dry tap. And the cultures? Group B Streptococcus. Not high on my list of causal agents, but it happens.

And was it meningitis or an extensive epidural abscess? I know, probably a distinction without significance. We never did get a clear-cut CSF sample. She was decompressed, her MRI of the head showed no enhancement, her mental status never improved, but there were CNS issues separate from the infection.

And the source? Her urine did grow a smattering of Group B Streptococcus, but due to bacteremia or the initial source? Regular readers know how much I love to blame Batson's plexus. It is nice to know I am not the only one:

...made it possible for S. agalactiae to cause an infection of the epidural space or paraspinal muscles via the spinal valveless venous system named Batson's plexus communicating with the sacral, pelvic, and prostatic venous plexus. Our case indicated the importance of Batson's plexus in group B streptococcus infections surrounding the spinal canal and the necessity to explore for intrapelvic lesions...

Rationalization

Spine J. 2015 Jan 1;15(1):95-101. doi: 10.1016/j.spinee.2014.06.008. Epub 2014 Jun 17. Predicting patients with concurrent noncontiguous spinal epidural abscess lesions.

https://www.ncbi.nlm.nih.gov/pubmed/24953159

Am J Emerg Med. 2013 Jul;31(7):1155.e5-6. doi: 10.1016/j.ajem.2013.03.013. Epub 2013 May 16. Panspinal epidural abscess concomitant with meningitis.

https://www.ncbi.nlm.nih.gov/pubmed/2368505

Neurologia. 2015 Apr;30(3):158-62. doi: 10.1016/j.nrl.2013.12.002. Epub 2014 Sep 10. Central nervous system involvement in adult patients with invasive infection caused by Streptococcus agalactiae.

https://www.ncbi.nlm.nih.gov/pubmed/25217065

Rinsho Shinkeigaku. 2011 Jul;51(7):493-8. [Group B streptococcus meningitis and infection surrounding the spinal canal caused by bacterial transmission from rectal ulcer via Batson's plexus].

https://www.ncbi.nlm.nih.gov/pubmed/21823509

The Next Martian?

Apr 2, 2018

The consult, a diabetic dialysis patient, is billed as a bad cellulitis.

I go see her, and the cellulitis does look bad—the typical look of bullous cellulitis from Group A Streptococcus. Large, torn blisters, subcutaneous bleeding, swelling, but the erythroderma outline is receding, the WBC is stable in the mid-teens, no fever, and for what it is worth, the patient doesn't look all that sick.

An old surgeon told me in my first year of practice, "never worry about a patient who can smile," and in my experience (yes, I know), I have found it a reliable marker for a patient who is going to do fine. If anyone has done a study on a patients ability to smile and the severity of illness and outcome, I can't find it.

The patient was on vancomycin/piperacillin-tazobactam, and who isn't? But not for long. I switched her to cefazolin and grumbled about the excessive antibiotics for cellulitis. Three drugs should be used for cellulitis: cefazolin, ancef, and kefzol. Anything else is overkill. And if you think broad-spectrum antibiotics are harmless beyond cost (and renal failure for the vanco-pip/tazo combo), I give you an

increased risk of sepsis within 90 days of discharge among patients with exposure to high-risk antibiotics or increased quantities of antibiotics during hospitalization.

those antibiotics being

such as third- or fourth-generation cephalosporin, lincosamide, fluoroquinolone, β-lactam/β-lactamase inhibitor combinations, oral vancomycin, and carbapenem were associated with an increased risk of sepsis. Furthermore, significant dose-response effects were observed for the number of antibiotic classes a patient received during the index hospitalization as well as the total duration of therapy. In contrast, the risk of postdischarge sepsis for exposure to low-risk antibiotics was diminished.

So I put her on cefazolin.

Then on day 5, a blood culture turned positive for a gram-negative rod. But the patient was doing fine, so I waited for an identification, wondering if it would be a Vibrio or Helicobacter, rare gram-negative rods that cause cellulitis. However, the lab told me it didn't look like those organisms on gram stain. And it wasn't.

Brevundimonas diminuta.

I never heard of that before. You?

Liar.

Found in the water/dirt, Brevundimonas rarely causes human disease, with the usual hodgepodge of infections, including one cellulitis and 16 other case reports. I assume this is a real bacteremia, but given the extensive blisters on the skin, I have wonder if it was a bacteremia from skin breakdown rather than primary cause of disease.

The bug is often resistant to quinolones but sensitive to most everything else. And she continued to get better.

If this were a journal article, I would end with the usual lame and stupid warning that in the future, you too should consider Brevundimonas diminuta in your next case of cellulitis. Don't. You will never see a case. Trust me. Keep treating for Streptococci.

I will end instead by noting one strain of Brevundimonas is quite hardy and could give Matt Damon a run for the money:

The high flux of cosmic rays onto the unshielded surface of Mars poses a significant hazard to the survival of martian microbial life. Here, we determined the survival responses of several bacterial strains to ionizing radiation exposure while frozen at a low temperature characteristic of the martian near-subsurface... Brevundimonas sp. MV.7 emplaced only 30 cm deep in martian dust could survive the cosmic radiation for up to 100,000 years before suffering 10⁶ population reduction.

That is something to consider.

Rationalization

Bad Cellulitis

http://boards.medscape.com/forums?128@@.2a7d9b7a!comment=1

The Role of Experience in Science-Based Medicine.

https://sciencebasedmedicine.org/the-role-of-experience-in-science-based-medicine/

Crit Care Med. 2018 Jan;46(1):12-20. doi: 10.1097/CCM.0000000000002769.

Vancomycin Plus Piperacillin-Tazobactam and Acute Kidney Injury in Adults: A Systematic Review and Meta-Analysis.

https://www.ncbi.nlm.nih.gov/pubmed/29088001

Risk of Subsequent Sepsis Within 90 Days After a Hospital Stay by Type of Antibiotic Exposure

https://doi.org/10.1093/cid/cix947

Astrobiology. 2010 Sep;10(7):717-32. doi: 10.1089/ast.2009.0439. Low-temperature ionizing radiation resistance of Deinococcus radiodurans and Antarctic Dry Valley bacteria.

https://www.ncbi.nlm.nih.gov/pubmed/20950171

Do you need more than steroids?

Apr 4, 2018

The patient is a middle-aged male admitted with S. aureus bacteremia from a port. The port is there for chemotherapy; he has metastatic cancer to the CNS and has been on high dose dexamethasone for edema from multiple irradiated brain metastasis.

Looking at his chest x-ray, I see no emboli but a diffuse, subtle interstitial infiltrate, the kind I always overcall. I would make a horrible radiologist; every CXR looks abnormal to me. His LDH is in the mid-300's, but he does have dying cancer. PJP? He has zero pulmonary symptoms. But here is a pearl: patients get symptomatic PJP as their steroids are weaned, and his dose is stable. So the lack of symptoms did not bother me overmuch. I sent off a beta D glucan, and it came back in the high 300's. So presumptive PJP it was.

One time, and one time only, have I had a case of PJP where the only risk was steroids, dexamethasone for a brain tumor that did not get chemo or radiation. When we weaned the steroids, he herniated, and when we increased the steroids, the PJP worsened. He died of respiratory failure. Awful case.

And this case is not my second, as he had a history of myeloma, albeit in remission for a couple of years.

Just how much of a risk is steroids, and only steroids, for PJP? It is hard to tell. Most patients have other comorbidities, such as rheumatologic disease:

During a total of 1474.4 person-years, 30 PCP cases occurred

I know that patients are often put on tmp/sulfa for more than 40 mg a day of prednisone for PJP prophylaxis. I had to grimace at rheumatology paper where they said

Only two serious ADRs (including one Stevens-Johnson syndrome case) occurred.

Only. I hate only. Like a case of Stevens-Johnson syndrome warrants an only. Tmp/sulfa is many things, but safe is not on the list.

Reviews suggest

Although there are a number of case reports in the literature, the only collagen vascular disease with an increased incidence of PJP is Wegener granulomatosis... There is currently no evidence to recommend PJP prophylaxis in the non-HIV/AIDS immunocompromised population

and

In summary, the role of PJP prophylaxis in non-HIV patients on chronic steroids remains controversial.

But side effects of antibiotics are usually under-appreciated, and patients and doctors are more forgiving of complications from action than inaction. I suspect both would take a dozen cases of Stevens-Johnson syndrome if it would prevent one PJP.

I remained unconvinced that steroids alone are worth the toxicities of tmp/sulfa. If there is good data, I can't find it. A few case reports do not impress. Add an immunosuppressive comorbidity? Sure. But if I were on high-dose steroids (and I have been), I would not take tmp/sulfa.

Rationalization

Prophylactic effect of trimethoprim-sulfamethoxazole for pneumocystis pneumonia in patients with rheumatic diseases exposed to prolonged high-dose glucocorticoids.

http://ard.bmj.com/content/early/2017/11/01/annrheumdis-2017-211796

J Cutan Med Surg. 2009 Nov-Dec;13(6):308-12. Fact or fiction: does the non-HIV/AIDS immunosuppressed patient need Pneumocystis jiroveci pneumonia prophylaxis? An updated literature review.

https://www.ncbi.nlm.nih.gov/pubmed/19919808

Prophylaxis for Pneumocystis jiroveci pneumonia: is it a necessity in pulmonary patients on high-dose, chronic corticosteroid therapy without AIDS?

https://doi.org/10.1586/17476348.2015.1002471

Mock

Apr 9, 2018

Well, as you know, there are many things in life that are not fair... Jimmy Carter

It is only fair to mention that the following entry has no content. I feel like whining, so whine I will. It is my blog and, to quote Cartman, whateva, I do what I want.

By a conservative estimate, I do about 450 hours of CME a year writing ID blogs, doing ID podcasts, and updating my ID app. I bet no one on the planet does as much patient-oriented, documented ID CME or produces more ID content. I do it because it keeps me up to date, and I do find ID endlessly interesting. I don't expect a Nobel or anything, but I thought I might get some CME.

So I write the AMA outlining my ongoing CME. Nope. Doesn't quality. I have to spend time and money doing accredited CME.

And by the way, don't make the mistake of emailing the AMA. They will send you endless emails to join. And have you ever noticed how the AMA sends letters that, unless you look at them carefully, look like renewal letters, I assume in an attempt to trick your office manager into sending them money. Typical. No AMA, you lost me years ago with Sunbeam, and I have yet to see any improvement. They are, like Congress, evidence that the worst of us go into politics.

OK. I have to be Board certified as part of my employment contract. Maybe I can get some MOC points. I sent the ABIM the same letter. Nope. Doesn't quality. I have to spend time and money doing an accredited MOC.

Crap.

While I have done a tonne of CME/MOC at Medscape, I need more. So I did the MOC 2017-2018 Update in Infectious Diseases. Two things in life are finite that you can't make more of: time and oceanfront property. Unless you are Lex Luthor.

I will never get the time back I wasted on the ABIM questions.

I know I learned nothing because when I do CME, I also open up Dreamweaver and add anything of note I learn to the app. 30 questions, nothing added.

And these questions are annoying. These long, pointless 'cases' that most serve to bore. Why include the O2 sat? Who cares? Why is it relevant? Why not just ask me the damn question?

One question, in particular, summed it up. A long case presentation, the only thing important was the smear at the end. And even that wasn't important for understanding the question. I always read the question first, and from the list of drugs as answers, I know it was going to be a question about Disease X. It was. In this case, how to treat Disease X in pregnancy. I live in the Great Pacific NW. I know about Disease X even though I have never seen a case and likely never will, much less a pregnant case.

So I put Disease X and pregnancy into google, and the first hit was a case report in the NEJM. A case report. A single case report. I am being pimped on a single case report of a disease I will never treat or see, and if I did, I would look it up.

You just know that one of the NEJM report authors submitted the question, and when they are on service, and Disease X comes up, they always, just always, somehow mention how to treat Disease X in pregnancy, just like their NEJM report. And I would too. If I got to play Augusta, I would brag as well. I just wouldn't make others look up my score.

And there are always a couple of questions that likely have the same origin story, submitted by some academic who has to offer up for my pain their wonky little case report. It might be interesting, but is it something I really have to pimped on for bored recertification?

There were a number of questions like that: of no applicability whatsoever to ID as I practice it, just some little factoid wrapped in a tedious and pointless 'case.' Why not just ask: The treatment of Disease X in pregnancy is? if it is so damn important.

If you were to draw a Venn diagram of my practice and new, applicable knowledge from the ABIM ID review, there would be no overlap. Lots of things I had to google, but little I had to know.

In the end, unlike my day-to-day patient-based CME for which I get no credit, I just got 10 hours for having good google skills. What a waste of time, time I will never get back.

I'll get the rest of my needed CME at ID Week, although I know from painful experience not to do the MOC classes. Not only boring, as they read you the questions (and what is ID Week but 5 days of people reading you Powerpoint slides so you can get CME?), and they have always gone over 30-45 minutes, screwing up the lunchtime. Dude. I'm in New Orleans. End on time so that I can get some jambalaya and beignets. Academics never respect others' time.

It really should be called the Mock, short for mockery, what they make of CME.

Me bitter much? Good thing I am taking a long weekend.

Bad Day

Apr 18, 2018

I cashed in some frequent flier miles and spent a long weekend in Coronado. The flight home had a few issues, and the 2-hour flight ended up taking 6 hours. No big from my perspective, just a little dull. But the way some people acted, you would have thought they were being told they had cancer of the puppy. I wonder how these folks would react to real stress, like an engine blowing up. People become asses and unpleasant for the pettiest of reasons.

I have long wished that we could stress test all hires like house staff to see how they react when it really hits the fan. Because a bad day in medicine is a really bad day for someone.

The patient, a homeless, cachectic, peripatetic African American, is admitted with shortness of breath and fevers. CXR shows diffuse interstitial infiltrates. He is placed on CAP therapy and, over the next 3 days, goes into progressive pulmonary failure and intubation and pancytopenia with no good diagnosis. They call me, and I did not have much to offer. The chest CT looked like a liver. The best I could say is that it looked like a rapidly progressive boop or interstitial cancer; I could not think of an infectious disease that acts like this.

Then the lab called: mold with septate hyphae. In the blood. I don't get that call very often. They could not tell me based on the blood cultures which mold it looked like. He was placed on lipid Amphotericin but died that night.

A week later, the lab identified it as Coccidioides immitis in both the blood and a bone marrow for cytopenia.

When I think Coccidioides, I think coccidioidal spherules. The hyphal forms are not usually seen in humans.

C. immitis proliferates in tissue, mainly as the parasitic spherule form. The saprobic mycelia, the soil form of C. immitis, are rarely identified in human tissue...A review by Meyer et al. in 1982 found 750 cases of C. immitis infection [4]. In only 12% (95) was mycelial formation noted...

As one case report suggests of CSF

The presence of hyphae, with or without formation of mature arthroconidia, in a leptomeningeal granuloma of coccidioidomycosis is distinctly unusual. Likewise, the presence of coccidioidal mycelia in CSF is unusual. Encountering C. immitis as mycelia in tissues and body fluids, suggesting mold infection, could confound the clinician.

Certainly did me, as I wondered about all sorts of septate hyphal fungi other than Coccidioides in my note: Aspergillus, Basidobolomycosis, and Emergemyces, to name a few as he had recently come to Oregon by way of the American SW.

Looking back in the literature on Coccidioides fungemia, they did all sorts of subcultures and techniques to isolate the organism. Still, none mention what is seen on direct stains from the positive blood culture bottle. I now know the answer, but I can't find it in the papers I could access.

He was HIV negative, but as noted, African American and malnourished as risks for fungemia. I had never seen rapidly progressive disseminated Coccidioides before. A positive blood culture has a high mortality, as noted here.

Rationalization

Medicine (Baltimore). 1986 Sep;65(5):312-21.Fungemia due to Coccidioides immitis. An analysis of 16 episodes in 15 patients and a review of the literature.

https://www.ncbi.nlm.nih.gov/pubmed/3528738

Hyphal Forms in the Central Nervous System of Patients with Coccidioidomycosis.

https://academic.oup.com/cid/article/30/2/349/381073

Goldberg

Apr 23, 2018

I did my Internal Medicine residency in a county hospital where we often did the blood draws, started IVs, did the ABGs, etc. Old school. Stupid school. I do not think patients benefited from an endless series of newbies trying to access a vein or an artery, although it did instill in me a deep appreciation of arm veins. To this day, the first thing notice in another human is the veins of their arms: are they an easy stick or not? Medical training instills all sorts of curious quirks.

I remember when of my fellow interns (it was NOT me) couldn't get a blood draw late one night on an IVDA who had used all her arm and leg veins. Finally, in disgust, the patient grabbed the syringe from him and stuck her own femoral vein, which she was quite adept at, and filled the syringe. He made the mistake of mentioning it and was reprimanded, but I thought it was a great idea.

Femoral vein users have been relatively rare in my practice, and

The main reason given for starting to inject in the groin was that 'no other sites were left'. However further discussion identified this meant no other convenient sites were accessible.

Although in one series, half of IVDA did so. And, of course, there is a how-to on the web. I suspect most IVDA learn from a friend, not the web, although I remain amazed at the number of homeless who appear to have the wherewithal to maintain a cellphone or tablet but little else. Certainly, garbage removal is not on the list here in PDX.

The patient is a long-standing IVDA admitted with progressive pain in the groin and fevers after injecting heroin. A CT showed a phlegmon around the femoral vein and a clot within the vein. Eventually, he ended up in the OR for ligation and removal.

The cultures were remarkable: Finegoldia (Peptostreptococcus magnus) magna, Actinomyces odontolyticus, Enterococcus faecalis, Peptoniphilus (Peptostreptococcus) asaccharolyticus.

He is homeless, so bathing is infrequent, as is dental care, given his advanced meth teeth. He is also a needle licker, so whether these organisms came from the mouth, the groin, or both is unknowable.

Septic thrombophlebitis is not uncommon in femoral vein injectors; it is usually Staphylococci and Streptococci. In one series, there was a single case of mixed anaerobes:

The organisms isolated were: Staphylococcus aureus (4); S aureus and group B streptococcus (2); S aureus and group G streptococcus (1); S aureus and Streptococcus intermedius (1); and Streptococcus pyogenes (1). Other associated conditions included groin abscess on white cell and ultrasound scanning (9), soft tissue infection of the groin, thigh, and lower abdomen (4) and upper limb (2), S aureus osteomyelitis (1), and pulmonary embolism (1). Pus from four abscesses grew S aureus; one also contained mixed anaerobes while five were sterile

I treated it as a variation of Lemierre’s syndrome. It is kind of a septic thrombophlebitis from mouth flora, even if in the groin. ID beats Bach for variations every time.

Up to one third of patients will have a polymicrobial bacteremia, with anaerobic streptococci and other miscellaneous gram-negative anaerobes frequently present

Just no Fusobacterium.

Antibiotics and anticoagulation, and he got all better.

Rationalization

Harm Reduct J. 2005; 2: 6. Published online 2005 Apr 15. doi: 10.1186/1477-7517-2-6 Use of the femoral vein ('groin injecting') by a sample of needle exchange clients in Bristol,

UK.

https://www.ncbi.nlm.nih.gov/pmc/articles/PMC1090606/

Going into the groin: Injection into the femoral vein among people who inject drugs in three urban areas of England.

https://www.sciencedirect.com/science/article/pii/S0376871615001854

FEMORAL INJECTING A GUIDE TO INJECTING IN THE GROIN USING THE FEMORAL VEIN

http://www.aivl.org.au/wp-content/uploads/Femoral-Injecting-Resource.pdf

High prevalence of iliofemoral venous thrombosis with severe groin infection among injecting drug users in North East Scotland: successful use of low molecular weight heparin with antibiotics

http://pmj.bmj.com/content/postgradmedj/76/899/561.full.pdf

Int J Emerg Med. 2013; 6: 40. Published online 2013 Oct 23. doi: 10.1186/1865-1380-6-40 Lemierre’s syndrome.

https://www.ncbi.nlm.nih.gov/pmc/articles/PMC4015694/

Injecting drug use is a risk factor for deep vein thrombosis in women in Glasgow.

https://onlinelibrary.wiley.com/doi/full/10.1046/j.1365-2141.2001.02633.x

Two zero 4

Apr 25, 2018

What is someone in their late 20's? Neither young nor middle-aged. Certainly not a kid, although everyone under the age of 40 seems a kid to me. Anyway, she has a history of recurrent infections.

As a child, sino-pulmonary. As a teenager, bronchitis. As a young adult, a refractory breast infection. And now recurrent leg abscess that are quite impressive. One had black pus come out when I eyeanddeed it. Gram stains have always had mixed skin flora and have never grown a pathogen: not bacteria, not fungi, not AFB.

I checked immunoglobulins looking for a CVIDS, and everything was low end of normal except the IgG4, which was non-existent.

That's two. Regular readers (as opposed to the constipated ones) will remember a case of Klebsiella liver abscess in a patient found to be IgG4 deficient. You don't remember? You and me both. It is curious how often I Google an uncommon disease, and one of my forgotten blog entries is a hit.

Given the history, the low IgG4 is probably the reason she has recurrent infections.

Isolated deficiency of IgG4 is associated with symptoms and signs which are similar to those associated with other IgG subclass deficiencies. The most common manifestation is an unusual frequency of pyogenic infections, most often of the respiratory tract.

But there is not a lot on the PubMeds. I have checked a tonne of IgG subtypes over the years in patients with recurrent infections but never saw an IgG4 deficiency until this year. Since events do come in three, I will see another one soon.

I suspect that there is more to her immune function than just an IgG4 deficiency, but my ability to evaluate the immune system is at the level of stone knives and bearskins.

So I will see what monthly IVIG will do for these infections.

Postscript

They improved.

Rationalization

N Engl Reg Allergy Proc. 1988 Jan-Feb;9(1):43-50. IgG4 immunodeficiency.

https://www.ncbi.nlm.nih.gov/pubmed/3362107

But Wait. There's More.

http://boards.medscape.com/forums/forums/?128@@.2a82aebd!comment=1

More Water

Apr 30, 2018

The patient has a TAVR placed. It was a wee bit complicated, and her femoral artery required a bovine patch. She did well initially but then developed increasing pain and a fever. The groin was explored, and a hematoma was found. The gram stain had gram-negative rods.

They called me.

Talking to the patient, I found she has well water, only filtered for sand and grit.

So I bet it would be a water organism from the well water. And it grew P. mirabilis.

Proteus spp. bacteria have been isolated from different human and non-human environments and their presence in higher organisms, soil, and water is well documented

such as

... in bore well waters in Mysore City, P. mirabilis and P. vulgaris strains dominated over the other H2S-producing strains (considered as associated to fecal coliforms in drinking water) . Poonia et al. reported on the alarming presence of multidrug resistant P. mirabilis and P. vulgaris strains among other bacteria in drinking water from springs and streams in the rural areas of Sikkim. Also, in Nigeria, Proteus spp. bacteria were detected in two of five studied well waters, treated as a source of drinking water.

Searching for 'Proteus' and 'well water' finds many a hit in the developing world. And, I suspect, wells in the rural Great Pacific NW.

I do not know if you have had surgery, but I have, and that first shower is unbelievably nice, washing off the sweat and grunge and blood and grease that rapidly builds up. The only time a shower feels better is after a 5-day hiking trip. You want to stand in the hot water until the water heater runs dry. So nice.

So it is likely a perfect storm: well water, a fresh wound, an extra long shower, and often a poor host. She is a diabetic.

This makes the fourth postop well water-associated infection I have written about, and I have an E. coli c-spine infection I can recall, but it looks like I never wrote up.

Currently, we are working on screening patients for their water source and education for post-op care. As one review notes

Although the evidence is limited one trial has suggested that the use of tap water to cleanse acute wounds reduces the infection rate and other trials conclude that there is no difference in the infection and healing rates between wounds that were not cleansed and those cleansed with tap water and other solutions. However the quality of the tap water should be considered prior to its use and in the absence of potable tap water, boiled and cooled water as well as distilled water can be used as wound cleansing agents.

Rationalization

UnWell.

http://boards.medscape.com/forums/?128@@.2a8165d9!comment=1

Water you doing?.

http://boards.medscape.com/forums/?128@@.2a82d61f!comment=1

Ugg.

http://boards.medscape.com/forums/?128@@.2a31442c!comment=1

Microb Ecol. 2016; 72(4): 741–758.cPublished online 2016 Jan 9. doi: 10.1007/s00248-015-0720-6 Significance and Roles of Proteus spp. Bacteria in Natural Environments

https://www.ncbi.nlm.nih.gov/pmc/articles/PMC5080321/

Cochrane Database Syst Rev. 2002;(4):CD003861. Water for wound cleansing.

https://www.ncbi.nlm.nih.gov/pubmed/12519612

Looking Is Not Seeing

May 2, 2018

I have to admit. One of the job's real joys is to make a non-infectious disease diagnosis that all my colleagues have missed. I am that petty. Last month I did it twice with the same diagnosis.

The first was an implant infection where the wound dehisced for no obvious reason. What was striking on exam was the soft robins egg blue sclera.

So I started asking her questions. She had hypermobile joints, and as a high school cheerleader, she could bend over backward and put her hands on the floor.

The second was a young female who started getting DJD of the back in her teens, surgery for a herniated disk in her early twenties, and now has what looks to be an infected herniated disk. What is striking on exam as she leaned forward during the interview just how hyperextended her elbows were. Upon direct questioning, she, too, had had teen years with hypermobile cheerleading.

Both could easily touch their thumb to their forearm and hyperextend their fingers.

I think both have Ehlers_x0013_Danlos syndrome on one sort or another.

In the first case, I suspect Ehlers_x0013_Danlos syndrome played a role in wound dehiscence/poor healing as

patients with EDS pose significant healing problems.

In the second case, perhaps it was why such a young human had such advanced spine disease at such a young age and with zero risks:

...genetic disorders that cause or contribute to radiculopathies. These genetic disorders include neurofibromatosis, Paget's disease of bone, and ankylosing spondylitis. Numerous genetic disorders can also lead to deformities of the spine, including spinal muscular atrophy, Friedreich's ataxia, Charcot-Marie-Tooth disease, familial dysautonomia, idiopathic torsional dystonia, Marfan's syndrome, and Ehlers-Danlos syndrome

and

Discopathy and early degenerative spondylotic disease manifest by spinal segmental instability and kyphosis, rendering EDS patients prone to mechanical pain, and myelopathy.

I think the infection was bad luck: I have found the information on secondary infection of herniated discs with (mostly) P. acnes to be reasonably convincing. She was evaluated and treated accordingly.

Both had been seeing doctors for years, and I am sure everyone looked at the joints and eyes. No one, however, saw what they represented.

So I get to gloat, and gloat I will.

Rationalization

J Wound Care. 2012 May;21(5):223-6. Some wounds are hard to heal: an interesting presentation of Ehlers-Danlos syndrome.

https://www.ncbi.nlm.nih.gov/pubmed/22584739

Am Surg. 2013 Jun;79(6):608-13. Surgical failures: is it the surgeon or the patient? The all too often missed diagnosis of Ehlers-Danlos syndrome.

https://pubmed.ncbi.nlm.nih.gov/23711271/

Semin Neurol. 2006 Nov;26(5):515-22. Genetic disorders producing compressive radiculopathy.

https://pubmed.ncbi.nlm.nih.gov/17048153/

Am J Med Genet C Semin Med Genet. 2017 Mar;175(1):195-211. doi: 10.1002/ajmg.c.31549. Epub 2017 Feb 21. Neurological and spinal manifestations of the Ehlers-Danlos syndromes.

https://www.ncbi.nlm.nih.gov/pubmed/28220607

Eur Spine J. 2013 Apr;22(4):690-6. doi: 10.1007/s00586-013-2674-z. Epub 2013 Feb 10. Does nuclear tissue infected with bacteria following disc herniations lead to Modic changes in the adjacent vertebrae?

https://www.ncbi.nlm.nih.gov/pubmed/?term=23397187

POLL RESULTS

I prefer to

  • look but not see 5%
  • inhale but not smell 26%
  • eat but not taste 11%
  • listen but not hear 47%
  • touch but not feel 0%
  • Other Answers 11%
  • chew 22 times, till it tastes like library paste.

Fluent

May 7, 2018

Striving to better, oft we mar what's well

King Lear

I have a secret to tell: ID isn't really all that hard. Like a language, it just takes time and practice to learn. You have to learn the vocabulary, the bugs, of which there are only 1200-1500. And the grammar, the pathophysiology. Over time, the language becomes internalized, so I don't have to think, what is the word for steak, what is the word for cook, what is the word for medium. You know it. It is a weird process, how my approach to ID has gone from conscious contemplation of a case to most of the time, I just know what to do.

Then you start to acquire the slang, the idioms, and the humor, and, after a couple of decades, you are fluent in the language. No one ever masters a language, as there are always new words, new idioms, new slang, and archaic usage that pops up in reading and conversation. This blog is a testament to either my denseness or the ongoing evolution of the ID language. I'll let you choose which.

I say this because I have noticed of late how little time it takes to get to the essence of most consults. 10-15 minutes tops. You have to make sure you dot your t's and cross your i's, but really, does a Group A Streptococcus lung abscess need the usual FUO exposure history? Nope. I had to smile when I read

...the infectious disease doctors who asked me about my sex life, how often I got high, my last period, whether I’d been anywhere near livestock.

It is what we do, after all. I would have as well, but because I realize the importance of habit in taking a complete history and have been burned by the occasional surprise. There is the occasional FUO or odd case that is like having to read Canterbury Tales in the original English. Is that even English? But they are few and far between.

I have noticed there is a tendency for ID docs to brag? whinge? complain? about how much time they spend doing a consult. I think it is because they are not yet fluent in ID. If you are on service 4 months a year and get the information up and down the hierarchy of student to resident to fellow to attending, you never really become fluent in ID. It is like learning and speaking all your French in college classes. Compared that to the immersion of private practice for 30 plus years. That's how you get fluent.

Do I know how to win friends and influence people or what?

I mention it as I came across this:

At the task-level, spending more time on something doesn’t always result in it being better. We know that longer, more complicated emails are less likely to be read and spending the time to provide more strategic options/choices generally leads to poorer decisions.

When this belief — that more time leads to better quality — is the unwritten rule, there is always more to do. As a result, work will fill the time it is permitted to fill, killing the hope that increased productivity will yield a better work-life balance. One participant in our program said, “Why should I get my work done more quickly if I’ll just be given more work to fill the time?” To break up this cycle, we need to stop associating more time with higher quality work.

As evidently

80% of the value comes from 20% of the work.

ID with its damn near endless rounds I remember is 5% value from 95% work. Pas moi dude.

The case today? The patient had a pneumonia at another hospital three months ago. Look at CXR and cultures. Left lobar consolidation, Pneumococcus, and Aspergillus.

Now has hemoptysis. Three months of weight loss. No other symptoms.

CT and CXR: big cavity filled with a mass the size of my fist (I have Trumpian hands) in the left lobe that looks like an Aspergilloma. Cultures now with only Aspergillus.

Well-controlled HIV for many years, but the CD4's hover just over 200.

Terribly cachectic, low protein. Horrible surgical risk.

Voriconazole and nutrition started, levels, and susceptibility pending.

Not terribly time-consuming, really. 15 minutes tops for the essence of how I describe my job: me find bug, me kill bug, me go home. Of course, the consult itself took more time: talking with the patient and the team, writing and dictating the note, etc. I bet I haven't had a dictation ever run more than a page and a half. We get paid for content, not verbosity, and no one wants a long note.

The fun part was trying to figure out why he had Aspergillus pneumonia. It is a ubiquitous mold, and he is a bad host, and occasionally fungus will follow other infections in poor hosts. But I wonder if it is the marijuana use, of which he partook daily until the pneumonia.

Where does Aspergillus (and other molds) thrive? Rotting organic material, which describes marijuana nicely. There are a smattering of cases of Aspergillus associated with smoking marijuana. I suspect we might be seeing more pulmonary molds with legalization and perhaps increased use in cancer and other at-risk populations.

No good deed goes unpunished after all.

Rationalization

How to Not Die in America

https://splinternews.com/how-to-not-die-in-america-1822555151?rev=1517341400470

How to Not Die in America.

https://splinternews.com/how-to-not-die-in-america-1822555151

The Lie That Perfectionists Tell Themselves

https://hbr.org/2018/05/the-lie-that-perfectionists-tell-themselves

J Clin Oncol. 2008 May 1;26(13):2214-5. doi: 10.1200/JCO.2007.15.2777. Invasive pulmonary aspergillosis associated with marijuana use in a man with colorectal cancer.

https://www.ncbi.nlm.nih.gov/pubmed/18445848

Marijuana use and aspergillosis

https://www.aspergillus.org.uk/content/marijuana-use-and-aspergillosis

Medical marijuana: Fungal, bacterial contaminants found, dangerous for immunocompromised patients.

http://outbreaknewstoday.com/medical-marijuana-fungal-bacterial-contaminants-found-dangerous-immunocompromised-patients-67341/

POLL RESULTS

I think

  • the longer I spend on a case, the better job I do 8%
  • brevity is the soul of wit 20%
  • short and sweet should apply to more than my sex life 24%
  • le mieux est l'ennemi du bien or Il meglio è nemico del bene. 28%
  • Give them the third best to go on with; the second best comes too late, the best never comes. 16%

Not the Pixar Movie

May 9, 2018

My old partner used to refer to the Whipple procedure as an ID employment act, and I think that is a reasonable description.

The first infection was S. infantarius bacteremia immediately following the procedure. No good source was noted on CT, just a lot of liver cysts, and she responded to a course of antibiotics.

Two months later, she was readmitted with fevers, rigors, and RUQ tenderness. CT this time showed one of her previous liver cysts was now much larger and denser. No surprise:

Pyogenic liver abscess occurred in 2.6% (22/839) of patients following PD, with 13 patients (59.1%) having a solitary abscess and 9 (40.9%) multiple abscesses

As best I can tell from the PubMeds, liver cysts are infected about as often as polycystic kidney cysts, which surprises me as the liver is downstream from the bowel and gets all the effluvium of the gi tract. The kidney no so much.

We identified 70 articles documenting a total of 215 cyst infection cases (renal n = 119; hepatic n = 96). Six studies, including 74 cases of renal and 61 cases of hepatic cyst infection

Like renal cysts, the second-best way to see it is infected is a PET scan. We did the best way, a drain.

There was pus, and the cultures grew Raoultella (Klebsiella) ornithinolytica.

That is unusual, with maybe a couple hundred or so cases in the literature from this organism causing a smattering of infections. And my first. This is the world's first reported liver abscess, although there are cases of biliary disease. Go me. Most are nosocomial, this one? Only one so far in my hospitals.

Where is R. ornithinolytica found?

R. ornithinolytica has been isolated from the gut of fish, ticks, and termites and from estuarine water, and it has been shown to produce histamine, contributing to fish poisoning...

She was exposed to none of the above.

The one review suggests R. ornithinolytica is often resistant:

The proportion of R. ornithinolytica isolates resistant to antibiotics was found to be relatively high: 4% of isolates were resistant to ceftriaxone, 6% to quinolones, and 13% to co-trimoxazole.

although this was pan-sensitive. So I called upon

the god of the wild, shepherds and flocks, nature of mountain wilds, rustic music and impromptus, and companion of the nymphs.

for help. And she got all better.

Rationalization

Nephrol Dial Transplant. 2015 May;30(5):744-51. doi: 10.1093/ndt/gfu227. Epub 2014 Jun 20. Diagnostic criteria in renal and hepatic cyst infection.

https://www.ncbi.nlm.nih.gov/pubmed/24950937

Int J Infect Dis. 2016 Apr;45:65-71. doi: 10.1016/j.ijid.2016.02.014. Epub 2016 Feb 24. Emerging role of Raoultella ornithinolytica in human infections: a series of cases and review of the literature.

https://www.ncbi.nlm.nih.gov/pubmed/26921549

J Gastrointest Surg. 2014 May;18(5):922-8. doi: 10.1007/s11605-014-2466-8. Epub 2014 Feb 8. Pyogenic liver abscess following pancreaticoduodenectomy: risk factors, treatment, and long-term outcome.

https://www.ncbi.nlm.nih.gov/pubmed/24510300

Other reports? Czech.

May 14, 2018

The patient was previously healthy when she was admitted for groin pain, fevers, and erythema. She mentioned she might have had a pimple in the region, but nothing else of note.

In the ER, she was febrile, hypotensive, (newly) hyperglycemic, and a CT showed diffuse groin edema but no abscess. She was whisked off the OR, where a Fournier's was diagnosed and debrided.

She rapidly improved after debridement, and they called me the next day for antibiotic choice.

Gram stain had the usual hodgepodge of gnb and gpc, but the cultures grew

Moderate growth Actinomyces species, heavy growth Prevotella species & heavy growth Peptostreptococcus anaerobius.

Actinomyces? Huh?

I called the lab for further information. There is A. israelii, of sulfa granule and lumpy jaw fame, and then there are the 30 plus other species found in the skin, gut, mouth, and GU tract of humans and animals. And this one?

Actinomyces turicensis by MALDI-TOF.

Never heard of it. But a quick search shows soft tissue infections are what this organism does:

The first five patients, all immunocompetent young people, as well as patient 6, with insulin-dependent diabetes mellitus, suffered from infected pilonidal sinus and perianal abscesses. In two cases, A. turicensis was the only possible pathogen isolated from the wound under aerobic and anaerobic conditions. In the other three cases, some concomitant flora represented by anaerobic pathogens

And of the non-israelii Actinomyces, Actinomyces turicensis may be the most common pathogenic strain:

The majority of skin-related infections caused by Actinomyces below the waistline are due to A. turicensis

Although I can find two other cases of Fournier's with Actinomyces, neither turicensis, I suspect in the era of the MALDI-TOF that lack will be remedied.

Treatment? I&D. But it is likely susceptible to most antibiotics

Susceptibility testing of anaerobes is seldom performed in clinical microbiology laboratories. In general, this may not be an important issue with Actinomyces species, since they are rarely, if ever, resistant to beta-lactam agents

and she did just fine, more from the surgeon than me.

Rationalization

Seven Isolates of Actinomyces turicensis from Patients with Surgical Infections of the Anogenital Area in a Czech Hospital.

https://www.ncbi.nlm.nih.gov/pmc/articles/PMC2897515/

Clinical Spectrum of Infections Due to the Newly Described Actinomyces Species A. turicensis,A. radingae, and A. europaeus.

http://jcm.asm.org/content/37/1/8.abstract

Clin Microbiol Rev. 2015 Apr;28(2):419-42. doi: 10.1128/CMR.00100-14. Actinomyces and related organisms in human infections.

https://www.ncbi.nlm.nih.gov/pubmed/25788515

Never Before Reported

May 16, 2018

The patient is a young man, no health issues who has his medial meniscus repaired. About a week later, the knee becomes red, hot, and tender. A tap shows 76,000 WBC, no organisms, but there are calcium pyrophosphate crystals. Pseudogout it is.

Pseudogout happens.

Acute onset of pain following an ACL reconstruction can raise the possibility of infective arthritis. In our case, the knee was washed out and the synovial fluid was positive for pseudogout crystals…Several causes of CPPD deposition have been proposed, with its occurrence being classified as either primary (non-traumatic) or secondary (traumatic). Primary deposits are thought to be due to a metabolic cause, such as hyperparathyroidism and haemochromatosis where crystallization occurs within the synovial fluid. Secondary deposits are more likely to occur in the presence of degenerative knee changes as a result of trauma and surgery. It has been proposed that the mechanism for this is either that the crystals are liberated or ‘shed’ from preformed deposits within the joint or alternatively that an intra-articular insult may trigger their formation.

Cultures were negative, so he was treated for pseudogout and did not get better. The knee continued to be swollen and painful, and the CRP skyrocketed.

It was retapped. Same WBC, same crystals, same gram stain. It looked like refractory pseudogout, but this time on day five, all the cultures grew Cutibacterium acnes, the organism formally known as Propionibacterium acnes. Pronounced cooty, not cutie, BTW.

Kind of odd.

There are no reported cases of C. acnes infecting either gout or pseudogout. C. acnes does infect native joints, albeit rarely and, as in this case, after orthopedic trauma:

A literature search for Propionibacterium septic arthritis was performed. Clinical course, treatment, and outcome are reviewed for all cases. Our three cases were combined with 15 cases from the literature. Fourteen cases showed few signs of acute infection, slow culture growth, and delayed diagnosis. In 3 cases an early culture was dismissed as a contaminant. Six cases were reported as caused by recent arthrocentesis.

And I was surprised by the increase in CRP, although that may have been due to the pseudogout.

Now the question is how long to treat. C. acnes tends to be indolent when associated with prosthetic material, and I suppose the crystals are the moral equivalent of prosthetic material. So we will see.

There is always the question with crystalline arthropathies whether or not there is a secondary infection or if infection can precipitate crystalline arthropathies. Pun very much intended. I have fretted about it in the past and do not have a clear answer. But, it is why we do cultures.

Rationalization

Pseudogout: A Rare Cause of Acute Arthritis Following Arthroscopic Anterior Cruciate Ligament Reconstruction.

https://www.ncbi.nlm.nih.gov/pmc/articles/PMC4570956/

Infect Dis Rep. 2017 Oct 2; 9(3): 7185. Published online 2017 Oct 2. doi: 10.4081/idr.2017.7185 Native join Propionibacterium septic arthritis.

https://www.ncbi.nlm.nih.gov/pmc/articles/PMC5641666/

International Journal of Clinical Medicine, 2013, 4, 13-19 Differences in Acute Phase Reactants between Gout and Pseudogout.

http://dx.doi.org/10.4236/ijcm.2013.412A2003

Chicken and Egg.

http://boards.medscape.com/forums/[email protected]@.2a7b8557!comment=1

Veni, vidi, vici.

http://boards.medscape.com/forums/[email protected]@.2a8292e6!comment=1

Podagra Plus.

http://boards.medscape.com/forums/[email protected]@.2a332c5b!comment=1

I Thought It Was a Classic Cause of Fever

May 31, 2018

Back from 10 days off. I have said this before, but do not go away on vacation. The first few days back are almost guaranteed to make the time off not worth it. Almost. I got to see my eldest graduate from Boston College Law, so one down, one to go.

But back to a life of fevers.

The patient has a classic FUO: 3 months of fevers, chills, sweats, malaise, weight loss, and an unimpressive outpatient workup.

I see the patient, and she has some risks for cat scratch and histoplasmosis, but evaluation for these infections is negative.

So the next step in the work-up is the donut of truth, as the house staff call it: CT of chest/abdomen/pelvis, and much to my surprise, there is a mass in the kidney.

In patients with FUOs and a renal mass, the differential diagnosis includes renal tuberculosis, renal cell carcinoma (hypernephroma), renal malakoplakia, and xanthogranulomatous pyelonephritis.

I think of two solid tumors as a reason for FUO: pancreatic and renal cell, as I have seen one of each. This does not seem to be the case in the literature.

Of 35 patients (72.9%) with haematological malignancy, most were either lymphoma (n=18) or malignant histiocytosis (n=13). Solid tumours were found in 13 patients (27.1%), of which seven were hepatic carcinoma.

and

Other malignancies presenting more than occasionally as FUO’s are colon cancer and large hepatic metastases or hepatocarcinomas.

but as they note

It should be stressed that many different tumors have been reported as having an initial presentation as FUO.

And this case was a urothelial cell carcinoma, of which there is one other in the Pubmeds as a cause of fever.

So I have doubled the reportable cases in the world, as

With the widespread use of CT, FUO due to deep-seated abscess or solid tumor is decreasing markedly.

I guess there was a reason to back to work after all.

Rationalization

Int J Clin Pract. 2003 Jul-Aug;57(6):508-12. Malignancy in Chinese adults presenting as fever of unknown origin.

https://www.ncbi.nlm.nih.gov/pubmed/12918890

Fever of unknown origin in cancer patients. DOI:

https://doi.org/10.1016/j.critrevonc.2016.02.015

Heart Lung. 2012 Nov-Dec;41(6):606-9. doi: 10.1016/j.hrtlng.2012.03.008. Epub 2012 May 31. Fever of unknown origin (FUO) and a renal mass: renal cell carcinoma, renal tuberculosis, renal malakoplakia, or xanthogranulomatous pyelonephritis?

https://www.ncbi.nlm.nih.gov/pubmed/22658892

Intern Emerg Med. 2012 Sep;7 Suppl 2:S117-8. doi: 10.1007/s11739-012-0783-z. Epub 2012 May 1. Upper tract urothelial cell carcinoma presenting as fever of unknown origin and acid-sterile pyuria.

https://www.ncbi.nlm.nih.gov/pubmed/22547368

BMJ Open. 2013; 3(12): e003971. Published online 2013 Dec 19. doi: 10.1136/bmjopen-2013-003971 Diagnostic workup for fever of unknown origin: a multicenter collaborative retrospective study.

https://www.ncbi.nlm.nih.gov/pmc/articles/PMC3884594/

How to Avoid Mass

Jun 4, 2018

The rule is don't take a temperature unless you want to deal with a fever. But others order unneeded tests all the time, and in some cases, I get to deal with the results.

Just today. A patient had a syndrome (history, physical, labs) that was almost, but not quite, entirely unlike EBV. So EBV serologies were ordered, the orderer had no idea what the serologies meant, so the patient was sent to me.

I had no idea. What to do with a test result ordered when the patient has no signs or symptoms of the disease in question? I ignore it.

And ECHOs. Don't get me started on ECHOs. Well, too late. My concept of medicine is to order tests that have a reasonable pretest probability.

I wonder how many ECHOs get ordered on IVDAs with fevers long before any blood cultures are back. Or just for a fever. A total waste of time and money.

Over 70% of TTEs were obtained in patients in whom the diagnosis of IE was rejected by Von Reyn criteria. Therapeutic management (prolonged antibiotic administration) was associated significantly with Von Reyn categorization, and not significantly affected by TTE results.

Conclusions Most TTEs are obtained in patients with a low pretest probability of IE and do not contribute to therapeutic decision making.

and

Evidence of infective endocarditis was detected on echocardiography in 43 of the 500 patients (8.6%). In 239 patients (48%), vegetations and certain prespecified clinical criteria were both absent. These criteria were: vasculitic/embolic phenomena; the presence of central venous access; a recent history of injected drug use; presence of a prosthetic valve; and positive blood cultures. The collective absence of these five criteria indicated a zero probability of TTE showing evidence of endocarditis.

But I see TTEs order all the time for these 'indications' and less. Like a heroin user with an OD. He got an ECHO. And there was a mass on the tricuspid valve. And zero and symptoms of endocarditis. Now what? Fortunately, there is a study for almost everything. If you do TTE's on random IVDAs, you find

valvular abnormalities assessed by echocardiography were prevalent in asymptomatic IDUs without a medical history of IE, and vegetations were seen in 5% of subjects.

So I said, let's do nothing, especially as the patient is now in a controlled environment.

Months pass. No problems. Repeat ECHO: no change.

But it would have been better if the ECHO had never been ordered at all. No ID consult, no repeat ECHO, no angst about what to do.

Get ECHOs on patients who meet the criteria for having endocarditis.

And me? If the patient has the clinical criteria, why get the ECHO? It doesn't change what is done:

Although 9 in 10 TTEs were appropriate by 2011 AUC, fewer than 1 in 3 TTEs resulted in an active change in care, nearly half resulted in continuation of current care, and slightly more than 1 in 5 resulted in no change in care. The low rate of active change in care (31.8%) among TTEs mostly classified as appropriate (91.8%) highlights the need for a better method to optimize TTE.

Most of the time, I order ECHOs not to make the diagnosis but when I am looking for something surgical like ring abscess or valve destruction.

Rationalization

Overuse of Transthoracic Echocardiography in the Diagnosis of Native Valve Endocarditis. Arch Intern Med. 2002;162(15):1715-1720. doi:10.1001/archinte.162.15.1715

https://jamanetwork.com/journals/jamainternalmedicine/fullarticle/754054

Am J Cardiol. 2014 Jul 1;114(1):100-4. doi: 10.1016/j.amjcard.2014.04.010. Epub 2014 Apr 18. Echocardiographic findings suggestive of infective endocarditis in asymptomatic Danish injection drug users attending urban injection facilities.

https://www.ncbi.nlm.nih.gov/pubmed/24819896

Clinical criteria and the appropriate use of transthoracic echocardiography for the exclusion of infective endocarditis.

http://heart.bmj.com/content/89/3/273.short

Appropriate Use and Clinical Impact of Transthoracic Echocardiography JAMA Intern Med. 2013;173(17):1600-1607. doi:10.1001/jamainternmed.2013.8972.

https://jamanetwork.com/journals/jamainternalmedicine/fullarticle/1718444

Why We Have Radiologists

Jun 7, 2018

The patient has substernal sharp chest pain that worsens with inhalation and sitting up. When asked about the pain, she literally draws a line around her heart. She also has new onset of atrial fibrillation.

In the ER, she has a fever, leukocytosis, and an echo that shows mild pericardial effusion and thickening.

Pericarditis it is. Or is it?

The fever persists, and they consult me. I talk to the patient and don't learn a whole lot. She had been ill for about a week and a half with the fevers but was otherwise nonfocal.

I have no good diagnosis and go out to write my note. She had just had a CT, but the report wasn't dictated. I saw a big spleen, a big gallbladder with some stones, and what I thought was the stomach in a weird place, just to the right of the heart.

I said as much as I wrote the note:

"ct unimpressive to my eye,? big spleen"

Then the hospitalist sat down beside me and said that there is a big liver abscess. I said I thought it was stomach, and he laughed and said he thought the same. That is why we have radiologists. My note then continued:

"on second thought, I am informed that what I thought is a stomach is an abscess."

The abscess was in an odd place, more perihepatic than in the left lobe of the liver and abutting the pericardium. And he had no transaminitis.

Not a common cause of pericarditis and atrial fibrillation.

Back last century, when I was a fellow, we had an amoebic liver abscess rupture into the pericardium, and there are a smattering of cases of bacterial abscesses doing the same.

In this case, the abscess was drained (E.coli), and she got all better with a course the antibiotics. The pericardial effusion being sympathetic rather than purulent, resolved on its own.

Rationalization

Pericardial tamponade due to ruptured pyogenic hepatic abscess November 2007Br J Cardiol 2007;14:296

https://bjcardio.co.uk/2007/11/pericardial-tamponade-due-to-ruptured-pyogenic-hepatic-abscess/

Case Rep Med. 2014; 2014: 735478. Published online 2014 Apr 29. doi: 10.1155/2014/735478 Purulent Pericarditis after Liver Abscess: A Case Report

https://www.ncbi.nlm.nih.gov/pmc/articles/PMC4020547/

POLL RESULTS

I am glad we have

  • radiologists 13%
  • mixologists 22%
  • microbiologists 52%
  • zymologists 2%
  • blogologists 11%

An Old Cause of Fever

Jun 11, 2018

The patient was admitted to Outside Hospital for a minor orthopedic procedure. Post-op, she was febrile, and a work-up was negative and, after the fevers abated for 12 hours, was discharged home.

The fever immediately returned, and she was readmitted. Fevers persist, and work up is again negative. No surgical infection, no abnormal labs, no nothing. So they call me.

She states she was not feeling all that ill with the fever and had not been taking her temperature at home in the days leading up to the procedure but did feel a bit punk.

A usual exposure history found nothing of note.

But the curious thing was her fever curve. While her temperature waxed and waned, the lowest it ever got was 37. To my mind, that is suggestive of drug fever.

I classify drug fever as toxic, allergic, and those drugs that directly mess with temperature homeostasis, like antipsychotics and malignant hyperthermia. As I was taught years ago, one form of drug fever is more of a toxin/poison and not an allergic reaction to the drug. Since the drug is always in the blood, it always incites a febrile response, hence a continuous fever. Works for me.

I do not know if the explanation is true, and I can find no supportive papers in the Pubmeds. As best I can tell, a variety of processes, both infectious and non-infectious, have been reported to cause a continuous fever. But in my experience, a continuous fever suggests a drug fever.

Looking at the drug list, there was hydralazine, which she said had been increased a month ago.

Back in the day, they used a lot of hydralazine. I learned as an intern how important it is not to give an after-load reducer in severe aortic stenosis when a cardiologist gave IM hydralazine for severe hypertension in just such a patient. My second code as a resident. Successful, but it did make quite the impression, let me tell you.

And there is the lupus-like reaction to hydralazine, which I haven't seen since back in the day:

After three years' treatment with hydralazine the incidence of the lupus syndrome was 6.7% (95% confidence limits 3.2-10.2%). The incidence was dose dependent, with no cases recorded in patients taking 50 mg daily and incidences of 5.4% with 100 mg daily and of 10.4% with 200 mg daily. The incidence was higher in women (11.6%) than in men (2.8%). In women taking 200 mg daily the three year incidence was 19.4%.

She had no other symptoms to suggest SLE. But I remember a smattering of cases of presumptive hydralazine fever from my training days.

Early reactions to hydralazine usually occur between 10 and 20 days after the beginning of therapy and are also more common in slow acetylators. Fever is a common manifestation, either alone or together with arthralgia, myalgia, rash, or lymphadenopathy.

So we stopped the hydralazine, and in about 4 half-lives, the fever was gone. I wonder if the fever was due in part to the recent dose escalation.

Causal? True/true and unrelated? The only way to know is to re-challenge, which, of course, we did not do.

As one of our cardiologists likes to note when leading noon report, the top three causes for a patient presentation are drugs, drugs, drugs.

Rationalization

The lupus syndrome induced by hydralazine: a common complication with low dose treatment. Br Med J (Clin Res Ed). 1984 Aug 18; 289(6442): 410–412.

https://www.ncbi.nlm.nih.gov/pmc/articles/PMC1442447/

Rev Infect Dis. 1982 Jan-Feb;4(1):69-77. Drug-induced fever: cases seen in the evaluation of unexplained fever in a general hospital population.

https://www.ncbi.nlm.nih.gov/pubmed/7071457

Hot Tub

Jun 14, 2018

The patient has had a month or so of a cough and shortness of breath. She comes into the hospital, has a reticular nodular infiltrate on CT, gets a negative bronchoscopy, and gets better on a quinolone.

After stopping the quinolone, she relapses and gets re-admitted, and now, after two and a half weeks, the cultures are growing an AFB. Too soon for TB, I take a history looking for a reason for a rapidly growing AFB. Yes, 2.5 weeks to grow is rapid for an AFB.

This is Oregon, and she is on well water, untreated, that keeps her hot tub filled.

Bingo.

I bet it would be Mycobacterium avium-intracellulare (MAI), but for the short term, we had to assume MTb. But MAI it was.

There are around 40 hits on the Pubmeds with Mycobacterium avium-intracellulare and hot tubs, a cause of 'hot tub lung.'

Is it infectious? Allergic? Both? Most point to an allergic reaction

With avoidance of exposure, clinical and radiologic improvement was observed in all patients. Additionally, 13 patients (62%) received corticosteroid therapy, 1 (5%) antimycobacterial therapy, 2 (10%) received both, and 5 patients (24%) received no pharmacologic therapy.

and while most patients present with

Dyspnea and cough were present in all patients, hypoxemia was noted in 10 patients (48%).

Fevers, as in this patient, are lacking.

It is kind of hard not to treat a positive culture under these circumstances, but I have never been enthusiastic about treating MAI and

M. avium complex was isolated from the hot tub water, respiratory secretions and/or lung tissue in all patients.

So we will see what steroids and hot tub avoidance do.

She got all better.

Rationalization

Nontuberculous Mycobacterial Disease Following Hot Tub Exposure.

https://wwwnc.cdc.gov/eid/article/7/6/01-0623\_article

AJR Am J Roentgenol. 2007 Apr;188(4):1050-3. CT findings of granulomatous pneumonitis secondary to Mycobacterium avium-intracellulare inhalation: "hot tub lung".

https://www.ncbi.nlm.nih.gov/pubmed/17377045

Respir Med. 2006 Apr;100(4):610-5. Epub 2005 Sep 27. Hot tub lung: presenting features and clinical course of 21 patients.

https://www.ncbi.nlm.nih.gov/pubmed/16194601

What's Past is Prologue

Jun 20, 2018

The patient presents with fevers, headache, ataxia, and an altered sensorium. Work-up ensues, and it is discovered that there is mastoiditis complicated by a cerebellar abscess. Not so odd. As was noted in 1961

the subject at brain abscess arising from an infected mastoid is not in any sense a recent one, nor does it appear to be of any less importance to the otolaryngologist and it was nearly a century ago

That was from the pre-CT/MRI era, and the diagnosis was often delayed with fatal results.

... a case which is reported about 70 years ago was accompanied by the comment that "this is one of the few cases of cerebral abscess in connection with middle ear disease which is been accurately diagnosed during life and successfully treated by operation" but "it is perhaps more frequently the subject of the postmortem record."

A bad disease back in the day.

As far back as 1948, the connection between mastoids and cerebellar abscesses was noted. In

... eighteen cerebellar abscesses. With one probable exception they were all due to mastoid infection...

Although this century we get our cerebellar abscesses in the hipster crowd:

We describe a previously healthy adult who had a solitary cerebellar brain abscess diagnosed. This infection occurred 4 weeks after the patient underwent a tongue piercing procedure that was complicated by an apparent local infection. The clinical history, abscess culture results, and lack of an alternative explanation suggest that infection of the tongue piercing site was the source of the cerebellar abscess.

Cultures grew S. pneumonaie, which is also unusual with only a couple of dozen cases reported:

The brain abscesses which are caused by Streptococcus pneumoniae account for less than one percent of all the brain abscesses, the commonest agents being the anaerobes and the non haemolytic streptococci

and even rarer in the cerebellum.

And it was one beast of a pneumococcus. The MIC to penicillin was 6, and to ceftriaxone, it was 3. And it was resistant to all the common antibiotics as well.

Back at the beginning of my career, resistant pneumococci were starting to be an issue. The conjugated vaccine came along and since the resistant strains were also the ones in the vaccine, one of the side effects of pneumococcal vaccination was a decline in resistant infections.

Although a reduction in disease caused by most penicillin-resistant strains has been reported, reductions in intermediately penicillin-resistant strains has not been shown to the same degree, because of increased intermediate resistance among non-PCV7 serotypes.

I have not seen a Pneumococcus this resistant this century; I suspect it is a harbinger that something wicked this way comes. Resistance, to misquote the Borg, is inevitable.

Where she got the Pneumococcus is a mystery. She is not around children, no travel, lives in a rural area, and did not get either pneumococcal vaccine.

She did fine with drainage and vancomycin.

Rationalization

J Natl Med Assoc. 1961 Nov; 53(6): 607–609. PMCID: PMC2642049 PMID: 13910495 The Problem of Brain Abscess Complicating Mastoiditis

https://www.ncbi.nlm.nih.gov/pmc/articles/PMC2642049/?page=1

CEREBELLAR ABSCESS TREATMENT BY EXCISION WITH THE AID OF ANTIBIOTICS

https://jnnp.bmj.com/content/jnnp/11/1/1.full.pdf

Cerebellar Brain Abscess Associated with Tongue Piercing Richard A. Martinello Elizabeth L. Cooney Clinical Infectious Diseases, Volume 36, Issue 2, 15 January 2003, Pages e32–e34,

https://doi.org/10.1086/345755

Pyogenic brain abscess caused by Streptococcus pneumoniae: case report and review.

https://www.ncbi.nlm.nih.gov/pubmed/9402366

Impact of conjugate pneumococcal vaccines on antibiotic resistance

https://www.sciencedirect.com/science/article/pii/S1473309908702810

Effect of Introduction of the Pneumococcal Conjugate Vaccine on Drug-Resistant Streptococcus pneumoniae.

https://www.nejm.org/doi/full/10.1056/nejmoa051642

POLL RESULTS

As the philosopher has noted

  • The more things change, the more they stay the same 14%
  • Those who forget the past are doomed to repeat it 17%
  • Those who remember the past are doomed to repeat it 6%
  • You will be assimilated. Resistance is inevitable 44%
  • What? Me Worry? 17%
  • Other Answers 3%
  • That which does not kill us makes us stronger, but can REALLY piss us off as well.

Old MacDonald

Jun 25, 2018

Old MacDonald Had a Farm

Eos Eos O

The patient had a renal transplant two years ago, and everything was peachy. She felt great, but over the last six months, her eosinophils kept rising. First, she was sent to heme-onc, who then sent her to me.

In the clinic, she was doing just fine and, through an interpreter, I found that she grew up on a farm in SE Asia, often swam in the local river and lakes, had immigrated to the US 20 years ago, and had not been back, had no medical problems until her kidneys failed from hypertension.

The exam was negative, as were the labs except for the aforementioned eosinophilia. She was on tacrolimus and the usual, almost homeopathic, 5 mg of prednisone. It is really amazing how little prednisone these patients are on, especially compared to the cyclosporine era. 5 mg hardly seems worth it.

With no masses/cysts anywhere on the old CTs, I figured it was due to one of two reasons. Schistosomiasis or Strongyloides and sent off the studies.

Wrong.

It was both.

Schistosomiasis was the easy one. A short course of the Mayan God, or is it Aztec?, Praziquantel should eradicate the likely Schistosoma mekongi.

The Strongyloides? I don't know about that.

First, screen all transplants from endemic areas. It can persist a long time, as the title makes clear:

Strongyloidiasis in Man 75 Years after Initial Exposure

That's a long, long time.

Second, I wonder if she did not get into trouble with hyperinfection as she was on such a piddly dose of prednisone, as

A review of Strongyloides hyperinfection syndrome in renal transplant recipients reported that disease typically occurred during the first 3 months after transplantation...

And she has been on immunosuppression for 2 years. Although maybe not, as one report suggests, tacrolimus isn't that much better, and in the old days

Cyclosporine probably has a protective effect against strongyloides.

And lastly, is it really a curable infection?

However, S. stercoralis DNA was detected by PCR in all patients, both in their first and subsequent stool samples, thus reflecting the poor efficacy of ivermectin at eradicating parasite from host tissues. Asymptomatic eosinophilia was the most frequent clinical form among chronically infected patients. These results suggest that the parasitological cure is unlikely. Strongyloidiasis must be considered a chronic infection and ivermectin administration schedules should be reevaluated.

Time will tell, as the cliche makes clear.

Rationalization

Southeast Asian J Trop Med Public Health. 1984 Dec;15(4):431-8. Epidemiology and control of schistosomiasis in Southeast Asia.

https://www.ncbi.nlm.nih.gov/pubmed/6535256

Strongyloidiasis in Transplant Patients.

https://academic.oup.com/cid/article/49/9/1411/302018

Strongyloidiasis in Man 75 Years after Initial Exposure.

https://www.ncbi.nlm.nih.gov/pmc/articles/PMC3321755/

Strongyloidiasis Outside Endemic Areas: Long-term Parasitological and Clinical Follow-up After Ivermectin Treatment.

https://academic.oup.com/cid/article-abstract/66/10/1558/4817467?redirectedFrom=fulltext

Transplantation. 1996 Oct 15;62(7):1038. Tacrolimus allows autoinfective development of the parasitic nematode Strongyloides stercoralis.

https://www.ncbi.nlm.nih.gov/pubmed/8878405

Transplant Proc. 2007 May;39(4):1014-5. Hyperinfection strongyloidiasis: an anticipated outbreak in kidney transplant recipients in Kuwait.

https://www.ncbi.nlm.nih.gov/pubmed/17524878

Reactivation

Jun 27, 2018

A week prior to admission, the patient has L34 disc surgery for a herniated disk with weakness. She did just fine, then had the abrupt onset of fevers and altered mental status.

She was seen in the ER with the worry that she had meningitis. MRI showed normal postop changes, no abscess. The LP had a normal glucose, 90 WBC, mostly lymphocytes, and a protein of 120.

Certainly not bacterial meningitis, but he was admitted, started on ampicillin, ceftriaxone and acyclovir.

The next day it was noted she had a few blisters in the L34 dermatome, and I was called.

It sure looked like shingles to me.

So we, and by we, I mean me, sent off both a VZV on the CSF and the skin blister.

And bingo was his name-o. Both were positive.

VZV can cause an encephalitis, so I continued the acyclovir, but I wondered: did the surgery lead to reactivation, or was it true/true and unrelated. HSV does like to re-activate with stress, and surgery is nothing if not stressful.

Pubmed to the rescue, as always.

Perhaps.

Herpes simplex or herpes zoster reactivation after spinal surgery is rarely reported. This case report and review of the literature describes patients in whom this reactivation occurs to clarify the diagnosis and management. In addition to reporting their case, the authors reviewed case reports and series published between 1980 and 2012 found through a PubMed search. Herpes reactivation is generally confined to a vesicular rash that can be treated with acyclovir. However, occasional dissemination has occurred and has led to myelitis or encephalitis.

Although there are only a handful of cases, including a case of

...foraminal disk extrusion that resulted in radicular pain in a dermatomal distribution. The resultant inflammatory response around the dorsal root ganglion triggered an episode of shingles, which elegantly highlights the pathophysiology of this condition at a gross anatomic level.

Elegant and gross. Such is medicine.

Rationalization

J Neurosurg Spine. 2013 May;18(5):519-23. doi: 10.3171/2013.1.SPINE12960. Epub 2013 Mar 1. Herpes myelitis after thoracic spine surgery.

https://www.ncbi.nlm.nih.gov/pubmed/23452248

World Neurosurg. 2018 Jan;109:138-139. doi: 10.1016/j.wneu.2017.09.132. Epub 2017 Sep 27. Diskogenic Reactivation of Herpes Zoster.

https://www.ncbi.nlm.nih.gov/pubmed/28962964

Dog Daze

Jul 2, 2018

All that glitters is not gold

All who wander are not lost

All that is coagulase-positive is not S. aureus

JRR Tolkien

He was quite the writer.

The patient fell and broke his leg, not an open fracture. It is repaired with a plate and screws. It initially does well, but three weeks in, it gets red, hot, swollen, and tender.

Debrided in the OR, pus tracks down to the plate.

Gram stain? Gram-positive cocci.

Coagulase-positive.

Staphylococcus.

pseudintermedius.

In humans, the coagulase-positive Staphylococcus is aureus, but

Other coagulase-positive staphylococci are primarily of relevance for dogs and cats, namely S. intermedius, S. schleiferi subsp. coagulans, and S. pseudintermedius.

The patient has 5 small dogs that sleep in his bed, and

Nearly half of all British dog and cat lovers sleep with their pets despite hygiene concerns, a new survey has found.

Nearly one in five (18 percent) of pet lovers also let their pooch or moggy eat from their family’s plates.

A third of dog or pet owners (32 percent) admit to regularly finding animal hairs in their food.

Shockingly, a third of Brits (30.5 percent) of animal lovers never wash or clean their pets, with around 25 percent not washing their food and water bowls regularly. It is no different in the US. At least the animals did not/do not lick his wound.

The prevalence of S. pseudintermedius colonization was significantly higher in dogs at 46.2% than in humans or cats...In dogs, S. pseudintermedius colonization was significantly greater than S. aureus, and rectal colonization was significantly more common than nasal colonization.

Still, I think of humans like Pig-Pen and dogs are no better. We share more than good times with our pets.

It has been under-identified in the past, being misidentified as S. aureus. New technologies make that less likely.

There are 65 hits on the Pubmeds for infections due to this organism, mostly cellulitis, but a hodgepodge of other infections.

Most patients had confirmed contact with dogs at time of infection. S. pseudintermedius was isolated in 18 cases (75.0%) of skin and soft tissue infections (SSTI)...

Mine, or the patients (who is the owner?), was methicillin-sensitive, and he did quite well.

Rationalization

Coagulase positive staphylococcal colonization of humans and their household pets.

https://www.ncbi.nlm.nih.gov/pmc/articles/PMC2726022/

Human infections due to Staphylococcus pseudintermedius, an emerging zoonosis of canine origin: report of 24 cases.

https://www.ncbi.nlm.nih.gov/pubmed/27241371

What's in a Name? The Impact of Accurate Staphylococcus pseudintermedius Identification on Appropriate Antimicrobial Susceptibility Testing.

https://www.ncbi.nlm.nih.gov/pubmed/26763965

Half of Brits let their pets sleep in their bed - despite dangers they pose to their health.

https://www.mirror.co.uk/news/uk-news/half-brits-pets-sleep-bed-10109148

A Bit of ID 4th of July History

Jul 4, 2018

Today is the fourth of July, and I am sort of working. So far, no consults. I hope it stays that way.

I have written about the infection risks associated with holiday traditions in the past. But 4th of July? What infection could you get on this holiday? Burgers, beer, and fireworks hardly seem to be a risk for anything interesting unless you are a trauma surgeon.

I put infection and 4th of July into PubMed and got nothing.

And then I tried the Google.

An amazing bit of history. July 4th fireworks used to be associated with tetanus and was so common it was called patriotic tetanus.

The July Fourth infection rate became so grave that the American Medical Association (AMA) began tracking patriotic tetanus in an annual report. In 1903, 406 fatal cases of patriotic tetanus were reported. The AMA stressed that all penetrating wounds from fireworks and blank cartridges were potential cases of tetanus.

It was noted at the time that the

...dead on the field of the Fourth of July will outnumber the dead on any battlefield during the Spanish-American war.

Many of whom died of lockjaw.

A thorough study concluded...that, as with other fireworks, tetanus-laden dirt from the surrounding environment was simply thrust into the wound at the time of the injury.

Although trauma at the time was remarkable:

Miss M. Postlewait, who was hurt by a flying iron bolt while she was riding a streetcar at North Avenue and Wells Street. The bolt was propelled by the explosion of a giant firecracker.

Robert Kuhn, 13, of 6607 S. Park Ave., who with his cousin Mary exploded firecrackers while his parents were out of the house. When the firecrackers were used up, he started playing with a revolver. He put a bullet through his brain and was killed.

Willie Theiss, 10, who was hit and injured by a piece of a wagon axle that had been turned into a homemade toy cannon and detonated at 441 Irving Ave.

A bank teller in Troy, N.Y., who blew off his hand with a firecracker. "Maddened by the pain," he stabbed himself to death with a potato knife.

A potato knife?!? Whoa. And did he use his dominant or non-dominant hand? More impressive if the latter.

But it does remind me of some of the dumbass things I did as a kid with fireworks. Improvised cannons and bazookas made with pipe, firecrackers, bottle rockets, and batteries. I am lucky to have my fingers and vision.

With a combination of aggressive public health interventions and the vaccine, firework-associated tetanus has become a historical curiosity but does confirm my belief that everything has an infection risk. And that ID is the most interesting field in medicine, if not the world.

Now we get to burn down the West instead.

Sigh.

I have a BBQ to attend, so I will let you read about the details at your leisure.

Happy 4th. And have your tetanus vaccine up to date.

Rationalization

When real patriots got Tetanus

http://americanhistory.si.edu/blog/patriots-got-tetanus

The Fourth of July Had a Tetanus Problem

https://www.theatlantic.com/politics/archive/2013/07/the-fourth-of-july-had-a-tetanus-problem/454578/

How the Tribune Saved America from Fourth of July Lockjaw

http://www.chicagomag.com/city-life/July-2015/How-the-Tribune-Saved-America-From-Fourth-of-July-Lockjaw/

How the Fourth became a day of celebration, not carnage

http://www.chicagotribune.com/news/ct-fireworks-danger-flashback-070112-story.html

Dozens Of 4th Of July Fireworks Cancelled Due To Heat, Wildfires Torching The West

https://www.forbes.com/sites/ericmack/2018/07/03/dozens-of-4th-of-july-fireworks-shows-cancelled-due-to-heat-wildfire-risk/#3972dac61485

Where Did That Come From

Jul 9, 2018

S. aureus bacteremia is bread and butter ID. In fact, bacteremias are bread and butter ID. Different bacteremias have different durations of antibiotics, depending on the source, the host, the bug, etc. etc.

It is clear that most of the time, however, repeating blood cultures to document clearing is a waste of time:

Repeat cultures after 48 h are low yield for most Gram-negative and streptococcal bacteremias.

and

FUBC added little value in the management of GNB bacteremia. Unrestrained use of blood cultures has serious implications for patients including increased healthcare costs, longer hospital stays, unnecessary consultations, and inappropriate use of antibiotics.

The big exception has always been S. aureus. That is because repeat positive blood cultures are the sine qua non of an endovascular infection.

Surveillance cultures are valuable for endocarditis and S. aureus bacteremia, since the presence of persistent bacteremia may guide decisions regarding the need for definitive source control.

So for S. aureus, I always recheck one set of blood cultures, and, positive or negative, I don't usually repeat the blood cultures as long as the patient improves on schedule.

One of the habits I have noted in others is they start the duration of antibiotics from the date of the last positive blood culture. Where did that come from? I assume there is data to support the practice. Not that I can find.

One review notes for S. aureus in dialysis patients

The 21-day duration is determined from the last positive blood culture

And references the IDSA guidelines, but such a recommendation is not in the guideline. It looks like that is how coagulase-negative bacteremia is treated in underweight neonates. But that is it.

If there is a reason for using the date of the last positive blood culture as day 1 of antibiotics, I can't find it.

It adds to the duration of antibiotics and with it cost and toxicities plus the cost of unneeded blood cultures. It is one of those habits that have crept in for no good reason.

Given the quasi-random nature of duration, I can't believe another day or two will make any difference in the outcome. If we had 8 fingers or 6 day weeks, the course of therapy would have different durations.

So I am sticking with my unusual practice of having the first day of antibiotics being the, well, first day of antibiotics.

Rationalization

Sending repeat cultures: is there a role in the management of bacteremic episodes? (SCRIBE study).

<https: data-preserve-html-node="true"//www.ncbi.nlm.nih.gov/pmc/articles/PMC4906775/>

Follow-up Blood Cultures in Gram-Negative Bacteremia: Are They Needed.

<https: data-preserve-html-node="true"//doi.org/10.1093/cid/cix648>

Management of Gram-Positive Coccal Bacteremia and Hemodialysis.

<https: data-preserve-html-node="true"//www.ajkd.org/article/S0272-6386(11)00034-5/pdf>

How to Figure Out the Length of Antibiotic Therapy

<https: data-preserve-html-node="true"//blogs.jwatch.org/hiv-id-observations/index.php/how-to-figure-out-the-length-of-antibiotic-therapy/2010/10/22/>

POLL RESULTS

Day one of antibiotics is from

  • the last positive culture 16%
  • the first dose on antibiotic 45%
  • day 2, I use zero-based counting 8%
  • the discharge date 0%
  • whatever day I can remember 26%
  • Other Answers 5%
    • First negative blood culture
    • day of first negative culture

      The Blame Game

Jul 12, 2018

The patient has a shoulder replacement and a month later comes in with an infected shoulder.

It is debrided, and all the cultures grow Enterobacter cloacae.

So they call me.

This organism is not in the top ten for prosthetic joint infection. In one series, Enterobacter accounted for 14% of prosthetic joint infections.

There are a few out there, but gram-negative prosthetic joint infections are unusual. I bet I see at most one a decade.

So I asked the usual ID exposure questions, and I did discover that the patients' cat loves to curl up on the shoulder/upper chest and that this occurred pre and post-op.

So was the cat the vector?

I don't know. I would have to culture the cat to know for sure, but a PubMed finds little concerning cats and Enterobacter. One case of otitis media and the occasional isolate from companion animals. And humans and pets often share the same microbiome.

So maybe. This is not the first or last infection I have seen that could be blamed on a pet, and there is no other reason for an Enterobacter.

If it was the vermin, er, I mean cat, I would wager there is little purpose in emphasizing it with the patient. People are remarkably attached to their pets, and often it is their primary companion/emotional support. Given that such an infection is a fluke and reinfection is unlikely, no sense in driving a wedge between the patient and their vermin, er, I mean pet.

The outcome was not good. The shoulder needed to be removed, and she is on a course of antibiotics.

In the future, to prevent SSI, perhaps we should suggest autoclaving the family pet?

Nah. That won't go over well.

Rationalization

Gram-negative prosthetic joint infection: outcome of a debridement, antibiotics and implant retention approach. A large multicentre study. https://www.clinicalmicrobiologyandinfection.com/article/S1198-743X(14)65347-7/fulltext

Clinical isolates of bacteria from domestic cats in Grenada, and their antimicrobial susceptibility https://sta.uwi.edu/fms/vet/documents/clinical.pdf

MCR-1 in Enterobacteriaceae from Companion Animals, Beijing, China, 2012–2016 <https: data-preserve-html-node="true"//wwwnc.cdc.gov/eid/article/23/4/pdfs/16-1732.pdf>

This Time It's Real

Jul 16, 2018

I often get called upon to decide if a culture is real. Well, the culture is real. Outside of germ deniers, and there are such people, bacteria are real. The question is whether it is pathogenic or, perhaps, a contamination.

Coagulase-negative Staphylococci are particularly problematic.

The patient, on prednisone and Imuran, comes in with fevers, chills, altered mental status, and a small patch of rubor, dolor, calor, and tumor on her leg.

Cellulitis.

She improves on vancomycin, and all the blood cultures grow gpc in clusters. S. aureus, right?

Nope.

Coagulase-negative. In 6 bottles. All positive in less than 24 hours. The MALDI calls it S. epidermidis.

Hm. If it had been a S. lugdunensis, maybe, but how often do coagulase-negative Staphylococci cause soft tissue infections? Not often, I suspect, but who knows since a positive culture from a skin source is likely to be ignored, as would a positive blood culture. Coagulase-negative Staphylococci doesn't cause cellulitis, so the cultures are interpreted accordingly.

One review suggests

Skin and soft tissue infections have been observed to be caused by many coagulase-negative staphylococcus organisms: Staphylococcus auricularis, Staphylococcus capitis, Staphylococcus epidermidis, Staphylococcus haemolyticus, Staphylococcus hominis, Staphylococcus lugdunensis, Staphylococcus saprophyticus, and Staphylococcus simulans. Coagulase-negative staphylococcus skin infections predominantly present as abscesses and paronychia. They are most common in elderly patients or those individuals who are immunosuppressed and tend to be broadly susceptible to antibiotic treatment.

ECHO showed no vegetation. So I suppose it is causative. Hard to ignore a high-grade bacteremia in a poor host with a compatible clinical syndrome.

And then how long to treat? Who knows. Most studies suggest longer courses of antibiotics are no better than 5-7 days. So I opted for 7. Why? Because

  • Seven, the fourth prime number, is not only a Mersenne prime (since 23 − 1 = 7) but also a double Mersenne prime since the exponent, 3, is itself a Mersenne prime. It is also a Newman–Shanks–Williams prime, a Woodall prime, a factorial prime, a lucky prime, a happy number (happy prime), a safe prime (the only Mersenne safe prime), and the fourth Heegner number.

  • Seven is the lowest natural number that cannot be represented as the sum of the squares of three integers. (See Lagrange's four-square theorem#Historical development.)

  • Seven is the aliquot sum of one number, the cubic number 8, and is the base of the 7-aliquot tree.

  • 7 is the only number D for which the equation 2n − D = x2 has more than two solutions for n and x natural. In particular, the equation 2n − 7 = x2 is known as the Ramanujan–Nagell equation.

  • 7 is the only dimension, besides the familiar 3, in which a vector cross product can be defined.

  • 7 is the lowest dimension of a known exotic sphere, although there may exist as yet unknown exotic smooth structures on the 4-dimensional sphere.

  • 999,999 divided by 7 is exactly 142,857. Therefore, when a vulgar fraction with 7 in the denominator is converted to a decimal expansion, the result has the same six-digit repeating sequence after the decimal point, but the sequence can start with any of those six digits. For example, 1/7 = 0.142857 142857... and 2/7 = 0.285714 285714....

  • In fact, if one sorts the digits in the number 142,857 in ascending order, 124578, it is possible to know from which of the digits the decimal part of the number is going to begin with. The remainder of dividing any number by 7 will give the position in the sequence 124578 that the decimal part of the resulting number will start.

  • A seven-sided shape is a heptagon. The regular n-gons for n ≤ 6 can be constructed by compass and straightedge alone, but the regular heptagon cannot. Figurate numbers representing heptagons (including seven) are called heptagonal numbers. Seven is also a centered hexagonal number.

  • Seven is the first integer reciprocal (multiplicative inverse) with infinitely repeating sexagesimal representation.

  • There are seven frieze groups, the groups consisting of symmetries of the plane whose group of translations is isomorphic to the group of integers.

  • There are seven fundamental types of catastrophes.

  • The Millennium Prize Problems are seven problems in mathematics that were stated by the Clay Mathematics Institute in 2000. Currently, six of the problems remain unsolved.

  • 7 is the last digit of Graham's number

Good reasons all and, besides, five can't hold a candle to that. And 6? Who gives six days?

And she did just fine.

Rationalization

"It's just a theory"

https://sciencebasedmedicine.org/its-just-a-theory/

Am J Clin Dermatol. 2018 Jun 7. doi: 10.1007/s40257-018-0362-9. Coagulase-Negative Staphylococcus Skin and Soft Tissue Infections.

<https: data-preserve-html-node="true"//www.ncbi.nlm.nih.gov/pubmed/29882122>

SAB

================ Jul 19, 2018

S. aureus bacteremia. SAB. Again. As I have said before, if I were a superhero, S. aureus would be my arch-nemesis.

Can't ever really kill it, and it always comes back stronger.

This time the source is a dialysis graft site that was debrided and, much to my surprise, was MSSA.

Patients fever resolved in 24 hours, repeat blood cultures were negative, so all that is left is to decide on a course of therapy.

There are a few things that bug me about SAB.

The first is getting an echo.

The guidelines suggest everyone should get a tee.

That's stupid, a waste of time and resources, and not without risk. There are two questions when getting any test. Why get it, and is it going to change therapy. So often, echos are done, and they do not change

the plan of antibiotics nor help with the diagnosis.

And ID doctors know this and often do not order echos:

Consulting Infectious Diseases physicians most frequently cited low suspicion for endocarditis due to rapid clearance of blood cultures and the presence of a secondary focus requiring an extended treatment duration as reasons for foregoing TEE.

Yep. That's me. I rarely get tee's to make a diagnosis, I usually get them looking for either ring abscess or valve destruction that would warrant surgical intervention

But then, if you are not going to get a tee, should you bother with a TTE?


Ordering a test with low sensitivity for a disease with little prior probability is the definition of irresponsible medical care.

And tte often add nothing :

We found that TTE results do not significantly affect duration of antibiotic therapy. Rather, duration depended solely on clinical factors. This result is also consistent with the results reported by Lindner et al, who found that clinical parameters were predictive of antibiotic course, and were unaffected by either TTE or TEE. Ali et al also found in a retrospective review of 2750 serial TTEs that demonstration of a vegetation in 15 of 20 cases had little effect on initiation of IE therapy in patients with a low clinical suspicion of IE.

So why do it? My colleagues suggest that it is because occasionally you find something. Perhaps. But practicing medicine based on the exception and not the rule is not the way to provide quality care. There are always exceptions, but as a rule, most endocarditis cases do not need a diagnostic echo (there are other reasons to get an echo), and for S. aureus bacteremia, probably either a tee or nothing.

But then I am old, so old I started taking care of endocarditis before echocardiography. Who needs those new-fangled tests when there is an H&P?

And the other thing that bugs me? In my system, SAB is an automatic ID consult. Many studies demonstrate improved care as a result. There was just another study validating decreased mortality with an ID consult:

Infectious disease (ID) consultation in patients with Staphylococcus aureus bacteremia who were at low risk for endocarditis and who had no secondary site of infection was associated with a longer course of antibiotics (median duration of intravenous antimicrobial therapy of 31 days and 15 days in those with and without ID consultation, respectively; P ≤ .01), and based on Kaplan-Meier survival analysis, reduced in-hospital mortality (P = .2), and reduced 30-day mortality after discharge (P = .4)

But 48% of patient with SAB did not have an ID consult.

That is similar to other studies. My take on these ID consults for SAB studies is they show that too many institutions let people who have no clue what they are doing manage SAB.

But the big question is who, in 2018, lets non-ID doctors kill people with their incompetence? Er, I mean, who lets non-ID doctors manage SAB? It would seem to be a very bad idea unless you lack cases for

M&M or morgue conference.

Rationalization

Clinical Reasoning of Infectious Diseases Physicians Behind the Use or Nonuse of Transesophageal Echocardiography in Staphylococcus aureus Bacteremia.

<https: data-preserve-html-node="true"//www.ncbi.nlm.nih.gov/pmc/articles/PMC5102142/>

Overuse of Transthoracic Echocardiography in the Diagnosis of Native Valve Endocarditis.

<https: data-preserve-html-node="true"//jamanetwork.com/journals/jamainternalmedicine/fullarticle/754054>

Association of Infectious Disease Consultation With Clinical Outcomes in Patients With Staphylococcus aureus Bacteremia at Low Risk for Endocarditis

<https: data-preserve-html-node="true"//doi.org/10.1093/ofid/ofy142>

The first? Probably not.

Jul 24, 2018

The patient comes in febrile, ill enough to require a short stay in the ICU. All her blood cultures grow Group A Streptococcus. Been there done that.

But. No source is evident. And there is a bioprosthetic valve. Endocarditis is not that common due to S. pyogenes.

Last century I saw a patient with cellulitis and S. pyogenes bacteremia. The hospitalist had ordered an echo, and I scoffed, albeit politely. Group A Streptococcus, I said, doesn't cause endocarditis. I know it does, but in medicine, you play the odds. Common things are common and uncommon things are, well, uncommon.

Group A streptococcus is the cause of infective endocarditis between 0 and 5% of cases in reported series. Since the introduction of penicillin 69 cases of group A streptococcus endocarditis have been reported in the literature.

That was from 1991. There have likely been a few more since. But the echo had been done, showed a large mitral vegetation, and the infection destroyed the valve requiring replacement—my one and only case of S. pyogenes endocarditis.

Until now. As the TTE showed a vegetation flopping on the prosthesis.

How common is prosthetic valve endocarditis due to S. pyogenes? Not very.

Now my searches for this blog, while not perfunctory, are also not exhaustive, limited by time and paywalls.

But neither PubMed nor the Googles yield a single case. Ever. So maybe this is the first reported case in the quasi-sort-of-blog-medical literature of S. pyogenes prosthetic valve endocarditis.

Six weeks of beta-lactams was evidently a cure. And that would be a first as well.

Rationalization

J Infect. 1991 Nov;23(3):307-16. Group A streptococcus endocarditis: report of five cases and review of literature.

https://www.ncbi.nlm.nih.gov/pubmed/1753141

Rapid

Jul 25, 2018

The patient, receiving docetaxel for prostate cancer, comes in neutropenic with a rapidly spreading necrosis from the rectum, around the perineum, and into the thighs. He had been ill for less than a day.

There is air in the soft tissues of the pelvis, and he is in multorgn systems failure.

They call me from the ER for antibiotic treatment of a Fournier's. Treat the usual mixed, I said, but it sounds like Clostridia, maybe septicum or perfringens, so add some clindamycin as well.

Rushed to the OR, extensive dead muscle and soft tissue are found, and he expires immediately post-op despite the usual care.

Gram-positive rods are on the gram stain, and all the blood cultures grow C. septicum. I hate that bug. Few bacteria kill as rapidly as C. septicum, with only Meningococcus and Plague as rivals.

The hint it was C. septicum? All the gas. I had one case where the tech told me the blood culture squirted like a seltzer bottle there was so much gas in the bottle. And the lack of hemolysis, which is seen with C. perfringens. And no bullae.

My only post mortem consult was due to this organism. In my first year in practice, I was called by the ICU staff, freaking out as a patient had come in coding and died. While they were preparing the body, they could see a spreading cellulitis over the buttock and thigh that continued after death. As I examined the body, the lab called with the positive blood culture, a gram-positive rod.

But why. Part the neutropenia. But perhaps the docetaxel, which may be associated with typhlitis.

Taxane therapy is associated with necrotising inflammation of the caecum (named also typhlitis) that could be a potential source for bacteraemia. We report the case of a sudden onset of septic shock by Clostridium perfringens in a young patient treated with docetaxel as adjuvant chemotherapy for early breast cancer.

J

ust what a Clostridia needs to get a toe hold from the cecum and spread. It probably didn't help that he had radiation as well.

There is one similar case in the literature:

The patient is a 44-year-old woman with metastatic grade 3 intra-ductal carcinoma of the breast who was started on palliative chemotherapy (docetaxel) 10 days prior to admission and presented to the emergency center complaining of diffuse abdominal pain and generalized weakness. CT abdomen showed diffuse bowel wall thickening from the cecum to the transverse colon with free fluid in the pelvis. The patient was neutropenic on admission (absolute neutrophil count of 600 cells/μl)....While C. chauvoei has a long history of veterinary importance, this is the first documented case of infection caused by C. chauvoei in a human in the United States. C. chauvoei has a close phylogenetic relationship with C. septicum making the two species difficult to differentiate using conventional microbiologic methods.

Awful case, awful bug, awful outcome.

Rationalization

Neutropenic enterocolitis (typhilitis) associated with docetaxel therapy in a patient with non-small-cell lung cancer: case report and review of literature

https://www.sciencedirect.com/science/article/pii/S0169500203005865

Fulminating septic shock from Clostridium perfringens in an early breast cancer patient with severe myalgia after docetaxel treatment.

https://www.ncbi.nlm.nih.gov/pmc/articles/PMC3981246/

Severe colitis associated with docetaxel use: A report of four cases

https://www.ncbi.nlm.nih.gov/pmc/articles/PMC2999675/

J Infect. 2012 Feb;64(2):225-7. doi: 10.1016/j.jinf.2011.09.004. Epub 2011 Sep 16. Lethal human neutropenic entercolitis caused by Clostridium chauvoei in the United States: tip of the iceberg? https://pubmed.ncbi.nlm.nih.gov/21945880/

**

Smile

Jul 30, 2018

Start every day off with a smile and get it over with.

W. C. Fields

The patient has quite the MSSA infection. Epidural abscess, bacteremia, endocarditis, septic hip. What could be debrided was debrided, and she was placed on cefazolin, and while she did not get better, she did not worsen. Fevers, leukocytosis persisted, and she just looked sick. In my first year in practice, an old surgeon told me don't worry about a patient who can smile. She was a long way from smiling.

Ten days in, I pick up the patients care.

If there is one thing I know how to treat, it's MSSA. And if there is one thing I don't know how to treat, it's MSSA.

When I was a medical student doing my ID rotation (a life-changing event, altering my career path from FP to ID), I was taught not to use cefazolin for MSSA infections. Cefazolin can be somewhat degraded by some Staphylococcal beta-lactamases, perhaps leading to an increase in treatment failure rate.

Who am I to argue with Dr. Bryant? I learned it as a medical student, and like much of what I learned back in the day, I took it as a commandment.

Then in the last few years, there have been some clinical studies to show cefazolin and nafcillin have equivalent outcomes, but cefazolin is cheaper and less toxic:

Cefazolin had clinical efficacy similar to that of nafcillin and was more tolerable than nafcillin for the treatment of MSSA bacteremia.

and

Compared to antistaphylococcal penicillins, cefazolin was associated with significant reductions in nephrotoxicity and hepatotoxicity in hospitalized patients and outpatients. Additionally, cefazolin was associated with lower likelihoods of discontinuation due to side effects in hospitalized patients and hypersensitivity reactions in outpatients. Cefazolin should be considered a first-line option for patients with MSSA infections for which efficacy is presumed to be similar to that of antistaphylococcal penicillin therapy.

and cefazolin may be superior with the only outcome that really matters

We found a large reduction in 90-day mortality in those receiving cefazolin compared to nafcillin for MSSA BSI, but this finding was not statistically significant.

and

In this large, multicenter study, patients who received cefazolin had a lower risk of mortality and similar odds of recurrent infections compared with nafcillin or oxacillin for MSSA infections complicated by bacteremia.

Kind of compelling. Cheaper, less toxic, less mortality.

So I have been using a lot more cefazolin.

But. The more things change, the more they say the same. The cefazolin inoculum effect keeps rearing its ugly head.

Type A Bla efficiently hydrolyzes cefazolin. Strains that produce a large amount of this enzyme have been associated with cefazolin failures in patients suffering from high-inoculum staphylococcal disease (e.g., endocarditis). Unlike low-level Bla producers, high-level producer strains typically show high cefazolin MICs when a large inoculum is used

with

19% of MSSA clinical isolates showed a pronounced inoculum effect with cefazolin, a phenomenon that could explain the cases of cefazolin failure previously reported for hemodialysis patients with MSSA bacteremia. These results suggest that for serious MSSA infections, the presence of a significant inoculum effect with cefazolin could be associated with clinical failure in patients treated with this cephalosporin, particularly when it is used at low doses.

Which increases the failures of cefazolin. In Argentina, it was found in 19% of strains, but in Chicago, it was low, leading to the conclusion that

Our predilection for cefazolin use, even early in the management of hospitalized MSSA infections, is tenable.

Cefazolin or Nafcillin? No clue. Safe to say, it gives me the Bla's.

But I changed her to nafcillin, and over the next five days, the white count fell, and the fevers normalized. True-true and unrelated? Cause and effect? No clue.

When it comes to treating MSSA, I really wonder if I have all the information I need to decide on the right therapy.

But today? She smiled. Good enough for me.

Rationalization

Antimicrob Agents Chemother. 2011 Nov;55(11):5122-6. doi: 10.1128/AAC.00485-11. Epub 2011 Aug 8. Is cefazolin inferior to nafcillin for treatment of methicillin-susceptible Staphylococcus aureus bacteremia?

https://www.ncbi.nlm.nih.gov/pubmed/21825299

Antimicrob Agents Chemother. 2018 Mar 27;62(4). pii: e01816-17. doi: 10.1128/AAC.01816-17. Print 2018 Apr. Systematic Review and Meta-analysis of the Safety of Antistaphylococcal Penicillins Compared to Cefazolin.

https://www.ncbi.nlm.nih.gov/pubmed/29437617

Antimicrob Agents Chemother. 2016 Jul 22;60(8):4684-9. doi: 10.1128/AAC.00243-16. Print 2016 Aug. Cefazolin versus Nafcillin for Methicillin-Sensitive Staphylococcus aureus Bloodstream Infection in a California Tertiary Medical Center.

https://www.ncbi.nlm.nih.gov/pubmed/27216053

Clin Infect Dis. 2017 Jul 1;65(1):100-106. doi: 10.1093/cid/cix287. Comparative Effectiveness of Cefazolin Versus Nafcillin or Oxacillin for Treatment of Methicillin-Susceptible Staphylococcus aureus Infections Complicated by Bacteremia: A Nationwide Cohort Study https://www.ncbi.nlm.nih.gov/pubmed/28379314

Antimicrob Agents Chemother. 2009 Aug; 53(8): 3437–3441. Published online 2009 Jun 1. doi: 10.1128/AAC.00317-09 Inoculum Effect with Cefazolin among Clinical Isolates of Methicillin-Susceptible Staphylococcus aureus: Frequency and Possible Cause of Cefazolin Treatment Failure.

https://www.ncbi.nlm.nih.gov/pmc/articles/PMC2715590/

Antimicrob Agents Chemother. 2018 Jul 27;62(8). pii: e00382-18. doi: 10.1128/AAC.00382-18. Print 2018 Aug. Prevalence of a Cefazolin Inoculum Effect Associated with blaZ Gene Types among Methicillin-Susceptible Staphylococcus aureus Isolates from Four Major Medical Centers in Chicago.

https://www.ncbi.nlm.nih.gov/pubmed/29891607

Poll Results

I don't worry about a patient who

  • smiles 21%
  • remembers my name 9%
  • has the health care plan of a US Senator 18%
  • has an ID consult 26%
  • asks for a beer 24%
  • Other Answers 3%
    • I am not connected with you in any way. All the others... yes.

Calling the Zebra

Aug 14, 2018

The patient is admitted with the worst headache of his life. In the ER an LP is done and it has 1000 WBC, 90% neutrophils, a normal protein and a slightly low glucose.

He is placed on the usual meningitis cocktail (not as good as an Army Navy, but then most medical cocktails never are) and over the next 48 hours gets better.

But the etiology remains uncertain, as all the cultures are negative, so they call me.

It is an odd history as it starts as low back pain that progresses to a headache. No fevers. He has no risks for odd infections, and when I see him, the exam is negative; he is laughing and visiting with friends.

I am at a bit of a loss as to the cause and order the CSF PCR panel. It wasn't ordered initially as the EPIC order has $$$$ with the test. Still, really, it is the job of an internist to make a diagnosis, and making the correct diagnosis is always the road to optimal care.

But that is negative as well.

I am flummoxed and suspect it is viral meningitis.

As the patient was being discharged, I get an idea. From where? Got me. I still find it odd how ideas burble to consciousness and how much of my cognition about cases is done below my awareness. Sure, I tell the house staff a smoke and mirrors post hoc explanation for the diagnosis, but in reality, I often have no idea how I arrive at a diagnosis.

Pop. An idea.

I suggested it could all be due to a ruptured epidermal cyst.

A week later, he is readmitted with the same symptoms, and this time, an MRI of the lumbar spine was done.

And bingo was his name-o.

data-url-image.png

He went to the OR, where the epidermal cyst, filled with a pearl-colored fluid, was removed.

Epidermoid cysts rupture and are a rare cause of a chemical meningitis, which this was clinically and on the lp.

We report a 53-year-old woman with a rare ruptured lumbar intraspinal epidermoid cyst causing chemical meningitis evaluated with MRI (including diffusion-weighted imaging), with histopathologic correlation.

After the diagnosis was made, the patient and the mother mentioned that a) they worried about a tethered spinal cord as a child and b) he always had this odd dimple over his lumbar spine. It never occurred to them to mention it, and we never asked. Now I will admit I never looked at his spine; I just thumped it looking for tenderness.

Sigh.

But of course, I bragged endlessly that I called it before the MRI. What is the point in false modesty? Plus, I am wrong so often, I want credit for when I get it right.

Rationalization

Recurrent meningitis due to epidermoid

https://www.ncbi.nlm.nih.gov/pmc/articles/PMC3401655/

Mysterious Meningitis: Recurrent chemical meningitis due to ruptured epidermoid cyst (P5.124)

http://n.neurology.org/content/90/15\_Supplement/P5.124

Unusual presentation of congenital dermal sinus: tethered spinal cord with intradural epidermoid and dual paramedian cutaneous ostia. https://www.ncbi.nlm.nih.gov/pubmed/23025446

Spontaneously ruptured intraspinal epidermoid cyst causing chemical meningitis. https://www.ncbi.nlm.nih.gov/pubmed/22249012

Wheel of Antibiotics

============================ Aug 15, 2018

What the? Had a couple of curbsides today. One was by the surgical resident, asking me if cefepime was the right antibiotic for a perforated appendix. Another was whether vancomycin and unisin (my spelling and my opinion of the antibiotic) were the right antibiotics for cellulitis.

Hrm.

Here is my latest theory on how antibiotics are chosen. At some point in medical school, a chip is inserted into the brain that models the Wheel of Fortune but instead has antibiotics in the slots. When it comes time to treat an infection, the wheel spins, and where the arrow stops decides the antibiotic given. Totally random. I think that is the best theory that accounts for the behavior I witness.

The patient is a healthy, elderly female who has had a month of fevers, rigors, weight loss, and progressive fatigue.

Work up yields little in the clinic, and she goes on vacation, where a fever and a rigor lead to an ER visit, a short hospitalization, and, after discharge, positive blood cultures.

Two sets grow Fusobacterium nucleatum, and she is given a 7-day course of Augmentin. It was odd that no one asked why the bacteremia?

She felt a bit better, comes home, and sees her PCP, who calls me.

What do you make of the Fusobacterium?

I don't know, but the source has to be in the belly. When I see Fusobacterium, I think septic thrombophlebitis/pyelephlebitis, Lemierre's syndrome of the gut.

Infectious suppurative thrombophlebitis of the portal venous system, referred to as pylephlebitis, is a rare complication of intra-abdominal inflammatory processes...The majority of patients have concomitant bacteraemia on presentation most commonly with typical gastrointestinal (GI) organisms. On rare occasion, patients have culture positive Fusobacterium,

It is what I have seen in the past, but it could be some other GI infection. Tics or appendicitis etc. Get a CT.

No septic thrombophlebitis, but the liver was full of abscesses. She had a full hepatitis evaluation in the clinic as her LFTs were about 2 times normal, hardly impressive for the amount of abscess. Eventually, I saw her, and by history, I think this was a complication of diverticulitis, as no other reason was evident by history or CT.

With Fusobacterium bacteremia

Fusobacterium nucleatum was the most common species identified (41.5%). The mean age +/- standard deviation of the patients was 62.6 +/- 18.9 years. The most common sources of bacteremia were the gastrointestinal tract, lower respiratory tract, and skin and soft tissue.

But as a cause of liver abscess, not so much. In a review

Forty-eight cases were identified... F. nucleatum and F. necrophorum were involved in 22 cases each, and 4 cases were not further speciated. Among cases of F. nucleatum liver abscess, nine were attributed to periodontal disease, four to lower gastrointestinal tract disease, one to Lemierre's Syndrome, and eight were considered cryptogenic. All patients treated made a full recovery.

As did mine with drainage and a course of oral metronidazole.

Rationalization

J Microbiol Immunol Infect. 2009 Aug;42(4):336-42. Fusobacterium bacteremia: clinical significance and outcomes.

https://www.ncbi.nlm.nih.gov/pubmed/19949758

BMC Infect Dis. 2017 Jun 20;17(1):440. doi: 10.1186/s12879-017-2548-9. Fusobacterial liver abscess: a case report and review of the literature.

https://pubmed.ncbi.nlm.nih.gov/28633639/

Infection. 2016 Aug;44(4):475-81. doi: 10.1007/s15010-015-0871-x. Epub 2016 Jan 18. Fusobacterium nucleatum infections: clinical spectrum and bacteriological features of 78 cases.

https://www.ncbi.nlm.nih.gov/pubmed/26783023

BMJ Case Rep. 2017 Oct 20;2017. pii: bcr-2017-221567. doi: 10.1136/bcr-2017-221567. Lemierre's syndrome variant of the gut.

https://www.ncbi.nlm.nih.gov/pubmed/29054895

POLL RESULTS

Antibiotics are chosen

  • by the wheel of fortune 14%
  • by the wheel of time 9%
  • by the most recent drug company lunch 20%
  • by whatever makes me feel good 16%
  • after a careful consideration of the microbiology of the likely infection. Ha. I kill me. 41%

Diarrhea

Aug 21, 2018

Everything causes diarrhea. That, I suppose, is because infections produce cytokines and cytokines cause diarrhea

(TNF) At the higher doses the animals developed hypovolemic shock with an increased hematocrit and watery diarrhea occurred.

and

Fever, chills, and flu-like symptoms were the most common side-effects. The maximum tolerated dose was 6 x 10(5) U/m2 (261 micrograms/m2). Severe hypotension, fluid retention, watery diarrhea, and central nervous deficits culminated in a profound prostration as the dose-limiting toxicity.

My just so explanation is that for most of human history, the major source of infection was food and water, so nausea/vomiting/diarrhea would be a nice evolved response for expeditious source control of ingested pathogens and toxins.

So the question is whether the diarrhea is due to a gastroenteritis or the epiphenomenon of another infection.

The patient, with long-standing controlled HIV, presents with fevers, rigors, bloody diarrhea, and, after 24 hours, negative stool cultures but positive blood cultures for a gram-negative rod.

He had no good exposure, although as far as I am concerned, everything we eat has a fine patina of stool, and with it, perhaps pathogens.

Leafy greens were involved in 363 outbreaks and about 13,600 illnesses, mostly caused by norovirus, E. coli, and salmonella.

Eggs, involved in 352 outbreaks and 11,163 reported cases of illness. Tuna, involved in 268 outbreaks and 2,341 reported cases of illness. Oysters, involved in 132 outbreaks and 3,409 reported cases of illness. Potatoes, involved in 108 outbreaks and 3,659 reported cases of illness. Cheese, involved in 83 outbreaks and 2,761 reported cases of illness. Ice cream, involved in 74 outbreaks and 2,594 reported cases of illness. Tomatoes, involved in 31 outbreaks and 3,292 reported cases of illness. Sprouts, involved in 31 outbreaks and 2,022 reported cases of illness. Berries, involved in 25 outbreaks and 3,397 reported cases of illness

Read the MMWR every week, and you may want to try Breatharianism. Me? I only eat deep-fried food.

I bet on Salmonella, given the bacteremia and HIV.

Nope. But I usually call the cause of diarrhea wrong.

Klebsiella oxytoca.

I have read about Klebsiella oxytoca but have never seen a case. Most of the time, Klebsiella oxytoca causes an antibiotic/hospital-associated diarrhea.

K. oxytoca is not the causative agent of nonhemorrhagic antibiotic-associated diarrhea. This is in contrast to the distinct clinical entity of antibiotic-associated hemorrhagic colitis. Testing for K. oxytoca is therefore only warranted for patients who experience bloody diarrhea during antibiotic therapy.

And may account for 6% of antibiotic-associated colitis and often leads to bacteremia.

In the K. oxytoca group, diarrhea was present in 42 (62%) patients with bacteremia.

Again, knowing the microbiology, no good exposure, and his only antibiotics were HAART:

Klebsiella oxytoca is a gram-negative bacterium that can be found throughout the environment as well as on mucosal membranes of mammals including humans. This bacterium is responsible for a variety of infections in humans including nosocomial infections resulting in hospital outbreaks. Reptiles including snakes, tuataras, and turtles have been shown to harbor this bacterium, and previous studies have shown that pet reptiles are a potential source for dissemination of pathogenic bacteria

So a mystery.

But it was quinolone susceptible, and he did just fine. Didn't even get C. difficle.

Rationalization

Phase I study of 24-hour continuous intravenous infusion of recombinant human tumor necrosis factor.

https://europepmc.org/abstract/med/3183683

Acute in vivo effects of human recombinant tumor necrosis factor.

https://europepmc.org/abstract/med/3599906

Clin Infect Dis. 2008 Nov 1;47(9):e74-8. doi: 10.1086/592074. Role of Klebsiella oxytoca in antibiotic-associated diarrhea.

https://www.ncbi.nlm.nih.gov/pubmed/18808355

S Afr Med J. 2015 Jan 5;105(2):121-5. doi: 10.7196/samj.8654. Prevalence of gastrointestinal pathogenic bacteria in patients with diarrhoea attending Groote Schuur Hospital, Cape Town, South Africa.

https://www.ncbi.nlm.nih.gov/pubmed/26242530

Int J Infect Dis. 2013 Jun;17(6):e472-3. doi: 10.1016/j.ijid.2013.02.003. Epub 2013 Mar 20. Klebsiella oxytoca bacteremia-causal relationship to symptomatic colitis?

https://www.ncbi.nlm.nih.gov/pubmed/23522636

Vector Borne Zoonotic Dis. 2016 Dec;16(12):800-801. Epub 2016 Oct 25. Prevalence of Klebsiella oxytoca in Anolis carolensis of Louisiana.

https://www.ncbi.nlm.nih.gov/pubmed/27779443

Best Morning Ever

Aug 22, 2018

I remember as a kid when my father, a cardiologist, got his first pager, freeing him from the phone. It only buzzed, so he still had to find a phone to find out the nature of the page, but it did increase his freedom on call. My practice precedes the cell phone, and I have kept my communication with the medical world strictly routed through my pager. I can turn the pager off, but I can't turn off the cell phone. My pager died today, and it was three hours before I could get it replaced, completely separated from the world. Best three hours of my career.

I have mentioned that what drives me is the who, what, where, when, why, and how of diseases. What Richard Feynman called the pleasure of finding things out. I'm no Feynman, obviously, but I understand the drive to understand. I usually rely on cultures to give me that understanding. Everyone lies but the cultures, and if you understand what they are saying, you will understand the disease.

But sometimes I need a Rosetta Stone.

The patient comes in with fevers, rigors and grows Citrobacter in blood and urine. Fine. But then his leg goes cold and painful, and the angiogram shows an acute arterial obstruction which in the OR is clot. But not any clot. I talked with the pathologist, and there were many more WBC in the clot than is usually seen. Septic embolism? Citrobacter endocarditis?

Gram-negative endocarditis is rare, but work-up is pursued. Repeat BC are negative, and TEE is pristine. But as part of the evaluation of the embolic event, a CT was done, and his spleen is mostly replaced with abscess and maybe embolic changes.

Hrm.

Eventually, he needs a splenectomy, and it too grows Citrobacter.

There is one case of Citrobacter splenic abscess I can find and no cases as a complication of urosepsis. So it seems odd. And how to factor in the arterial emboli? Venous thromboemboli are common after infections, but arterial? Not so much. In kids, a URI is a risk for arterial stroke, but I can't find much of anything in adults.

And the patient has no risk factors or history to account for a clotting disorder. So how to put together Citrobacter urosepsis, arterial clot, and a splenic abscess that isn't a just-so story? Am I missing something, or is this one big accumulation of bad luck, hitting the ID Powerball?

I can't really say.

But at least the patient did ok.

Rationalization

Am Surg. 2001 Jan;67(1):80-5. Splenic abscess: report of six cases and review of the literature. https://www.ncbi.nlm.nih.gov/pubmed/11206904 Proc (Bayl Univ Med Cent). 2017 Apr; 30(2): 173174.

Systemic infection and splenic abscess

https://www.ncbi.nlm.nih.gov/pmc/articles/PMC5349817/

Predictors of Cerebral Arteriopathy in Children with Arterial Ischemic Stroke: Results of the International Pediatric Stroke Study.

https://www.ncbi.nlm.nih.gov/pmc/articles/PMC4205969/

POLL RESULTS

I am happiest at work when

  • my beeper goes down 10%
  • my cell phone goes down 24%
  • my internet goes down 7%
  • my emr goes down 17%
  • my hearing aids go down 28%
  • Other Answers 14%
    • Taking a nap
    • I don't get hurt.

Never Heard Of It But I Think I Can Pronounce It

Aug 27, 2018

The patient presents with fevers and chills. As do most of my consults. But he also has a Groshong that had developed a leak, so the reason for the symptoms was evident.

Within 24 hours, the blood cultures are all growing E. faecalis, not high on my list of organisms to cause catheter infections, not even showing up in one review, but it happens:

Sources of bacteremia included urinary tract infections (36.4%), vascular catheters (15.1%), abscesses (9.1%), and unknown (48.5%).

But then everything happens at least once. Except for me winning Powerball.

Then, 36 hours later, one blood culture bottles also grow a yeast. So the line is pulled, and it grows the same Enterococcus. But no yeast. And several days go by with no identification.

I call the lab to ask what it is. The MALDI-TOF called it Candida pararugosa. The biochemicals not so certain.

I never heard of that Candida.

It sounds like a Harry Potter spell, but at least I think I can pronounce it.

I winge all the time in my podcasts that ID papers seldom have a pronunciation guide for the polysyllabic obscure organisms they write about.

How the hell am I supposed to know how to say it? I would have thought P. acnes, now Cutibacterium acnes would be pronounced cutie, but no, there was one article that was kind enough to make it known that

As an aside, the authors who proposed the novel genus name Cutibacterium stated that the name should be pronounced “cootie” or koo-tee, as in a child’s term for an imaginary—or real—germ

Thank you.

It really should be de rigueur that, if nothing else, there should be a pronunciation guide for the organisms at the end of every article. Hear me, editors? Like anyone at CID/JID/OAID etc., reads this blog.

Anyway, back to the Candida in question. One case of fungemia in the PubMeds. That's it.

C. pararugosa is now recognized as a distinct Candida species different from Candida rugosa spp complex. Though morphologically different, misidentification of this species as Candida rugosa (C. rugosa) is common when phenotypic characteristics are analyzed. C. pararugosa has been isolated from various environmental sources and foods and has been mentioned as a potential probiotic for its ability to resist toxic effects of bile and gastric secretions. Human isolations of this yeast were initially from feces and the oral cavity, suggesting a potential role as a colonizer of the GI tract.

So it was likely a fellow traveler sneaking in with the Enterococcus. The one case was resistant to fluconazole but sensitive to micafungin and amphotericin.

Does it even need to be treated? The patient did just fine once the line was removed, and these rare Candida tend to be relatively non-pathogenic,

The pathogenic role that this yeast actually plays in human disease is still uncertain.

but given the host we, and be we I mean I, opted for a short course of micafungin in addition to treating the Enterococcus.

Rationalization

Int J Crit Illn Inj Sci. 2014 Apr-Jun; 4(2): 162–167. doi: 10.4103/2229-5151.134184 Catheter-related bloodstream infections

https://www.ncbi.nlm.nih.gov/pmc/articles/PMC4093967/

J Infect Dev Ctries. 2015 Nov 30;9(11):1195-203. doi: 10.3855/jidc.6587. Clinical and microbiological features of bacteremia caused by Enterococcus faecalis.

https://www.ncbi.nlm.nih.gov/pubmed/26623628

CELEBRATING SUCCESSES AND CONTEMPLATING MESSES IN BACTERIAL TAXONOMY.

https://www.asm.org/index.php/clinmicro-blog/item/6822-celebrating-successes-and-contemplating-messes-in-bacterial-taxonomy

Cureus. 2017 May; 9(5): e1283. Published online 2017 May 29. doi: 10.7759/cureus.1283 Candida pararugosa: First Reported Bloodstream Infection in an Adult

https://www.ncbi.nlm.nih.gov/pmc/articles/PMC5491335/

POLL RESULTS

Every article needs

  • a pronunciation guide for organisms 17%
  • bigger print 12%
  • a cup of very strong coffee 12%
  • cartoons like the New Yorker 17%
  • full text to never be behind a paywall 38%
  • Other Answers 5%
  • all of the above!
  • all of the above!

So Last Century

Aug 30, 2018

I have started a nascent antibiotic stewardship program for my medical system, and it has taken me into areas of medical practice where I am not usually consulted.

My review starts with the antibiotics. Recently I was going through the list, and I came across clindamycin/gentamicin.

Who gives that anymore?

Aminoglycosides have been passé in medicine for 15-20 years. Aminoglycosides do no work well, if at all, in the pus milieu, and, like alcohol, there is likely no safe dose of aminoglycosides, and they lack the pleasant buzz. Unless you find 8th nerve toxicity pleasant. In my world, aminoglycosides are only given when there are no other options. Maybe a bit of an overstatement, but not much.

The nephrotoxic effect of gentamicin is directly related to treatment duration, with a decrease in EECC of 0.5% per day of gentamicin treatment.

And I have also given up on clindamycin. Besides driving C. difficile, I can't trust it for anaerobes:

...clindamycin is no longer recommended as therapy for community-acquired intra-abdominal infections in adults due to increasing resistance among the B. fragilis group. Clindamycin resistance is mediated by erm genes located on transferable plasmids that can also carry tetracycline resistance genes. Worldwide, clindamycin resistance is increasing and approaches 60%

and

clindamycin susceptibility in Prevotella species has been decreasing from 91% in 1993–1994 to 69% in 2012

and

Clindamycin resistance rates among anaerobic GPC range from 7% to 20%, and this resistance is rising especially in Finegoldia magna and Peptoniphilus species.

Also, we can't rely on clindamycin for Streptococci, with clindamycin resistance hovering around 10%.

So when is this standard of care?

Ob. For post-partum endometritis.

Gentamicin/clindamycin is the go-to combination for endometritis, and the 2015 Cochrane review says

The combination of clindamycin and gentamicin is appropriate for the treatment of endometritis.

It is a weird literature because the comparators were penicillins and cephalosporins, neither of which would be expected to be optimal therapy for the likely microbiology. It is like comparing the safety and speed of a 1956 Chevy to a Model T and concluding your best car in 2015 is the 1956 Chevy.

Well, a lot has happened in the intervening 60 years with antibiotics and bacteria.

They do have healthy patients, and most of the cure is in source control, so it is a population I would expect to do well no matter what antibiotic they receive. But there is no way that I can see clinda/gent as optimal therapy.

I am a big proponent of science-based medicine, but I like my science and evidence to be modern. It must be odd to have such a dated literature to rely on.

Rationalization

Clin Infect Dis. 2009 Jan 1;48(1):65-71. doi: 10.1086/594122. Severity of gentamicin's nephrotoxic effect on patients with infective endocarditis: a prospective observational cohort study of 373 patients.

https://www.ncbi.nlm.nih.gov/pubmed/19046065

Antimicrobial Resistance and Susceptibility Testing of Anaerobic Bacteria

https://academic.oup.com/cid/article/59/5/698/2895712

Antimicrobial susceptibility patterns of β-hemolytic and viridans group streptococci: report from the SENTRY Antimicrobial Surveillance Program (1997–2000)

https://www.sciencedirect.com/science/article/pii/S0732889302003747

Cochrane Database Syst Rev. 2015 Feb 2;(2):CD001067. doi: 10.1002/14651858.CD001067.pub3. Antibiotic regimens for postpartum endometritis. https://www.ncbi.nlm.nih.gov/pubmed/25922861